-------------------------------------------------
Theoretische-Physik-FAQ -------------------------------------------------
Tippfehler korrigierte Kopie von:

 http://www.mat.univie.ac.at/~neum/physik-faq.txt Dies ist eine Ergänzung zum englischsprachigen ''theoretical physics FAQ'' auf http://wwwi.mat.univie.ac.at/~neum/physfaq/physics-faq.html
und enthält Antworten zu einigen Fragen der theoretischen
Physik, die ich meinen Beiträgen zur unmoderierten Newsgruppe
de.sci.physik entnommen und editiert habe. Es betrifft Themen,
die bisher nicht im (empfehlenswerten, viel umfangreicheren)
''theoretical physics FAQ'' enthalten sind, weil ich die Fragen
original auf Deutsch beantwortet habe und nicht die Zeit zur
Übersetzung fand.

Das FAQ wächst mit der Zeit.
Die erste Version mit 10 Beiträgen zu allgemeinen Themen
und 17 Beiträgen zur Thermischen Interpretation
stammte vom 01.03.2005; die Version vom 13.03.2006 enthielt
27 Beiträge zu allgemeinen Themen und 35 Beiträge zur
Thermischen Interpretation (ursprünglicher Name:
Konsistent-Experimente-Interpretation); die Version vom
01.10.2007 enthielt 40 Beiträge zu allgemeinen Themen und
37 Beiträge zur Thermischen Interpretation;
seither hinzugekommene Beiträge sind mit einem * im
Inhaltsverzeichnis markiert.

Korrekturen bzw. Verbesserungsvorschläge bitte an mich persönlich
schicken (Arnold.Neumaier@univie.ac.at), Fragen oder Diskussion
bitte in der Newsgruppe de.sci.physik stellen!

Viel Spass und Einsicht beim Lesen!

Arnold Neumaier
Universität Wien
http://www.mat.univie.ac.at/~neum/ Prüft alles, und das Gute behaltet. (Paulus, 1. Thess. 5:21) Angaben wie quant-ph/0303047 verweisen auf Artikel im arxiv, die z.B. von http://xxx.uni-augsburg.de/form aus heruntergeladen
werden können.


------------------
Inhaltsverzeichnis
------------------

(Um ein spezielles Thema zu lesen, suche man nach dem nächsten
Auftreten der Kennzeichnungsnummer, z.B. S2l.
Die Nummern können sich ändern, wenn neue Antworten hinzukommen.
Daher bitte FAQ-Beiträge mit ihren überschriften zitieren!)

S1a. Was ist ein Feld?
S1b. Tensoren oder Matrizen?
S1c. Wie ist die Spur definiert?
S1d. Quantenmessungen
S1e. GNS-Konstruktion und Quantenmechanik
S1f. Welche Mathematik braucht man für die Physik?
S1g. Ist ein Elektron punktförmig?
S1h. Thermodynamik und Gleichgewicht
S1i. Photonen als elektromagnetische Austauschteilchen
S1k. Quantentomographie
*S1l. Quantensprünge
*S1m. Klassische statistische Mechanik

S2a. Physik und Informationstheorie
S2b. Quantensysteme sind praktisch immer verschränkt!
S2c. Nichtlokalität und Kausalität
S2d. Ununterscheidbarkeit
S2e. Was für Beobachtungen erklärt die Quantenmechanik?
S2f. Teilchenspuren und Quantenmechanik
S2g. Quantensysteme sind praktisch immer gemischt!
*S2h. Zeit in der Quantentheorie
*S2i. Kopenhagen und Ensemble-Interpretation
*S2j. Zweierlei Naturgesetze in der Kopenhagen-Interpretation
*S2k. Kollaps und offene Quantensysteme
*S2l. Quantenmechanik von Einzelsystemen
*S2m. Der HERA Speicherring bei DESY als Quantensystem
*S2n. Kollaps und Relativitätstheorie
*S2o. Effektive quantenmechanische Modelle
*S2p. Quantenmechanik von Molekülen
*S2q. Der Hilbertraum des Wasserstoffgases
*S2r. Genauigkeit der QED

S3a. Beobachter, Raum und Zeit
S3b. Was heißt 'homogen' und 'isotrop'?
S3c. Wie homogen und isotrop ist das Weltall?
S3d. Wie soll man sich den gekrümmten Raum vorstellen?
S3e. Universum ohne Urknall?
S3f. Das Alter des Universums
S3g. Woraus besteht das Universum?
S3h. Ist die Welt determiniert?
*S3i. Diffeomorphismeninvariante klassische Mechanik

S4a. Sind alle dynamischen Systeme linear?

S5a. Was sind Anomalien?

S6a. Gibt es Kugelblitze?

S7a. Erklärt die Wissenschaft die Welt?
S7b. Das Uhrmacheruniversum
S7c. Gott und Physik
*S7d. Warum kann die Welt mathematisch beschrieben werden?

S8a. Andere Physik FAQs

S9a. Dank




Die Antworten zu obigen Fragen beziehen sich nur auf die
Erläuterung traditioneller Aspekte der theoretischen Physik.

Sie sind völlig unabhängig von der Diskussion in den nun folgenden
Abschnitten über verschiedene Aspekte meiner neuen und daher sicher
kontroversen Thermischen Interpretation, mit der ich
das Messproblem und den Zufall in der Quantenmechanik erkläre.

Die Anfänge meiner Thermischen Interpretation werden
in der (in diesem FAQ immer mit [EECQ] zitierten) Arbeit
A. Neumaier,
Ensembles and experiments in classical and quantum physics,
Int. J. Mod. Phys. B 17 (2003), 2937-2980.
quant-ph/0303047
http://www.mat.univie.ac.at/~neum/papers/physpapers.html#ensembles http://www.mat.univie.ac.at/~neum/ms/ensembles.pdf mit allen formalen Details beschrieben. Vieles Weitere (insbesondere die Herleitung der Bornschen Regel) ist aber noch unveröffentlicht und wird in diesem FAQ erstmals wenigstens qualitativ beschrieben. Siehe dazu auch A. Neumaier, Optical models for quantum mechanics, Slides of a lecture given on February 16, 2010 at the Institute for Theoretical Physics, University of Giessen, http://www.mat.univie.ac.at/~neum/ms/optslides.pdf und Kapitel 7 meines Online-Buchs Arnold Neumaier and Dennis Westra, Classical and Quantum Mechanics via Lie algebras, 2008. (arXiv:0810.1019)

S10. Die Thermische Interpretation
S11. Wie liest man [EECQ] am besten?
S12. Kernaussagen der Thermischen Interpretation
S13. Motivation für die Thermische Interpretation
S14. Die einzigen Observablen der Physik sind Erwartungswerte
S15. Zwei Arten von Erwartungswerten
S16. Thermische und Kopenhagen-Interpretation
S17. Warum ist niemand vor mir darauf gekommen?

S20. Der Messprozess im Quantenuniversum
S21. Das Quantenuniversum als formales Modell
S22. Ein Modelluniversum
S23. Physikalische Systeme und ihre Messung
S24. Vorhersage im Stern-Gerlach Experiment
S25. Was passiert einzelnen Photonen am Doppelspalt?
S26. Der Quantenradierer in der Thermischen Interpretation
S27. Muss man den ganzen Zustand des Universums kennen?
S28. Kann man den Zustand des Universums falsifizieren?

S30. Wie erklärt sich der Zufall?
S31. Ist der quantenmechanische Zufall objektiv?
S32. Wie fasst man Wahrscheinlichkeitsverteilungen?
S33. Was wird aus dem Superpositionsprinzip?
S34. Spinmessung formal betrachtet
S35. Was ist an Wigner's Analyse idealisiert?
S36. Kollaps als bedingte Wahrscheinlichkeit?

S40. Was sind die Beables der Interpretation?
S41. Was ist ein Erwartungswert?
S42. Was ist eine Präparation?
S43. Was ist eine mikroskopische Messung?
S44. Aber man kann doch einzelne Photonen messen?
S45. Was ist denn eigentlich ein Photon?
S46. Gibt es Probleme mit Lokalität und Bells Ungleichungen?
S47. Wie vertragen sich denn objektive Messwerte und Unitarität?
S48. Wie verborgen sind die 'verborgenen Variablen'?

S50. Wofür steht das Fragezeichen auf S.30 von [EECQ]?
S51. Warum verlangt man (S1) auf S.30 von [EECQ]?



----------------------
S1a. Was ist ein Feld?
----------------------

Ein Feld ist einfach eine Größe, die vom Ort abhängt.
Ein Brötchen in meiner Hand hat eine inhomogene Massenverteilung
und kann daher durch ein skalares Massenfeld rho(x) beschrieben
werden. Dass das Brötchen durch Kräfte zusammengehalten wird,
ist auch klar; sonst würde es ja bei Berührung zu Staub zerfallen.
Diese Kräfte sind ortsveränderlich und gerichtet, bilden also
ein Vektorfeld F(x).

Alle Felder, die die Physik betrachtet, kommen auf ähnliche
Weise zustande. Dass ein paar davon unsichtbar sind
(Gravitationsfeld, elektromagnetisches Feld), ist kein
echtes Problem: Schall (ein Druckfeld) oder Rosenduft (ein chemisches
Feld) sehen wir ja auch nicht, sondern erleben sie nur durch ihre
materiellen Auswirkungen.


Ein Feld ist die mathematische Beschreibung der Tatsache,
dass dort, wo das Feld ist (genauer: von Null verschieden ist),
Kräfte auf ein dafür empfängliches Objekt wirken.

Dass letzteres etwa beim Magnetismus der Fall ist,
leuchtet sofort ein, wenn man einen Magneten hat und ein
Eisenobjekt in die Nähe bringt. Damit kann man die Kräfte
auch ausmessen und so das zugehörige Feld kennenlernen.
Erzeugt werden diese Kräfte (also das Feld) von gewissen
Objekten, die man in diesem Fall Magneten nennt.

Ob man sich nun denkt, ''der Magnet wirkt auf die Ferne
auf das Eisen'', oder ''der Magnet erzeugt in seiner
Umgebung ein Feld, das auf das Eisen wirkt'',
sind zwei unterschiedliche Ansichten desselben Prozesses,
und mit obiger Definition des Feldes experimentell nicht
unterscheidbar. Die letztere hat den Vorteil, dass sich
die physikalischen Gesetzmäßigkeiten lokal formulieren
lassen, und hat sich deshalb als die 'fundamentalere'
durchgesetzt - also aus rein pragmatischen Gründen.

Das 'ist' das Wesen des Felds - mehr gibt es dazu nicht zu
sagen, ohne ins Metaphysische zu geraten. Und mehr braucht
man auch nicht zu wissen, um damit arbeiten zu können.
Mit dieser Beschreibung hat man also ein vollständiges
Verständnis der qualitativen Aspekte.

Wenn man allerdings kompliziertere Vorhersagen machen will,
braucht man etwas mehr Wissen und Verständnis;
das liefern dann die Maxwell-Gleichungen...


Für das Gravitationsfeld und jedes andere Feld gilt
entsprechendes. Allgemein gilt:

Ein Vektorfeld ist genau dort, wo Kräfte wirken,
wenn bestimme Objekte dort anwesend sind.

Für Skalarfelder gilt dasselbe mit der Modifikation,
dass die Kräfte vom zugehörigen Gradientenfeld bestimmt
werden, dass also nur _Unterschiede_ im Feld Kräfte bewirken.
Z.B. das Temperaturfeld: Temperaturunterschiede bringen die
Moleküle in eine kollektiv organisierte Bewegung, sofern sie
sich bewegen können; sonst bewirken sie Stress im Material...
Beides sind Zeichen für vorhandene Kräfte.



----------------------------
S1b. Tensoren oder Matrizen?
----------------------------

Hier ist eine mathematisch rigorose Version der Tensorrechnung,
in der Tensoren zweiter Stufe auf natürliche Weise mit Matrizen
identifiziert werden können.

T^q = Tensorprodukt von q Räumen T;
insbesondere ist T0 die Algebra S der Skalarfelder und T1=T.
T_p^q = Raum aller linearen Abbildungen von T^q nach T^p;
Elemente sind (p,q)-Tensoren mit p oberen und q unteren Indices.
T_0^q = T^q
T_p0 =: T_p = (T^p)^* ist der sogenannte Dualraum von T^q;
insbesondere ist T_1 = T^* der Dualraum von T;
seine Elemente sind die Linearformen = Kovektoren.

Die Äquivalenz mit der traditionellen Definition von Tensoren als
multilinearen Abbildungen von T_q Tensor T^p --> S ergibt sich
daraus, dass zu jedem A in T_p^q kanonisch ein
B: T_q Tensor T^p --> S mit
B(s,t) = t(As) für s in T^q, t in T_p
zugeordnet werden kann, und umgekehrt. Da das Bild As von s unter A
in T^p liegt, ist t(As) wohldefiniert und ein Skalar.

Wenn eine Basis und eine Metrik auf T gegeben ist, ist auch eine
Kobasis (auf T^*) gegeben, und man kann Koordinatenschreibweise
verwenden. Dann schreiben Physiker
- Elemente von T als Vektoren = Spaltenvektoren mit einem oberen Index
- Elemente von T^* als Linearformen = Kovektoren = Zeilenvektoren
mit einem unteren Index
- Elemente von T^q als Multivektoren mit q oberen Indices,
- Elemente von T_p als Multikovektoren mit p unteren Indices,
- Elemente von T_p^q als gemischte Multi/ko/vektoren mit p unteren
und q oberen Indices.

Insbesondere ist

(0,0)-Tensor = Skalar
(1,0)-Tensor = Vektor (Vektor in T=T1) = Spaltenvektor
(0,1)-Tensor = Kovektor (Vektor im Dualraum T^*=T_1) = Zeilenvektor
(1,1)-Tensor = Matrix (lineare Abbildung von T nach T)
Die Spalten der Matrix A_i^k sind dann Spaltenvektoren = Vektoren,
und die Zeilen sind Zeilenvektoren = Kovektoren, und die ganze
Indiziererei ist konsistent.

(Man kann dasselbe Spiel auch treiben, indem man in allen Aussagen
systematisch Zeilen und Spalten vertauscht. Das entspricht dann der
Version, wie Matrizen und Vektoren etwa in misner/Thorne/Wheeler
benutzt werden.)

Kontraktionen gemäß der Einsteinschen Summenkonvention beziehen
sich immer auf Summen über Paare gleich bezeichneter Indices,
und zwar immer genau einer unten und genau einer oben.


Die Mathematiker schreiben in der linearen Algebra (wo keine
Tensoren höherer Ordnung als 2 verwendet werden) aber alle
Indices unten (egal ob Zeilenvektor, Spaltenvektor oder Matrix),
geben alle Summen explizit an, verwenden explizite Buchstaben
nur für Spaltenvektoren und Matrizen, und schreiben bei fester
Basis alle Zeilenvektoren mit Transposition (x^T ist der
Zeilenvektor mit denselben Komponenten wie der Spaltenvektor x).
Das hat viele Vorteile und erlaubt insbesondere eine indexfreie
Notation, die besonders bei längeren Rechnungen den Überblick
deutlich vereinfacht:

Phys. Notation: s = x^k y_k x Vektor, y Kovektor
Math. Notation: s = sum_k x_k y_k
oder einfach s=x^Ty.

Phys. Notation: y_i = A_i^k x_k x,y Vektoren, A Matrix
Math. Notation: y_i = sum_k A_ik x_k
oder einfach y=Ax.

Phys. Notation: y_i = A_i^j B_j^k x_k x,y Vektoren, A,B Matrizen
Math. Notation: y_i = sum_jk A_ij B_jk x_k
oder einfach y=ABx.

Phys. Notation: y_i = A_i^j B_j^k C_k^l D_l^m x_k
x,y Vektoren, A,B,C,D Matrizen
Math. Notation: y_i = sum_jklm A_ij B_jk C_kl D_lm x_k
oder einfach y=ABCDx.

Bei Tensoren höherer Ordnung gehen die Vorteile der Math. Notation
wieder verloren.


In der Riemannschen Geometrie ist eine Metrik g, ausgezeichnet,

Phys. Notation: symmetrischer (0,2)-Tensor g_jk,
g_jk=g_kj,
g(x,y)=x^j g_jk y^k,
Math. Notation: symmetrische Matrix g = g^T,
g(x,y)=x^Tgy = sum_jk x^j g_jk y^k,

Eine metrik ist stets als nichtausgeartet vorausgesetzt; d.h.,
g ist als matrix regulär, und hat daher eine Inverse g^{-1}.
Die zu g gehörige inverse Matrix bestimmt einen (2,0)-Tensor mit
Komponenten g^jk = (g^{-1})_jk:

Phys. Notation: g^jk
Math. Notation: g^{-1}

Mit ihrer Hilfe kann man (kontravariante) Vektoren und (kovariante)
Kovektoren ineinander abbilden:
Phys. Notation: x_k = x^j g_jk, x^j = g^jk x_k
Math. Notation: braucht zwei verschiedene Buchstaben, da die beiden
Vektoren numerisch unterschiedliche Komponenten haben.
Bezeichnet x den (Spalten)Vektor und y^T den
zugehörigen (Zeilen)Kovektor, so ist
y = gx, x = g^{-1}y.
Analog geht man bei mehreren Indizes vor.

Das Hochziehen nur eines Indexes in g ergibt die Einheitsmatrix:
g^j_k = g_j^k = delta_jk (Kroneckersymbol)
denn es ist
g^j_k = g^jl g_lk (Hochziehen des ersten Index)
= (g^{-1})_jl g-lk
= (g^{-1}g)_jk (Summationskonvention und Produkt-Definition)
= (1)_jk (Definition der Inversen)
= delta_jk (Definition des Kroneckersymbol)



--------------------------------
S1c. Wie ist die Spur definiert?
--------------------------------

Wenn der Zustandsraum des gesamten Universums endlich wäre
und N levels hätte, wären S(x) und rho(t) beides
N x N -Matrizen, und die Spur
S(x,t) := trace rho(t) S(x)
wäre die Summe der Diagonalelemente des Matrizenprodukts.

Da aber schon ein einziges Teilchen ein kontinuierliches
Spektrum hat, erst recht also das ganze Universum,
ist N unendlich groß und man braucht Hilberträume,
um das Ganze zu beschreiben. In jedem Hilbertraum
gibt es eine Klasse von Operatoren A, für die die
Spur trace A definiert ist (sogen. Spurklasseoperatoren).

Aber ich will hier nicht die Hilbertraumtheorie erläutern;
dafür gibt es viele Lehrbücher. Ich empfehle, mal in
ein Buch über statistische Mechanik hineinzuschauen und
dort die Anfangsseiten des Quantenteils zu studieren.



---------------------
S1d. Quantenmessungen
---------------------

Eine Messung is eine Multiplikation des Zustands mit einem
Operator P_s mit Wahrscheinlichkeit p_s=|P_s\psi|^2, und
anschließender Reskalierung
psi' = P_s psi/sqrt(p_s).
Dabei sind die P_s durch die experimentelle Anordnung bestimmte
Operatoren mit
sum_s P_s^*P_s = 1.
Dies garantiert dass
sum_s p_s = 1.
(Für Messungen mit nichtdiskreten Messwerten muss man die
Wahrscheinlichkeiten durch Wahrscheinlichkeitsdichten und die
Summen durch Integrale ersetzen.)

Mehrere Messungen entsprechen dem Produkt solcher Einzelmessungen.
Warten von Zeit t bis Zeit t' ohne Messung entspricht der
Multiplikation mit dem unitären P = exp (i(t-t')H) mit
Wahrscheinlichkeit 1. Eine idealisierte von-Neumann-Messung entspricht
dem Fall, dass P ein orthogonaler Projektor ist.

Nachzulesen (in mehr oder weniger ähnlicher Form) in jedem guten Buch
über reale Messprozesse, z.B.
V.B. Braginsky and F.Ya. Khalili,
Quantum measurement,
Cambridge Univ. Press, Cambridge 1992



-----------------------------------------
S1e. GNS-Konstruktion und Quantenmechanik
-----------------------------------------

Mathematische Physiker betrachten Quantenmechanik oft
nicht im traditionellen Hilbertraumgewand, sondern
etwas allgemeiner als Darstellungstheorie von C^*-Algebren.

Die sogenannte GNS-Konstruktion bildet die Brücke zwischen der
Betrachtungsweise der traditionellen theoretischen Physik und der
der mathematischen Physik. Man braucht sie, um einander zu verstehen.
Allein schon deshalb ist sie auch physikalisch relevant. Aber nicht
_nur_ deshalb...


Eine C^*-Algebra ist im Wesentlichen eine Menge E von Objekten,
mit denen man hier mit komplexen Matrizen rechnen darf.
Ein Zustand ist eine lineare Abbildung w:E-->C mit den Eigenschaften
w(1) = 1,
w(A^*A) >= 0 für alle a aus A.
Der Zustand heißt rein (oder extrem), falls er sich nicht als echte
konvexe Kombination anderer Zustände schreiben lässt.

In der C^*-Algebra E der nxn-Matrizen hat jeder reine Zustand die Form
w(A)= psi^* A psi / psi^*psi mit psi ungleich 0.
Insbesondere ist die durch
w(A):=A_11
definierte Abbildung ein reiner Zustand.
In der traditionellen Dirac-Schreibweise ist es der durch |1>
repräsentierte Zustand. Das weiß die C^*-Algebra aber nicht,
da sie keinen Hilbertraum kennt.

Wir definieren nun auf E das Skalarprodukt
<A|B>:= w(A^*B).
Leider ist immer dann <A|A>=0, wenn die erste Spalte von A
verschwindet. Da in einem Hilbertraum nur die Null die Norm Null
haben soll, muss man alle A identifizieren, die sich in der ersten
Spalte nicht voneinander unterscheiden.

Mathematisch gesprochen gibt das denn Faktorraum.
Intuitiv gesprochen ignoriert man einfach alle Spalten außer
der ersten. Die Objekte im Faktorraum sind also im Wesentlichen die
Spaltenvektoren psi, und sie werden durch eine beliebige Matrix
A mit der ersten Spalte psi repräsentiert. Man sieht leicht,
dass das Skalarprodukt nicht vom Repräsentanten abhängt und
gerade das traditionelle Skalarprodukt und damit den Hilbertraum
rekonstruiert.


Die GNS-Konstruktion sagt nun einfach, dass dies nicht Zufall ist,
sondern immer so sein muss, gleich welcher Zustand und welcher
Hilbertraum.

Die C^*-Algebra E der traditionellen Betrachtungsweise
in einem festen Hilbertraum ist einfach die Algebra der
beschränkten Operatoren (also nicht aller traditionellen Observablen);
und daraus bekommt man den Hilbertraum per GNS zurück.

Unbeschränkte Operatoren sind einfach Limiten von Elementen der
C^*-Algebra. Sie sind also immer noch vorhanden, nur nicht mehr
als primäre Objekte. Zum Beispiel liegen die unitären Matrizen U(t),
die die Zeitentwicklung beschreiben, in E, der Hamilton-Operator
H := lim_[t to 0} (U(t)-1)/it
aber nicht. Sämtliche Observablen entstehen auf diese Weise als
infinitesimale Erzeugende von unitären Einparametergruppen in E.


In der N-Teilchenmechanik bekommt man so auch nichts Neues,
sondern genau das Alte in abstrakterem Gewand. Die Hamiltonsche
Dynamik erscheint im neuen Gewand einfach als eine 1-Parameterschar
von inneren Automorphismen der C^*-Algebra.


Aber in der nichtrelativistischen Feldtheorie - und dazu gehört
die gesamte statistische Mechanik im thermodynamischen Limes
mit dazu! - bekommt man etwas flexibleres.

Denn dann sind die kanonischen Kommutatorrelationen nicht mehr
eindeutig repräsentierbar, sondern haben nichtäquivalente
Darstellungen. Und die Dynamik spielt sich nicht mehr in einem
einzigen Hilbertraum ab, was sich darin äußert, dass die
Zeitentwicklung statt durch innere Automorphismen
A(t) = U(t)A(0)U(t)^*
jetzt durch äußere Automorphismen gegeben sind, die
die Darstellungen vermischen.

In der statistischen Mechanik spiegelt sich das in der Schwierigkeit,
den thermodynamischen Limes zu vollziehen. Man umgeht dort gewöhnlich
die Probleme, indem man sich zuerst auf endliches Volumen beschränkt,
damit man weiter in _einem_ Hilbertraum arbeiten kann.


Das gilt schon im Nichtrelativistischen (wo man das meiste versteht)
und wird im Relativistischen soweit verschärft, dass man bisher
den relativistischen Fall nur in Dimensionen 2 und 3 versteht.

Die ungelösten Probleme liegen genau hier - der thermodynamische
Limes mutiert zum Infrarotproblem der QFT, das für die starke
Wechselwirkung (Confinement) nicht einmal auf dem nicht rigorosen
Niveau der theoretischen Physik verstanden wird.


---------------------------------------------------
S1f. Welche Mathematik braucht man für die Physik?
---------------------------------------------------

Ich würde empfehlen, sich am Bronstein (Taschenbuch der Mathematik) zu
orientieren - der enthält alles, was man an Grundlagen wirklich
braucht, auf knappem Raum zusammen. D.h. gerade soviel
Funktionalanalysis, Funktionentheorie oder Maßtheorie wie unbedingt
nötig ist. (Die Vorlesungen dazu machen es natürlich viel
gründlicher und mit mehr Stoff, was für die einen interessant
und für die anderen Ballast ist.) Damit kann man sich dann leicht
in Originalliteratur oder weiterführende Bücher einarbeiten,
wenn es nötig ist.

Übung im Stoff des Bronstein muss man sich natürlich anderweitig
besorgen, da es kein Lehrbuch ist.

Insbesondere lernt man Numerik (und das zugehörige Programmieren)
nur durch Tun!


Für echte Grundlagenforschung in der theoretischen Physik
(d.h., wenn man wirklich die anstehenden Probleme lösen will)
kann man aber nicht genug Bereiche der Mathematik verstehen!
Jedenfalls ein Stück weit.
Vor allem die konzeptuelle Basis ist wichtig, und dass man die
Hauptsätze und Hauptbegriffe wirklich in ihrer Bedeutung erfasst hat.

Denn man weiß nicht, was die Lösung bringen wird. Die traditionelle
mathematische Schmalspurausbildung der Physiker jedenfalls nicht,
sonst wären diese Probleme schon gelöst!

Ein paar grundlegende offene Fragen sind in den Abschnitten
S14a. Theoretical challenges close to experimental data
S9b. Is QED consistent?
S10c. Difficulties in quantizing gravity
aus meinem theoretical physics FAQ auf
http://www.mat.univie.ac.at/~neum/physics-faq.txt angesprochen (leider auf Englisch). Was ''verstehen'' und ''Bedeutung erfassen'' genau besagt, ist schwer in ein paar Worten zu erläutern. Man braucht nicht alle Beweise verstanden zu haben, um ein Verständnis der Begriffe zu haben. Aber man sollte genug durchgearbeitet zu haben, um ein zweites Buch über dasselbe Thema diagonal lesen zu können, und daraus schon soviel Information zu gewinnen, dass man beurteilen kann, ob und ggf. was daraus noch wichtig zu lesen wäre. Man muss genug gesehen haben, um zu wissen, wie die Begriffe verwendet werden, welche dauernd vorkommen (und deshalb genau verstanden werden wollen) und welche peripher sind (nur sporadisch und an unbedeutender Stelle vorkommend) und daher nicht so wichtig. Man muss wissen, wie die Begriffe auf außerhalb der Disziplin liegende Sachverhalte angewandt werden. (Z.B. in der Differentialgeometrie: Was bedeuten alle Begriffe, wenn man die Mannigfaltigkeit auf die Vollkugel und die Kugeloberfläche spezialisiert?) Man muss sich soviel eingearbeitet haben, dass man selbst ein Gefühl dafür hat, was wichtig und unwichtig ist, also etwa _warum_ ein Satz Hauptsatz genannt wird und ein anderer nur ein Lemma. Das hat in der Regel seine inneren Gründe... Wenn man den Eindruck bekommt, 'jetzt passiert eigentlich nur noch dasselbe in Grün', weiß man, dass man das Wesentliche hat. ----------------------------------- S1g. Ist ein Elektron punktförmig? ----------------------------------- Genau gesagt, gibt es einzelne Elektronen gar nicht, sondern nur ein Quantenfeld vom Spin 1/2, das alle (ca. 10^80?) Elektronen im
Universum gleichzeitig beschreibt. Der Versuch, daraus einzelne
Elektronen zu identifizieren, führt (ähnlich dem Versuch, in einem
turbulenten Flüssigkeitsfeld einzelne Wirbel oder Wellen zu
identifizieren) auf solche Paradoxien wie die Ununterscheidbarkeit
der Elektronen.

Ein einzelnes Elektron ist einfach ein bequemes Kürzel für eine
lokale Konzentration des Elektronenfelds in einem interessierenden
Bereich des Raums, wenn es sich bei Ignorieren der Umgebung als
1-Elektron-Zustand des Elektronenfelds interpretieren lässt.
Die Zahl der Elektronen ist eine Kenngröße des Zustands
(d.h. der Wellenfunktion) und _nicht_ des Quantenfelds.

Je gröber die Approximation, desto mehr Objekte erscheinen
punktförmig. Unserer Sonne sind wir zu nah, aber alle Sterne
in der Magellanschen Wolke, unsrer Nachbargalaxie, sind punktförmig.
So ähnlich wie wir unsere Sonne als bequemes Kürzel für die
größte lokale Konzentration des Wasserstofffeldes in unserer
Nähe (sagen wir bis 1 Lichtjahr Abstand) ansehen können, und
weit entfernte Sterne als punktförmige Lichtquellen, erscheint
ein Elektron im gleichen Sinn punktförmig - der aus der QED
folgende Ladungsradius eines einzelnen physikalischen Elektrons
liegt weit unterhalb der Nachweisgrenze. In Wirklichkeit
sind beides, Stern und Elektron, ausgedehnte Quantenobjekte,
lokale Manifestationen des Zustands einiger weniger Quantenfelder,
die durch das Standardmodell beschrieben sind.

Eine detailliertere Diskussion des Ladungsradius findet sich im Beitrag
''Are electrons pointlike/structureless?''
meines theoretical physics FAQ auf
http://www.mat.univie.ac.at/~neum/physics-faq.txt ------------------------------------ S1h. Thermodynamik und Gleichgewicht ------------------------------------ Die Thermodynamik beschreibt das Verhalten makroskopischer Systeme im Gleichgewicht (statisch) und in Veränderung (dynamisch), obwohl in den gängigen Darstellungen dies oft nicht klar herausgearbeitet wird. Im Gleichgewicht stehen alle Zustandsgrößen in festen Beziehungen, die aus der Zustandsgleichung durch die traditionellen Umformungen gewonnen werden können. Im Nichtgleichgewicht gilt das nicht mehr. Denn dann braucht man zur vollständigen Beschreibung des Systems orts- und zeitabhängige Felder, die die örtliche und zeitliche Variation der thermodynamischen Größen beschreiben. Der Physiker oder Ingenieur, der (z.B.) ein Gas beobachtet, weiß, dass das Gas im Gleichgewicht ist, wenn er lange genug gewartet hat (bei konstanten Randbedingungen), und dass dann alle intensiven Variablen konstante Werte haben, und sich die extensiven daraus bis auf einen Proportionalitätsfaktor ausrechnen lassen. Das ergibt sich aus den Formeln der statistischen Mechanik. Es ist einfach eine Folge vom Gesetz der großen Zahlen. Von reversiblen Vorgängen spricht man, wenn man ein System die ganze Zeit über als im Gleichgewicht befindlich betrachten kann. Dann gelten die Formeln der Gleichgewichtsthermodynamik, und die Energie bleibt erhalten. Die Änderungen der Energie genügen daher der traditionellen Bilanzgleichung. Von irreversiblen Vorgängen spricht man, wenn das nicht der Fall ist, d.h., wenn mindestens eine der intensiven Größen (also etwa die Temperatur, der Druck, oder das chemische Potential) ortsabhängig sind. Insbesondere gilt das * wenn man die Randbedingungen verändert (z.B. Erhitzen einer kalten Flüssigkeit; Erhöhen des Drucks, usw.) * wenn man zwei Gleichgewichtssysteme, die in verschiedenen Zuständen sind, miteinander in Kontakt bringt (Wegziehen einer Schiebewand, Zugeben einer Chemikalie, usw.) * bei chemischen Reaktionen. Dann gelten die Formeln der Gleichgewichtsthermodynamik nicht, und
die Energie bleibt nicht erhalten. (Der Energieerhaltungssatz wird
verletzt oder nicht, je nachdem, wie man die Bilanz aufsetzt.
Es hängt von den Randbedingungen ab, die man im Experiment aufrecht
erhält. Die Energie des Systems, die verloren geht, findet sich
natürlich in seiner Umgebung wieder, aber das hat man normal nie
vollständig unter Kontrolle, daher geht für die Praxis Energie
verloren.)

Die Änderungen der Energie genügen daher nicht der traditionellen
Bilanzgleichung, sondern nur einer entsprechenden Ungleichung, da
Energie 'dissipiert', d.h. verloren geht - in unbeobachtbare
mikroskopische Details der Molekularbewegung. Die Thermodynamik
erfasst nämlich nur das mittlere Verhalten.
Diese Ungleichung ist der Inhalt des zweiten Hauptsatzes in
seiner allgemeinsten Form. Je nach den zusätzlich angenommenen
Randbedingungen ergeben sich als Spezialfälle unterschiedliche
Formulierungen, mit denen der Anfänger konfrontiert wird. Etwa:
* In einem isolierten System nimmt die Entropie nie ab.
* Bei fester Temperatur nimmt die freie Helmholtz-Energie nie zu.
* Bei fester Temperatur und festem Druck nimmt die freie Gibbs-Energie
nie zu.
In diesen Fällen braucht man zur korrekten Beschreibung des Systems
dynamische Kenntnisse (z.B. Navier-Stokes-Gleichungen).
Die Gleichgewichtsthermodynamik kann in diesen Fällen höchstens den
Endzustand vorhersagen - nämlich falls das System von der Dynamik
wieder in einen Gleichgewichtszustand getrieben wird, was wegen der
Dissipation oft der Fall ist.

Nach Wiedererreichen des Gleichgewichts (zur Zeit t2) gelten wieder die
allgemeinen Zustandsgleichungen, etwa S2=S(T2,P2,mu2) für die Entropie
wie im Gleichgewicht am Anfang (zur Zeit t1), etwa S1=S(T1,P1,mu1).
Also kann die Entropie nach wie vor aus T,P,mu ausgerechnet werden,
ist also durch den Zustand vollständig bestimmt. Aber...

Bei konstanter Temperatur (T2=T1=T) gilt für eine reversible Änderung
S2=S1, also können Druck und chemisches Potential nicht beliebige
Werte angenommen haben, sondern nur solche, für die S2=S(T,P2,mu2)=S1
ist. In einem irreversiblen Prozess können dagegen P2 und mu2 im
Prinzip alle Werte mit S2=S(T,P2,mu2)>=S1 annehmen. Die Differenz
T(S2-S1) ist die irreversibel verlorengegangene Energie. Ihr Wert wird
nicht durch die Gleichgewichtsthermodynamik vorhergesagt, sondern
nur durch eine Theorie des Nichtgleichgewicht.

Der Wert der Entropie beim Wiedererreichen des Gleichgewichts hängt
daher zwar nur (wie bei reversiblen Vorgängen) davon ab, welche Werte
die intensiven Variablen am Ende haben, aber ihr Wert ist (anders als
bei reversiblen Vorgängen) größer als vorher.

Verändert sich auch die Temperatur (und jegliche Wärmeleitung ist
schon Nichtgleichgewicht), so ist allerdings eine kompliziertere Bilanz
nötig (Navier-Stokes-Gleichungen, Reaktions-Diffusionsgleichungen,
usw.) Das Problem ist daher unterbestimmt, wenn man
nur die Ausgangsdaten und die zugeführte Wärme kennt.
Die zugeführte Wärme entscheidet erst zusammen mit den Details der
Dynamik darüber, welcher Zustand erreicht wurde, und daher, wie
gross die Entropie ist.

Um aber eine Einschätzung des Ergebnisses zu bekommen (z.B. für eine
Abschätzung des maximalen Wirkungsgrads) tut man zunächst so, als
sei der Prozess reversibel - dann lässt sich ein idealisiertes
S2_rev ausrechnen, und betrachtet die Differenz S2_irr = S2-S2_rev,
die nach dem 2. Hauptsatz stets positiv ist, als den von der
idealisierten Rechnung ignorierten irreversiblen Anteil der Entropie.
Das ist aber ein reiner Buchhaltungstrick, und hat nichts mit den
physikalischen Vorgängen zu tun.

Dass man also sehr viel mit Gleichgewichtsthermodynamik machen kann,
liegt daran, dass man nur den Anfang und das Ende vergleicht, und
dann Ungleichungen bekommt, die einem das theoretisch denkbar
Günstigste (d.h. reversibel vorhergesagte) liefert. In der Praxis
ist man nie so gut, hat daher einen geringerer Wirkungsgrad. Der durch
Gleichgewichtsthermodynamik nicht vorhersagbare Teil ist eben der
irreversible.

Jede Änderung ist in der Praxis irreversibel, also darf man da die
Gleichgewichtsregeln streng genommen gar nicht mehr anwenden.
Wenn man aber genügend vorsichtig zu Werke geht und wegen dem
verbleibenden Rest ein Auge zudrückt, kann man so tun, als sei man
jeden Moment in einem neuen Gleichgewicht.

Für die Änderungen gelten im Nichtgleichgewicht also nur
Ungleichungen. Je weniger Dissipation, desto eher kann man sie durch
die Gleichungen approximieren, die man aus der Gleichgewichts-
Betrachtung bekommt. Die Idealisierung auf Null Dissipation nennt
sich reversibel. In Wirklichkeit ist aber kein Vorgang exakt reversibel
realisierbar.

Bei einem zeitabhängigen Vorgang besteht das Problem oft darin,
vorherzusagen, wie sich p und T im Lauf der Zeit ändern.
Beliebige Zustandsgrößen f(p,T) ergeben sich (bei konstanter
Teilchenzahl) daraus dann immer eindeutig. Aber bei einem reversiblen
Vorgang bleibt die dem Experiment entsprechende Erhaltungsgröße
(je nachdem Entropie, Helmholtzenergie, Gibbsenergie)
konstant, und das schränkt die möglichen Werte von T und p ein.
Man kontrolliert einen der Parameter, der andere reagiert darauf.

Bleibt z.B. die Helmholtzenergie A(T,V) fest, und ändert man T
reversibel, so ändert sich auch V derart, dass A konstant bleibt,
und p kann man nach den üblichen Formeln aus T und V und der Formel
für A ausrechnen. Darin liegt die Vorhersagekraft.

Bei einem irreversiblen Vorgang nimmt A ab (Energieverlust durch
Dissipation), und man kann aus der Änderung von T weder V noch p
ausrechnen (sondern bekommt nur Ungleichungen dafür). Wenn man
allerdings V misst, ist P wieder dadurch festgelegt, nach derselben
Formel.

Im reversiblen Fall besteht also ein Zusammenhang zwischen p und T,
der durch den Anfangszustand festgelegt ist; im irreversiblen Fall
aber nicht.


------------------------------------------------------
S1i. Photonen als elektromagnetische Austauschteilchen
------------------------------------------------------


Auf der formalen Ebene kann man die Quantenmechanik der
elektromagnetischen Wechselwirkung beschreiben, indem man das Photon
als Austauschteilchen der Wechselwirkung sieht. Wie soll man sich das
vorstellen?

Nicht als einen tatsächlich passierenden Ablauf, wie es
http://solstice.de/grundl_d_tph/sm_ww/sm_ww_01.html zu tun versucht, sondern als Bildchen in der formalen Beschreibung der Wechselwirkung durch Feynman-Diagramme. Die Bildchen stellen keine Ereignisse dar, sondern sind eine Abkürzung für komplizierte Mehrfachintegrale, die hinzuschreiben außer in den einfachsten Fällen zu mühsam wäre. Deshalb heißen die Austauschteilchen auch ''virtuell'' (d.h. fiktiv), im Unterschied zu den ''reellen'' Teilchen, die sich wirklich messen lassen. Die Physiker lieben es, ihre abstrakten Gedankengänge mit Vokabeln, die eine Vorstellung suggerieren, etwas konkreter aussehen zu lassen, aber allzu ernst soll man diese Vorstellungen nicht nehmen. Auch die Mathematiker lieben das, weil auch ihre Theorien abstrakt sind. Aber kein Mathematiker denkt beim Körper der reellen Zahlen, die reellen Zahlen seien ein Tier oder Geist mit einem Körper, und den Ring der ganzen Zahlen stellen sie sich auch nicht als rund vor... ----------------------- S1k. Quantentomographie ----------------------- Zur Quantentomographie gibt es ein hübsches Büchlein von Ulf Leonhardt, Measuring the Quantum State of Light, Cambridge Univ. Press 1997. Wer lieber etwas online dazu lesen will, findet einen Überblicksartikel in http://arxiv.org/pdf/quant-ph/0302028 (was allerdings nicht so leicht genießbar ist). Hier sind die Details für den Fall eines n-Level-Quantensystems, gegenüber obigen Referenzen soweit vereinfacht, dass es jeder Abiturient verstehen sollte (auf die Singuläre-Werte-Zerlegung, die sicher kein Schulstoff ist, kann man notfalls auch verzichten...). 1. Ich habe einen unbekannten reinen Zustand psi in C^n, der von einer stationären Quelle produziert wird und von dem ich zeigen will, dass er verschränkt ist (wenn er es ist). 2. Durch geeignetes Filtern kann ich ihn in den Zustand T psi transformieren, wobei T eine beliebige lineare Transformation ist. 3. Anschließend messe ich z.B. eine binäre Eigenschaft, die durch einen Projektionsoperator P = P^* = P2 gegeben ist. Ich bekomme - in einem Zeitintervall, das sich durch Vergleich mit einer trivialen Messung, die die Intensität des Strahls bestimmt, ergibt - den Wert 1 mit einer Wahrscheinlichkeit von p_T = |P T psi|^2/|T psi|^2. (Eigentlich hat man ja einen Poissonprozess mit einer entsprechenden Eintrittsrate, und es können in den Messzeitraum auch mal 2 Detektionen fallen. Das muss man natürlich entsprechend mitzählen.) Durch genügend häufige Wiederholung kann ich p_T beliebig genau bestimmen. 4. Nun habe ich eine Beziehung der Form |P T psi|^2 = p_T |T psi|^2 die sich als <T^*P T - p_T T^*T> = 0 (1_T) schreiben lässt, unter Benutzung der allgemeingültigen Beziehung |A*psi|^2 = <A^*A>. 5. Jetzt wiederhole ich dieselbe Messreihe mit einer Reihe von anderen linearen Transformationen T. Jedesmal bekomme ich eine neue Gleichung (1_T). 6. Die Operatoren 1 und B_T = T^*P T - p_T T^*T spannen bei geeigneter Wahl der T's (Übungsaufgabe) den n^2-dimensionalen (reellen) Vektorraum aller Hermitischen Operatoren auf. Daher kann man jeden Operator A als Linearkombination solcher Operatoren darstellen und wegen <1>=1 und (1_T) den Erwartungswert von A bestimmen. Damit hat man alles, was nötig ist, umz.B. eine Verletzung der Bellschen Ungleichung nachzuprüfen. Da man schon ein ganzes Quantentomogramm aufgenommen hat, bekommt man mit diesem Protokoll sogar vollständige Information über den Zustand: 7. Die Gleichungen (1_T) stellen ein homogenes lineares Gleichungssystem für die zehn unabhängigen Komponenten der Dichtematrix rho = psi psi^* dar. Bei geeigneter Wahl der T's hat der Lösungsraum die Dimension 1 (z.B. mit 100% Wahrscheinlichkeit bei n^2-1 zufällig gewählten T's) und bestimmt daher rho bis auf einen Skalierungsfaktor, der aber durch die Normierungsbedingung <1>=1 festgelegt ist. Damit habe ich die Dichtematrix rho bestimmt. War das System wirklich rein präpariert, so hat dieses rho notwendig den Rang 1 und damit die Form rho = psi psi^*, mit psi in Richtung einer der Spalten von rho. Damit habe ich ein vollständiges Messprotokoll für die Bestimmung des Zustands psi. 8. Bei endlich vielen Rohmessungen gilt natürlich (1_T) nur approximativ; in diesem Fall misst man p_T für mehr als n^2-1 verschiedene T's, bestimmt die Koeffizienten von rho als singuläre Vektoren zum kleinsten singulären Wert der Koeffizientenmatrix der (1_T), und psi als singuläre Vektoren zum grössten singulären Wert von rho. Bei unabhängigen Messfehlern erhält man nach dem Gesetz der großen Zahlen den tatsächlichen Zustand mit einer Genauigkeit von O(1/sqrt(N)). -------------------- S1l. Quantensprünge -------------------- Vielfach wird behauptet, Quantensprünge gäbe es nicht. Zu dieser Schlussfolgerung kam z.B. ein bekannter Vortrag von Schrödinger, den man in http://www.mikomma.de/schroe/quantumjumps.htm nachlesen kann. Der Vortrag stammt allerdings aus dem Jahr 1952 und ist längst überholt. Heutige Quantenoptiker beobachten und analysieren Quantensprünge routinemäßig. Siehe etwa RG Hulet, DJ Wineland, JC Bergquist, WM Itano Precise test of quantum jump theory Phys. Rev. A 37, 4544 - 4547 (1988) oder N Gisin, PL Knight, IC Percival, RC Thompson, and DC Wilson Quantum State Diffusion Theory and a Quantum Jump Experiment Journal of Modern Optics 40, 1663 (1993) Eine vielzitierte Arbeit ist MB Plenio, PL Knight The quantum-jump approach to dissipative dynamics in quantum optics Rev. Mod. Phys. 70, 101 - 144 (1998). ------------------------------------- S1m. Klassische statistische Mechanik ------------------------------------- Die folgende deterministische Theorie klassisch ununterscheidbarer Teilchen reproduziert die klassische statistische Mechanik mit korrekter Mischungsentropie. Observable sind Folgen f=(f_N, N=0,1,2,...) von Funktionen f_N von p_k und q_k (k=1:N), die unter Vertauschungen der Indizes invariant sind. Die Dynamik ist gegeben durch df_N(t)/dt = {f_N(t),H_N}, (1) wobei H_N die übliche N-Teilchen-Hamiltonfunktion und {.,.} die N-Teilchen-Poissonklammer ist. Offensichtlich ist dies eine deterministische Theorie für eine unbestimmte Anzahl von ununterscheidbaren klassischen Punktteilchen. Die deterministische Dynamik lässt den gemischten Zustand rho = Z^{-1} e^{-(beta H_N + mu N), der offensichtlich permutationsinvariant ist, invariant. Ein in diesem Zustand präpariertes grosskanonisches Ensemble von Systemen bleibt also in diesem Zustand; es liegt Gleichgewicht vor. Aus der Partitionsfunktion Z = sum_{N=0}^inf (h^3N/N!)^{-1} Z_N, (2) Z_N = integral dp^N dq^N e^{-(beta H_N + mu N), wobei h eine im Prinzip experimentell bestimmbare universelle Konstante ist, ergibt sich auf traditionelle Weise die gesamte Gleichgewichtsthermodynamik, mit korrekter Mischungsentropie. Das Postulieren von (1) ist analog zum Postulieren von dA(t)/dt = i/hquer [H,A(t)] (1') für Operatoren in der Quantenmechanik. Das Postulieren von (2) ist analog zum Postulieren von Z = sum_l e^{-(beta H_l+ mu N_l) (2') in der Quantenmechanik, wobei (H_l,N_l) das gemeinsame Spektrum von H und N durchläuft. ----------------------------------- S2a. Physik und Informationstheorie ----------------------------------- Interpretiert man Quantenmechanik oder Thermodynamik im Lichte der Informationstheorie, so schleichen sich leicht Aussagen ein, die sich nach Subjektivität anhören - Bestimme Dinge hängen anscheinend davon ab, was der Beobachter weiß, misst, zur Kenntnis nimmt, oder vergisst. Abgesehen davon, dass solche Begriffe in einer als objektiv geltenden Wissenschaft nichts zu suchen haben, sollte wenigstens eine fundamentale Theorie wie die Quantenmechanik genau definieren
können, was 'Wissen', 'Kenntnisse', usw., sofern sie einen Einfluss
auf das Geschehen haben, eigentlich sind,


Wie sich ein Experiment verhält, hängt sicher nicht davon ab,
was der davorstehende Experimentator weiß oder vergisst,
sondern nur davon, wie das Experiment aufgebaut ist.

Nehmen wir an, zwei Physiker stehen vor demselben Doppelspaltexperiment,
das vor ihren Augen abläuft, ohne dass sie nach der Präparation
darin eingreifen. (Das Interferenzmuster bei einem Doppelspaltversuch
hängt im Allgemeinen nicht davon ab, wie viele Leute sich den Schirm
anschauen.) Währenddessen verwirft einer einige seiner Kenntnisse
über das System, der andere nicht.

Es wäre sehr verwunderlich, wenn dann der eine ein anderes
Interferenzbild sehen würde als der andere.

Objektive Physik muss unabhängig vom Beobachter und seinen
Kenntnissen formulierbar sein.


Wenn der zweite Physiker (oder ein Apparat) von
jedem Lichtblitz weiß, ob er von einem linken oder rechten
Photon erzeugt wurde, so liegt das an der experimentellen Anordnung,
und keiner der beiden Physiker sieht ein Interferenzmuster.

Wenn ein Interferenzmuster entsteht, kann man also mit Sicherheit
prinzipiell von keinem der Lichtblitze wissen, ob er von einem
linken oder rechten Photon erzeugt wurde.

Das ist stock-konservative Kopenhagener Interpretation!

Der zweite Physiker kann sich natürlich einbilden, es zu wissen,
und dann die zwei Computerbilder vorzeigen. Das nennt man dann aber
Phantasie, Wahnsinn oder Betrug. Soll's ja manchmal geben...

Ein Apparat kann das natürlich nicht. Daher können Apparate
nicht wahnsinnig werden und nicht betrügen (außer wenn sie von
Menschen gesteuert oder falsch programmiert werden).

Tja, das wirft noch ganz andere Fragen auf.
Sind wir Menschen weniger oder mehr als Apparate? Und warum??



-----------------------------------------------------
S2b. Quantensysteme sind praktisch immer verschränkt!
-----------------------------------------------------

Zwei disjunkte Teilsysteme eines Quantensystems heißen
miteinander verschränkt, falls ihr gemeinsamer Zustand sich
nicht als Tensorprodukt der Zustände der beiden Teilsysteme
schreiben lässt. So wird dieser Begriff von allen
Quanteninformationstheoretikern benutzt. Es gilt nun ganz
allgemein:

Quantensysteme sind mit Wahrscheinlichkeit 1 mit ihrer Umgebung
verschränkt.


Nehmen wir mal an, ein System S und seine Umgebung R seien
zum Zeitpunkt t=0 unverschränkt, also in einem Zustand
rho_ = rho_S tensor rho_R.
Da das Universum
U = S union R
das einzige abgeschlossene physikalische System ist, das S enthält,
muss im Fall, dass S nicht ganz U ist, S mit R wechselwirken. Also
hat der Hamiltonoperator die Form
H = H_S tensor 1_R + 1_S tensor H_R + sum_l V^l_S tensor W^l_R
mit einer nichttrivialen Summe. Zum Zeitpunkt t>0 ist der Zustand
dann
rho_t = exp(-itH/hbar) rho_0 exp(itH/hbar).

Nun ist es eine einfache Übungsaufgabe, zu verifizieren, dass das
System für kleine t>0 notwendig mit der Umgebung verschränkt ist,
falls nicht ganz spezielle Voraussetzungen vorliegen, die
unter Störungen nicht stabil sind und daher nicht präpariert
werden können. ('Einfach' auf der Ebene, in der Physiker gewöhnlich
argumentieren. Mathematisch rigoros ist das nicht mehr ganz trivial,
sondern man muss schon sorgfältig argumentieren - im
Wechselwirkungsbild und unter gewissen Kompaktheitsvoraussetzungen
an die Wechselwirkungen.)

Falls zufällig mal noch ein Zeitpunkt t_1 eintreten sollte,
wo das System unverschränkt ist, gilt ein analoges Argument
für Zeiten t mit kleinem t-t_1. Es gibt also ein Kontinuum von
Zeitpunkten wo das System mit seiner Umgebung verschränkt ist,
und höchstens eine diskrete Menge von Zeitpunkten, wo dies
nicht der Fall ist.

Daher ist das System mit Wahrscheinlichkeit 1 zu allen Zeiten
mit seiner Umgebung verschränkt.


Überhaupt ist Verschränkung von Systemen das Generische
(Typische), und Fehlen von Verschränktheit bei Teilsystemen
mit gemeinsamer Vergangenheit die Ausnahme.

Man kann diese Verschränktheit unter sorgfältig gewählten
Bedingungen gezielt präparieren, und auf diese Weise z.B.
Aspekt-artige Experimente über viele Kilometer durchführen,
die die Verletzung der Bellschen Ungleichungen bestätigen.

Unverschränkte System präpariert man dagegen, indem man
unabhängig gewonnene Teilsysteme koppelt.




------------------------------------
S2c. Nichtlokalität und Kausalität
------------------------------------

Kausalität ist 'Ausbreitung im Vorwärtslichtkegel',
und eine rein dynamische Angelegenheit.
Nichtkausalität bedeutet, dass, wenn jemand am Punkt A etwas
tut, er damit etwas an einem beliebig weit entfernten Punkt B
instantan (oder wenigstens mit Überlichtgeschwindigkeit)
beeinflussen kann, d.h., dass die Folgen einer lokalen
Änderung der Lagrangefunktion des Systems, die durch eine
externe Handlung am Punkt A bedingt ist, sich instantan
(oder wenigstens mit Überlichtgeschwindigkeit) im Punkt B
spürbar sind.

Lokalität ist eine kinematische Angelegenheit. Nichtlokalität
(d.h. Fehlen von Lokalität) bedeutet, dass es nichtklassische
Korrelationen zwischen beliebig weit entfernten Objekten A und B
geben kann.

(Nicht damit verwechselt werden sollte der quantenfeldtheoretische
Begriff der Lokalität, der durch ein Verschwinden der
Kommutatoren lokaler Felder an jedem Paar raumartig
zueinander liegender Punkte charakterisiert ist.
Bells Ungleichungen und die dadurch implizierte Nichtlokalität
vertragen sich vollkommen mit der quantenfeldtheoretischen
Lokalität und Kausalität, wie sie etwa durch die Wightmanaxiome
spezifiziert sind.)



Die Korrelation ist eine mathematische Größe, die nichts mit
Dynamik zu tun hat. Sie liegt dann vor, wenn man sie entweder
aus der Theorie oder aus Daten ausgerechnet hat. Zum Vergleich
zwischen Theorie und Wirklichkeit braucht man natürlich beides.

Das Entscheidende ist, man kann sie erst dann aus den messbaren
Daten ausrechnen, wenn man die Daten beider Seiten vorliegen hat.

Also dann, wenn sich A und B treffen oder telefonieren, oder
sonst wie auf normale, kausale Weise die entsprechende Information
übertragen haben. (Und das geht nicht schneller als das Licht!)
Dann rechnet man, vergleicht mit der Theorie, und stellt fest:
''Tatsächlich, die Korrelationen sind genau so [nichtlokal],
wie es die Quantenmechanik vorausgesagt hat!''

Aber die Vorhersagen sind zu wenig informativ, um es B zu erlauben,
aus seinen eigenen Messungen auch nur das Geringste zu schließen,
was A getan hat. Daher kann B seinen Messungen keine Information
über A entnehmen.

Außer natürlich solchen, die kausal gesendet wurden, oder
solche, die eine gemeinsame Ursache in der Vergangenheit haben.
Mehr dazu von Google unter dem Stichwort ''Bertlmanns Socken -Dr''.
(Das '-Dr' schließt Informationen über ein Buch mit dem Titel
''Dr. Bertlmanns Socken'' aus, das auch zum Thema gehört,
aber alle andere Information dominieren würde...



-------------------------
S2d. Ununterscheidbarkeit
-------------------------

Ein Ensemble im Sinn der statistischen Mechanik wird
gewöhnlich gedacht als eine unbegrenzte Menge fiktiver (oder
manchmal - nämlich wenn sie gemessen werden - auch realer)
Kopien des einen zu analysierenden Systems.
Kopien, die in den relevanten Größen übereinstimmen, sich
aber in den irrelevanten mikroskopischen Details unterscheiden.
Dies erlaubt es, über die irrelevanten Details zu mitteln
und so relevante statistische Aussagen über relevante Größen
zu bekommen.

Das ist völlig unabhängig davon, ob das System und die Kopien
unterscheidbare Teilchen enthalten oder nicht. Die fiktiven
Kopien wären durchaus unterscheidbar, wenn man sie mikroskopisch
genau kennen würde. Wären sie das nämlich nicht, so würde
sich aus einem Ensemble reiner Zustände nie ein gemischter
ergeben können:

Ein System ist nämlich durch seinen Zustand charakterisiert,
zwei Systeme unterscheiden sich also definitionsgemäß genau dann,
wenn sie unterschiedliche Zustände haben.

In der traditionellen Interpretation hat ein vollständig beschriebenes
Einzelsystem S_k einen reinen Zustand, gegeben durch
rho_k = psi_k psi_k^* = |k><k|
wobei psi_k = |k> eine auf Norm 1 normierte Wellenfunktion ist.
Das Ensemble, das das System S_k mit der Wahrscheinlichkeit p_k
(sum_k p_k = 1) enthält, ist durch den gemischten Zustand
rho = sum_k p_k rho_k
charakterisiert. (Vgl. Landau und Lifschitz, Lehrbuch der
Theroetischen Physik, Band 5, Statistische Physik I, Abschnitt 5,
Die Dichtematrix.)

Wären alle S_k ununterscheidbar, so wäre
psi_k = psi_1 = |1> für alle k,
also
rho_k= |1><1| = rho_1 für alle k
und daher
rho = sum_k p_k rho_k = sum_k p_k rho_1 = rho_1
Also wäre rho derselbe reine Zustand.

In einem idealen Gas beispielsweise ist (wie in jedem anderen
mit statistischer Mechanik behandelten System) der Zustand aber
hochgradig gemischt.



In jedem einzelnen System sind nun Teilchen, und bei denen kann
es sich um unterscheidbare oder ununterscheidbare Teilchen handeln.

Ein einzelnes Wassermolekül H_2O besteht zum Beispiel aus einem
Sauerstoffkern O, zwei Wasserstoffkernen H und 10 Elektronen e
Die Elektronen sind nicht im chemischen Symbol mit drin,
da der Chemiker weiß, dass ein Wasserstoffatom normalerweise
ein Elektron hat und ein Sauerstoffatom acht. Wenn das nicht der
Fall ist, redet er von Ionen und schreibt fehlende Elektronen als
zusätzliche hochgestellte + und überzählige als zusätzliche
hochgestellte -, wie etwa die Reaktion
H_2O <--> H^+ + OH^-,
die in wässriger Lösung zum Teil stattfindet. (Das mittlere Plus
ist nicht hochgestellt und bedeutet einfach 'und'.)

H und O und e sind unterscheidbar, die beiden O's und die 10 e's
aber nicht. Und zwar in jedem einzelnen H_2O-Molekül. Wenn man
zwanzig H_2O-Moleküle hat, sind die zwanzig O's ununterscheidbar,
die vierzig H's und die 200 e's ebenso.
Das bedeutet, dass sich
nichts geändert hätte, wenn man welche der O's heimlich
vertauschen würde ohne sonst den Zustand zu verändern, etc.
(Formal, dass sich die Wellenfunktion bei entsprechenden
Koordinatenvertauschungen nicht ändert.)


Aber die H-Atome sind nicht vollkommen ununterscheidbar,
da man unterscheiden kann, ob zwei H-Atome am selben oder an
verschiedenen H_2O-Molekülen sitzen. Der Symmetriefaktor für
(sagen wir n) H_2O-Moleküle, der in der Partitionsfunktion
berücksichtigt werden muss, ist nicht (2n)!*n!, wie es im Fall
vollkommen ununterscheidbare Teilchen wäre, sondern nur 2^n*n!,
da man ohne experimentelle Konsequenzen die Moleküle untereinander
vertauschen kann und dann noch innerhalb jedes Moleküls die beiden
H_Atome.
In Benzoldampf ist der Symmetriefaktor 12^n*n!, da man ohne
experimentelle Konsequenzen einzelne Benzolmoleküle vertauschen kann,
innerhalb eines (ringförmigen) Benzolmoleküls aber nur die
Symmetrien einer Diedergruppe mit 12 Elementen erlaubt sind.
(Siehe auch den Eintrag ''Thermodynamik von Molekülen''
in diesem FAQ.)

Man kann dies experimentell nachweisen, indem man
a) die Mischungsentropie ausrechnet und mit dem Experiment
vergleicht,
b) gemessene Absorptions- oder Emissionsspektren misst und mit
dem Ergebnis quantenmechanischer Rechnungen vergleicht.
Nur bei korrekter Berücksichtigung der Symmetriefaktoren entsprechend
dem illustrierten Rezept ergeben sich korrekte Vorhersagen.


Alles hat aber an sich nichts mit Quantenphysik zu tun:
Die Teilchen in einem idealen Gas müssen auch klassisch
ununterscheidbar sein, damit die Eigenschaften mit den gemessenen
übereinstimmen.
(Siehe den Beitrag '' Klassische statistische Mechanik'' in diesem FAQ.)



Experimentalphysiker können natürlich längst nicht alles
im formalen Sinn Unterscheidbare experimentell unterscheiden.
Sie unterscheiden nur das, was ich 'relevante Größen'
genannt habe. Das ist die Terminologie der statistischen
Mechanik, die den Begriff 'ununterscheidbar' im von mir
beschriebenen mikroskopischen, nicht im experimentellen Sinn
verwendet.


---------------------------------------------------------
S2e. Was für Beobachtungen erklärt die Quantenmechanik?
---------------------------------------------------------

Die traditionelle QM erklärt nur _einige_ Beobachtungen, nämlich
die Beobachtungen über relative Häufigkeiten von Ereignissen,
die oft genug passieren, um sie statistisch interpretieren zu können.
Also z.B. Zerfallswahrscheinlichkeiten, Reaktionsraten,
Streuquerschnitte, und Materialeigenschaften, die sich durch
Mittelung über astronomisch viele Teilchen ergeben.

Eine hervorragende Übereinstimmung mit dem Experiment (auf viele
Dezimalstellen genau) gibt es allerdings nur bei nichtstatistischen Größen wie dem Lamb Shift, dessen 'Messung' von der traditionellen Karikatur des quantenmechanischen Messprozesses gar nicht erfasst wird. Das Lamb Shift ist nämlich gar keine Observable im Sinn der Lehrbuch- Quantenmechanik, nämlich kein hermitescher Operator, sondern eine reine Zahl, die Differenz zweier möglicher Energielevels im Spektrum eines solchen Operators. Die traditionelle QM erklärt auch nicht, weshalb 1999 eine in Mitteleuropa sichtbare Sonnenfinsternis war, nicht einmal, wenn man die Wellenfunktion des Sonnensystems am 1. Januar 1999 um 0:00 MEZ exakt angeben könnte. Und zwar prinzipiell nicht, wenn man
der traditionellen Deutung folgt.

Newton's klassische Mechanik hingegen erklärt es, wenn man Ort und
Impuls der Erde und der Sonne am 1. Januar 1999 um 0:00 MEZ
(und ein paar geographische Daten) kennt. Das ist eben der Unterschied.


In der klassischen Physik (und viel unserer Wirklichkeit wird davon
beschrieben) tritt von dem Möglichen nur ganz wenig ein.
Die Stärke der Naturwissenschaften liegt gerade darin, dass sie oft
vorherzusagen erlauben, was von dem Möglichen verwirklicht werden kann.

Sonnenfinsternisse können z.B. genau vorhergesagt werden, und sie
treffen dann und nur dann ein, wenn es die Astronomen vorhersagen.
Obwohl es einzelne Ereignisse sind, über die eine Statistik wenig
hergibt.

Aber nicht mit der Quantenmechanik. Beschreibt man das Sonnensystem
als quantenmechanisches N-Teilchensystem (was es angeblich ist), so
verschwindet plötzlich die Sicherheit, und alles, was man hat, ist
eine komplizierte Wellenfunktion, die mit dem Rest des Weltalls
verschränkt ist und nur noch Möglichkeiten enthält. Jedenfalls
in der traditionellen statistischen Interpretation der Quantenmechanik.


In der Quantenmechanik gibt es - den traditionellen Interpretationen
zufolge - keine Gewissheiten, das ist das Problem.

Früher, in der klassischen Physik, war das anders. Und daher braucht
die gängige Interpretation auch heute noch die klassische Physik,
um überhaupt sagen zu können, was die Quantenphysik bedeuten soll.
Experimente werden stets auf der klassischen Ebene gemacht, wo Dinge
wirklich und nachprüfbar passieren.

Man kann Experimente mit konkreten Apparaten nur mittels klassischer
Physik, nicht mittels Quantenphysik beschreiben; letztere beschreibt
nur die statistische Seite der Experimente.

Jede Messung in einer Messkammer eines Teilchenbeschleunigers
besteht aus klassischen Beobachtungen klassischer Variablen,
aus denen die Quantenereignisse (Zerfälle) mittels Methoden der
klassischen Physik (Fit von Teilchenbahnen, die es
quantenmechanisch angeblich gar nicht gibt) rekonstruiert werden.

Selbstverständlich darf man die klassische Physik nur in ihrem
Geltungsbereich benutzen, wenn man Experimente analysiert.
Aber dort braucht man sie auch. Die Quantenmechanik kennt den Begriff
einer Teilchenbahn nicht. Aber klassische Teilchenbahnen sind relativ
gut bestimmt, weil hochenergetische Teilchen große Impulse haben,
was laut Heisenberg'scher Unschärferelation eine gute Ortsauflösung
erlaubt.

Alle Hochenergieexperimente, und damit auch all unsre Kenntnisse über
Quarks, Z-Bosonen, schwache und starke Wechselwirkung, etc. beruhen
auf solchen oder ähnlichen Techniken.

Das, was in den Quantenmechanik-Lehrbüchern über Messung eines
Quantensystems steht, ist eine skurrile Karikatur dessen, was
wirklichkeitsnahe Messungen von relevanten Quantensystemen beinhalten.


Solange man die Quantenmechanik nicht so erklären kann, dass sie auf
die klassische Ebene ganz verzichten kann (indem sie mit all ihren -
definiten - Eigenschaften innerhalb der Quantenwelt rekonstruiert wird),
fehlt etwas Entscheidendes.

Die traditionelle Quantenphysik braucht also die klassische Physik,
um überhaupt interpretiert werden zu können, und ist _daher_
unvollständig. Weil und solange dies fehlt, bleiben die Grundlagen
der Quantenmechanik die trüben Gewässer, die sie derzeit sind.

Ohne klassische Physik gibt es auch keine Quantenphysik.


---------------------------------------
S2f. Teilchenspuren und Quantenmechanik
---------------------------------------

In der alten, aber nach wie vor interessanten Arbeit
N.F. Mott,
The wave mechanics of alpha-ray tracks,
Proc. Roy. Soc. London A126 (1929), 79-84,
wird erstmalig beschrieben, wie es kommt, dass Alphateilchen in
Nebelkammern sichtbare und klassisch aussehende Spuren hinterlassen.

In seiner Analyse der Teilchenspuren bekommt Mott aber nur eine
Superposition von Spuren heraus, die nach Dekohärenz eine Mixtur von
Spuren werden, aber nicht eine einzelne Spur, wie man sie misst.

Mott rechnet nur Korrelationen aus. Im wesentlichen zeigt er, dass die
Wahrscheinlichkeit, dass zwei Atome ionisiert werden, nur dann
signifikant ist, wenn diese praktisch genau auf einer Geraden durch
den radioaktiven Atomkern liegen, der das Alphateilchen aussendet.

Alle Geraden sind völlig gleichberechtigt, und das Gesamtsystem
(Teilchen + Gas) ist in einem reinen Zustand, einer Superposition
aller dieser Geraden. Nichts von Dekohärenz, nichts vom übergang
zu einem gemischten Zustand, nichts vom Kollaps der Wellenfunktion.
Er erklärt nicht, warum man statt der Superposition eine einzelne
Gerade beobachtet.

Mott zeigt, dass ein Ensemble aus vielen Alphateilchen geradlinige
Nebelkammerspuren im Plural hinterlässt. Man kann aber beobachten,
dass ein einzelnes Alphateilchen eine Spur hinterlässt.
Wenn man danach das Experiment abbricht, hat man etwas Objektives
beobachtet, und man weiß, es ist ein einzelnes Alphateilchen
dagewesen. Man weiß auch ungefähr, wo es gewesen ist.

Wenn man - wie die minimale Interpretation - nicht behauptet,
dass dem Einzelsystem eine objektive Wellenfunktion zukommt,
ist in dieser Hinsicht nicht mehr zu fordern; das gibt die
minimale Interpretation.

Die Interpretation der Quantenmechanik geht aber gewöhnlich von der
Behauptung aus, messen könne man nur Eigenwerte von Operatoren,
und bei der Messung gehe das Einzelsystem in einen zugehörigen
Eigenzustand über.
Das ist aber in Mott's Analyse nicht berücksichtigt; es scheitert
auch schon daran, dass zu Orts- und Impulsoperatoren gar keine
normierten Eigenzustände existieren.


Was ist nun die - offenbar unkontrovers objektiv vorhandene - einzelne
Spur? Sie ist das Ergebnis einer deterministischen Dynamik für die
Schrödingergleichung des einzelnen Messapparats (d.h. der gesamten
Nebelkammer). Man hat kein Ensemble, sondern ein großes Einzelsystem!
Was dagegen ins Spiel kommt, ist die makroskopische Natur des
Messapparats; die sorgt für die Objektivität. Aber wie, das bleibt
in der minimalen Interpretation schleierhaft. Selbst Dekohärenz
sagt nur eine Mixtur von Spuren voraus, und nicht die einzelne Spur!

Aber von dieser Problematik ist bei Mott mit keinem Wort die Rede!
Er behandelt nicht einen dissipativen Vielteilchenprozess, der für
die irreversible Beobachtungsprodukte verantwortlich ist, sondern ein
reversibles Dreiteilchenproblem (Alphateilchen und zwei Streuzentren)..


Außerdem lässt die minimale Interpretation etwas offen.
Zumindest für große Systeme wie Galilei's Bleikugeln sollte ein
Einzelsystem jedoch eine objektive Beschreibung haben.
Die minimale Interpretation lässt dies aber nicht zu.
Da man keine Grenze angeben kann, ab der das der Fall ist, sollten
aber auch kleine Einzelsysteme und daher einzelne Alphateilchen und
andere Teilchen eine objektive Beschreibung haben. Dann ist die
minimale Interpretation völlig unzureichend.

Wer andrerseits dem Einzelsystem eine objektive Beschreibung zugesteht,
muss auch zeigen können, dass - wie man stets beobachtet -
das einzelne Alphateilchen eine einzelne Spur hinterlässt.
Mit Mott kann man aber nicht verstehen, wie aus der angeblich
objektiven Beschreibung des einzelnen Systems die einzelne Spur
herauskommt, sondern nur, dass ein ganzes Ensemble von Versuchen eine
statistische Verteilung von Spuren liefert. Das ist ein signifikanter
Unterschied.


--------------------------------------------------
S2g. Quantensysteme sind praktisch immer gemischt!
--------------------------------------------------

In den Lehrbüchern findet man oft die Aussage, dass gemischte
Zustände (Dichtematrizen) nur auf der Unkenntnis des Betrachters
beruhen.

Letzteres ist aber häufig nicht der Fall. Gemischte Systeme entstehen
auch auf völlig beobachterunabhängige Weise als Teilsysteme eines
größeren reinen Systems, da die die Beschränkung auf das Teilsystem
(partielle Spurbildung der Dichtematrix) aus einem reinen Zustand
in der Regel einen gemischten Zustand macht. Diese gemischten
Zuständen können nicht sinnvoll als auf Unkenntnis des Betrachters
beruhend angesehen werden.

Aber für jemanden (nicht mich), der von einer subjektiven
Interpretation der QM ausgeht (also: der Zustand codiert nur das,
was wir von System wissen), kann ein nichttrivial großes Gesamtsystem
nie in einem reinen Zustand sein. Denn niemand ist in der Lage,
ein vollständiges System von kommutierenden Variablen eines
Quantensystems mit mehr als ein paar Freiheitsgraden auch
nur einigermaßen genau zu messen. Und alle Ungenauigkeiten
übersetzen sich in Unkenntnis, also Unreinheit...

Unabhängig davon ist der Fall eines größeren reinen Systems
natürlich eine Idealisierung. Denn das größere System ist selbst
auch Teil eines noch größeren Systems und damit nach demselben
Argument mit ziemlicher Sicherheit auch ein gemischter Zustand.


Die einzigen Systeme, die - streng genommen - in einem reinen Zustand
vorliegen können, sind also (unabhängig davon, ob man eine
subjektive oder objektive Interpretation der Quantenmechanik bevorzugt):

1. Das Universum als Ganzes,

2. Systeme mit so wenigen Freiheitsgraden (typischerweise nur ein
paar Spins), dass man sie durch Präparation (d.h.von-Neumann-Messung
eines maximalen Systems von kommutierenden Variablen) in einen reinen
Zustand bringen kann.



-------------------------------
S2h. Zeit in der Quantentheorie
-------------------------------

In der traditionellen Formulierung der Quantentheorie ist die
Zeit keine Observable.

Die Kommutatorregeln werden in der Quantenmechanik durch Ersetzen
der klassischen Poissonklammern durch skalierte Kommutatoren ermittelt.
Man sieht aber leicht, dass die Zeit in der klassischen
nichtrelativistischen Mechanik keiner {t,H}=1 entsprechenden
Relation genügt - die Zeit ist klassisch nur ein Parameter, von dem
alle Observablen abhängen. Daher gibt es auch keine Ursache,
quantenmechanisch einen Zeitoperator t zu postulieren und von
[t,H]=i hquer auszugehen, obwohl die entsprechenden Relationen
{x_j,p_k} = \delta_{jk},
[x_j,p_k] = \delta_{jk} i hquer
für Ort und Impuls gelten.


Die Relation [t,H]=i hquer hätte auch zur Folge, dass das Spektrum von
H nach unten unbeschränkt ist, was physikalisch keinen Sinn macht.
Der Grund dafür ist, dass die kanonischen Kommutatorrelationen die
Darstellung bis auf Isomorphie eindeutig festlegen:

Sind p und q selbstadjungierte Operatoren und gilt
[q,p]=i hquer, so ist in der Darstellung, in der q diagonal ist,
der Hilbertraum isomorph zu L^2(R) und p = - i hquer d/dq.
Im Wesentlichen ist das Schrödinger's Äquivalenzbeweis der
Heisenbergschen und der Schrödingerschen Quantenmechanik. Traditionell
läuft diese Aussage allerdings unter dem Namen Stone - von Neumann
Theorem.
http://en.wikipedia.org/wiki/Stone-von_Neumann_theorem
Mit t und H statt q und p folgt, dass H dasselbe Spektrum wie p hat,
also ganz R als Spektrum hat und insbesondere nach unten unbeschränkt
ist.

Dies kann man heuristisch auf einfachere Weise sehen, wenn man
voraussetzt, dass H einen nicht notwendig normierten Eigenvektor |E_0>
zu einem Eigenwert E_0 hat. Dann ist |E>:=exp(i(E-E_0)t/hquer)|E_0> ein
Eigenvektor von H zum Eigenwert E, wie man aus
H|E> = H exp(i(E-E_0)t/hquer)|E_0>
= exp(i(E-E_0)t/hquer)(H+E-E_0)|E_0>
= exp(i(E-E_0)t/hquer) E|E_0> = E|E>
sieht. Die Existenz von exp(i(E-E_0)t/hquer) ist wegen der
Selbstadjungiertheit von t gesichert, allerdings nur als Operator auf
dem Hilbertraum. Der Eigenvektor |E> liegt aber nicht im Hilbertraum;
dazu müsste er normierbar sein, was be kontinuierlichem Spektrum
nicht geht. Daher ist dieses Argument mathematisch nicht stichhaltig.

Ein mathematisch korrekter Beweis des Stone-von Neumann Theorems ist
dagegen nichttrivial. Man kann nämlich zu bestimmten Ausdrücken
für H formale Ausdrücke für einen Operator T angeben, so dass
[T,H]=i hquer ist.
Z.B. gilt dies mit T=i hquer/2 (p^{-1} d/dp + d/dp p^{-1}) für
H=p^2/2m (freies Teilchen) und mit T = arctan(q/p) für H=(p^2+q^2)/2
(harmonischer Oszillator). Dieser Operator T ist aber im ersten Fall
bei p=0 singulär und nicht selbstadjungiert, und im zweiten Fall wegen
der Mehrwertigkeit von arctan nicht einmal wohldefiniert.
Die Selbstadjungiertheit wird aber gebraucht, um die Spektralzerlegung
zu garantieren, die für die Bornsche Regel unverzichtbar ist.


Nahegelegt wird [t,H]=i hquer nur von zu oberflächlichen Anleihen
bei der Relativitätstheorie, in der Raum und Zeit analog
behandelt werden. Aber dort haben weder Raum noch Zeit
Observablencharakter, und auch die Relation [x_j,p_k]=i \delta_{jk}
verliert dort ihre Bedeutung. Denn in einer irreduziblen Darstellung
der Poincaregruppe (die ein relativistisches Teilchen beschreibt)
haben nur der 4-Impuls und der 4-Drehimpuls Observablencharakter.
Daraus lassen sich zwar (beobachterabhängig) 3D Ortsvariable
mit den richtigen Kommutatorregeln rekonstruieren (Newton-Wigner
Positionsoperator), aber kein Zeitoperator.


In der traditionellen Formulierung der Quantentheorie ist die
Zeit also keine Observable. Trotzdem kann sie gemessen werden...


Im Schrödingerbild ist der Zustand für fixe Zeiten definiert,
und damit die Zeit ausgezeichnet. Hier ist Zeitmessung schwierig zu
diskutieren, da die Zeit, zu der ein Zustand betrachtet wird, immer
scharf ist.

Im Heisenbergbild kommt die Zeit als Parameter in den Observablen vor,
und ist damit auch ausgezeichnet, aber auf andere Weise.
Parameter sind de facto einfach kontinuierliche Indices und keine
Observablen. So wie 3 keine Observable ist, p_3 aber schon, so ist t
keine Observable, H(t) aber schon. Observablen haben zu jedem
Zeitpunkt einen mittleren Wert; der Zeitpunkt (''jetzt'') ist dagegen
nicht als Observable modelliert.

Was man aber modellieren kann, ist dagegen eine Uhr, d.h. eine
Observable, die sich auf vorhersagbare Weise mit der Zeit ändert.
Hat man ein System, in dem eine Observable u(t) das Verhalten
ubar(t) := <u(t)> = u_0 + v (t - t_0) (v nicht 0) (*)
mit genügender Genauigkeit erfüllt, so hat man eine Uhr,
und kann anhand von <u(t)> feststellen, wie viel Zeit
T = Delta t
zwischen zwei beobachteten Datensätzen vergangen ist.
Das ist die normale Art, wie wir auch klassisch Zeit messen.

Dazu muss natürlich T gegenüber der intrinsischen Unschärfe
Sigma_T := |v^{-1}| sigma(u(t))
von T gross genug sein. Dabei ist
sigma(u(t)) = sqrt(<(u(t)-ubar(t))^2>)
die Standardabweichung von u(t) im ordnungsgemäßen
(quantenmechanischen) Zustand <.>. Ist (*) signifikant fehlerbehaftet,
so ist Sigma_T natürlich entsprechend größer.


In der relativistischen Quantenfeldtheorie (die fast immer im
Heisenbergbild formuliert wird) wird aus der 1-dimensionalen Zeit t
die 4-dimensionale Raumzeitposition x. Auch x tritt als Parameter der
Observablen (Felder) auf, und ist daher keine Observable.
Ort und Zeit sind zwar jetzt gleichberechtigt, aber beide als
Nichtobservable. Die Observablen sind Felder; Orte und Zeiten werden
durch unscharfe 1-dimensionale Weltlinien mit hoher <Felddichte>
modelliert. (Man denke an die Spur eines Teilchens in der Blasenkammer.)

Jetzt braucht man zur Orts- und Zeitmessung ein 4-Vektorfeld u(x)
mit
<u(x)> = u_0 + V (x - x_0)
und einer regulären 4x4-Matrix V, und die intrinsische Unschärfe
nimmt die Form
Sigma_T := sigma(V^{-1}u(x))
an, wobei
sigma(a(x)) = sqrt(<(a(x)-abar(x))^*(a(x)-abar(x))>),
abar(x)=<a(x)>.
ist.


Fazit: In der nichtrelativistischen Quantenmechanik wird Zeit immer
indirekt über Observablen von Uhren in kalibirierten Zuständen
gemessen. In der relativistischen Quantenfeldtheorie gilt dasselbe
für Zeit _und_ Position.


Das funktioniert allerdings nur, wenn man einzelnen Uhren einen
wohldefinierten Zustand zuordnet, also eine Version der
Kopenhagen-Interpretation zugrunde legt.

Nach der minimalen statistischen Interpretation braucht man ein
ganzes Ensemble von identisch präparierten Uhren, um Zeit messen
zu können...


-------------------------------------------
S2i. Kopenhagen und Ensemble-Interpretation
-------------------------------------------

http://de.wikipedia.org/wiki/Kopenhagener_Deutung
''Die Ensemble-Interpretation ist eine Variante der Kopenhagener
Deutung, bei der die Quantentheorie nicht bezüglich einzelner Systeme,
sondern bezüglich Ensembles von identisch präparierten Systemen
betrachtet wird.''

Das ist der Hauptunterschied, und zwar ein wichtiger.

Die Kopenhagen-Interpretation schreibt jedem Einzelsystem einen
Zustand zu, die statistische Interpretation nur einem Ensemble von
identisch präparierten Systemen.

Erstere hat oberflächliche Probleme damit, zwei unterschiedliche
Dynamiken für ein System anzubieten - unitäre Schrödingergleichung
wenn es unabhängig ist, Kollaps wenn es gemessen wird.
Das überrascht aber nicht, da im letzteren Fall die Messumgebung,
die für den Kollaps verantwortlich ist, nicht mit modelliert wird,
aber die Dynamik natürlich ändert. (Ein nicht wechselwirkendes
System kann natürlich auch nicht gemessen werden.)

Letztere hat Probleme damit, Systeme zu beschreiben, die (wie
die Sonne) nicht mehrfach präpariert werden können.

Beide haben Probleme damit, zu sagen, was passiert, wenn das
System immer größer gewählt wird, bis es schließlich das
ganze Universum umfasst.


Es gibt keine wohldefinierte Version der Kopenhagen-Interpretation;
unterschiedliche Leute verstehen darunter unterschiedliches;
der deutsche Wikipedia-Artikel
http://de.wikipedia.org/wiki/Kopenhagener_Deutung gibt kein korrektes Bild von der Bandbreite des de facto Spielraums in der Kopenhagen-Interpretation. Das gewichtigste Dokument ist wohl HP Stapp The Copenhagen Interpretation American Journal of Physics 40 (1972), 1098-1116. Für die statistische Interpretation gilt üblicherweise L.E. Ballentine, The Statistical Interpretation of Quantum Mechanics, Rev. Mod. Phys. 42, 358-381 (1970) als autoritatives Dokument. ------------------------------------------------------------ S2j. Zweierlei Naturgesetze in der Kopenhagen-Interpretation ------------------------------------------------------------ In der Kopenhagen-Interpretation gibt es zweierlei Naturgesetze, also zwei Dynamiken - die unitäre, solange das System isoliert, also unbeobachtbar ist, und die dissipative, wenn das System beobachtet wird. Die unitäre Dynamik gilt nur für abgeschlossene Systeme. Bekanntlich ist kein beobachtbares System (abgeschlossen außer dem Universum als Ganzen), denn ohne Wechselwirkungen mit der Umgebung ist ein System unbeobachtbar. Also gilt die unitäre Dynamik nur, solange das System isoliert, also unbeobachtbar ist. Dies gilt auch in der modernen Form der Quantenmechanik. Wenn man also ganz penibel ist, ist das Universum als Ganzes das einzige System, das eine unitäre Dynamik hat, alle Teilsysteme wechselwirken mit ihrer Umgebung und sind daher offene Systeme mit Dissipation. Glücklicherweise für die Praxis (und die Entdeckung der Quantenmechanik) kann man die entstehende Dissipation oft vernachlässigen, ohne das Problem allzu sehr zu ändern (aber man verliert z.B. Information über die Breite von Spektrallinien...), so dass man kleine Systeme für Lehrbuchzwecke oft als konservativ idealisiert. Aber aus der Quantenoptik weiß man, dass reale Prozesse nie ganz konservativ sind, und die Modellierung als offenes System wesentlich für die Übereinstimmung mit dem Experiment ist. Die dissipative Dynamik heißt in der Kopenhagen-Interpretation Kollaps, und ist in der alten Form (den experimentellen Möglichkeiten in der Anfangszeit entsprechend) stark idealisiert und zunächst nur für eine bestimmte Klasse von Messprozessen präzisiert: Im Fall einer sogenannten vollständigen von-Neumann-Messung geht das System während der Messdauer (die meist idealisiert als instantan angenommen wird) in einen Eigenzustand des gemessenen Operators über. Ein Kollaps tritt laut Kopenhagen dann auf, wenn (durch Wechselwirkung mit der Umgebung) eine Beobachtung gemacht wurde, die in der Umgebung irreversibel gespeichert ist - unabhängig davon, ob es ein Mensch zur Kenntnis nimmt oder nicht. Der Kollaps ist experimentell nachprüfbar; in der Quantenoptik werden regelmäßig dissipative Quantensysteme modelliert. Die verwendeten Modelle sind detaillierter als in der Kopenhagen-Interpretation, die idealisierte von-Neumann-Messungen voraussetzt, reproduzieren aber den Kollaps völlig objektiv, außer dass er nicht instantan vor sich geht (wie es bei einer idealisierten von-Neumann-Messung sein müsste), sondern ein klein bisschen Zeit braucht. Der Zustand eines einzelnen Atoms in einer Ionenfalle oder eines Elektrons in einem Quantenpunkt springt experimentell beobachtbar zwischen Eigenzuständen von angeregt auf nicht angeregt und umgekehrt. Siehe z.B. den Überblicksartikel R. Hanson et al., Spins in few-electron quantum dots, http://arxiv.org/abs/cond-mat/0610433 Die Zeiten, wann der Zustand springt, sind nicht vorhersagbar. Aber die Statistik der Springzeiten schon. Für das Einzelsystem! Die Statistik bezieht sich zwar auch auf ein Ensemble, aber auf ein Ensemble von Observablen desselben Einzelsystems. (Sie bezieht sich insbesondere nicht auf ein Ensemble von ähnlich präparierten Systemen. Präpariert wird die ganze experimentelle Anordnung vor Beginn der ersten Messung, und dann nicht mehr). Die Messungen liefern Information über den Zustand dieses Einzelsystems als Funktion der Zeit. Der Zustand muss also zum Einzelsystem gehören. Dass man das auch statistisch interpretieren kann, ist kein Argument gegen die Kopenhageninterpretation - man behandelt ja auch Messungen an klassischen Einzelsystemen (Brownsche Bewegung etwa) statistisch, ohne daran zu zweifeln, dass ein einzelnes klassisches System wohldefinierte Zustände hat. Die Versuchsanordnung wird vor Beginn der Messungen präpariert. Die Präparation stellt sicher, dass das System genau ein Atom (Ionenfalle) bzw. maximal ein Elektron (Quantendot) enthält, dass man das System also wirklich as eine Ionenfalle bzw. einen Quantendot betrachten darf. Mehr nicht. Der Zustand des Atoms/Elektrons kann zu Beginn in einen Eigenzustand von H (=H_atom bzw =H_elektron) gebracht werden. Dann werden Laser-Impulse dazugegeben und der Zustand kontinuierlich in der Zeit vermessen. Da das System jetzt wechselwirkt, kennt man den Zustand des Atoms/Elektrons nur noch aus der Messung, nicht mehr aus der Präparation. Die Präparation verhindert nur, dass z.B. ein zweites Elektron in den Quantendot kommen kann. Nur wenn man das Wort 'statistisch' ganz informell verwendet, hat man eine statistische Interpretation. Aber das ist dann nicht mehr die statistische Interpretation in der traditionellen Form von L.E. Ballentine, The Statistical Interpretation of Quantum Mechanics, Rev. Mod. Phys. 42, 358-381 (1970), Das System passt also auf keinen Fall in die von Ballentine postulierte Ensemble-Form. Er fordert nämlich (S.361 links oben): ''For example, the system may be a single electron, Then the ensemble will be the conceptual (infinite) set of all single electrons which have been subjected to some state preparation technique (to be specific for each state), generally by interaction with a suitable apparatus. [...] We say that a quantum state is a mathematical representation of the result of a certain state preparation procedure.'' Viele Systeme sind hinreichen isoliert, um approximativ als konservativ beschreiben zu werden. Aber diese Approximation produziert entsprechende Artefakte, z.B. unendlich dünne Spektrallinien - die Linienbreite lässt sich nur ausrechnen, wenn man Dissipation in Betracht zieht. Will man diese berücksichtigen, muss man das System als offenes System behandeln. Will man das auf unitäre Weise tun, muss man also das System als Teilsystem eines grösseren behandeln, und Teilsystem+Messapparat modellieren. (Siehe den folgenden FAQ-Beitrag ''Kollaps und offene Quantensysteme''.) Allerdings gilt für das System aus Teilsystem+Messapparat wieder dasselbe: Es ist nicht abgeschlossen und unterliegt einer dissipativen Dynamik, allerdings nur mit teilweisem Kollaps, da die Systemumgebung sicher nicht mehr ein vollständiges System kommutierender Observablen misst. Daher muss man dessen Umgebung auch mit einbeziehen. Das so vergrößerte System hat wieder dasselbe Problem, und muss daher wieder vergrößert werden, usw. usw.. Man endet also damit, dass man das Universum als Ganzes vollständig beschreiben müsste, denn nur dieses ist abgeschlossen genug, um wirklich einer unitären Dynamik zu genügen.

Die Frage erhebt sich daher, ob man das Universum als Ganzes
quantenmechanisch beschreiben kann - zunächst unabhängig davon,
ob man seinen Zustand auch wirklich finden kann.

In Ballentines statistischer Interpretation, die Hendrik van Hees
zu vertreten vorgibt, geht das offenbar nicht, da man das Universum
nicht präparieren kann, sondern es so nehmen muss, wie es ist.
Messen kann man jedoch viele seiner Observablen; allerdings kein
vollständiges System kommutierender Observablen.

In der in dem FAQ-Beitrag ''Quantenmechanik von Einzelsystemen''
angegebenen ensemblefreien, an Kopenhagen angelehnten Interpretation
geht es dagegen problemlos, ohne dass sich irgendwelche andere
Probleme ergeben.

Also kann man für die Interpretation der Quantenmechanik auf
Ensembles ähnlich präparierter Systeme verzichten; man hat
nur Vorteile davon.

Und da man es kann, sollte man es (nach Ockhams Rat) auch tun.


--------------------------------------
S2k. Kollaps und offene Quantensysteme
--------------------------------------

Die Kopenhagen-Interpretation ist dem einzelnen System angemessen,
allerdings in einer Idealisierung, die den Messprozess als instantan
ansieht. Sie ist daher nur dann gültig, wenn die zeitliche Auflösung
nicht allzu hoch ist. Bei besserer Auflösung stellt sie nur noch eine
bequeme aber ungenaue Approximation dar.


Die Theorie, die den alten Kollaps modern präzisiert, wird weit
und breit unter dem Stichwort offene Quantensysteme diskutiert.
Ein modernes Buch dazu ist
HP Breuer, F Petruccione
The Theory of Open Quantum Systems
Oxford Univ. Press, Oxford 2002
Nach Approximation der Umgebung durch eine Gibbsverteilung
und Projektion auf die relevanten Variablen des Systems bleibt
ein dissipatives Quantensystem übrig, das im Mittel durch eine
Lindblad-Dynamik charakterisiert ist, und im Einzelnen durch einen
quanten-statistischen Prozess.

Zur Herleitung der Dynamik wird stets eine einzelne Umgebung
modelliert, nicht ein Ensemble von Umgebungen. Man schreibt dieser
einen (im Gesamtexperiment tatsächlich vorliegenden) Umgebung
einen wohldefinierten Gibbs-Zustand zu - eine relative grobe
Approximation, aber ausreichend zur Herleitung der Grundgleichungen.
(Die Koeffizienten werden sowieso an die Daten gefittet - vgl.
den FAQ-Beitrag ''Effektive quantenmechanische Modelle'', also kommt
es nicht auf ein quantitatives Übereinstimmen der Herleitung an.)

Die Umgebung ist z.B. das einzelne Halbleiterplättchen, auf dem
der Quantendot präpariert ist. Man braucht zur Herleitung der
dissipativen Dynamik des Quantendots den Zustand des einzelnen
Halbleiterplättchens.

(In der statistischen Interpretation von Ballentine
braucht man dagegen ein Ensemble von ähnlich präparierten
Halbleiterplättchen, um die Modellierung des Systems aus
Quantendots und Halbleiterplättchen zu interpretieren.
Man macht die Versuche aber nicht mit einem Ensemble von
Halbleiterplättchen, sondern nur mit einem präparierten
solchen - und ist trotzdem erfolgreich.
Offenbar ist das Ensemble von ähnlich präparierten
Halbleiterplättchen irrelevanter Ballast, der von Ballentine's
Definition erzwungen wird, mit Occam's Rasiermesser aber
besser weggelassen wird.)


Der alten Einteilung der Kopenhageninterpretation in Quanten- und
klassische Welt entspricht in der modernen Theorie offener
Quantensysteme die Einteilung in System und Umgebung. Inhaltlich ist
es genau dasselbe. In der Umgebung werden nämlich nur noch klassische
(nämlich makroskopische, thermodynamische) Variable modelliert.

Dieser Einteilung haftet eine gewisse Willkür an, sowohl in der alten
als auch in der modernen Form. Denn in der Natur gibt es keine
abgegrenzten Systeme, nur die Natur als Ganzes. Es ist also Sache des
Physikers, der ein Problem analysiert, wie er das Quantensystem
abgrenzt. Normalerweise wird die Abgrenzung so vorgenommen, das das
System analysierbar bleibt und die Genauigkeit für den Vergleich mit
dem Experiment noch ausreicht. Diese Willkür ist also dem Problem
angemessen und nichts Fragwürdiges.

Der Fortschritt der modernen Theorie offener Systeme gegenüber der
einfachen Kopenhagen-Betrachtungsweise besteht darin, dass man
quantitative und zeitlich aufgelöste Details versteht,
wo vorher nur Grundzüge verstanden waren, und dass man das größere
System als Quantensystem behandeln kann, ohne den Formalismus wechseln
zu müssen. Man braucht also die klassische Welt nicht separat
vorauszusetzen (aber dieser Teil geht schon auf von Neumann 1932
zurück).

Aber das Klassische wird dafür in die Approximationsmethode
hineingesteckt. Wenn man ein System über einen Gibbszustand modelliert,
ist das äquivalent dazu, nur die klassischen thermodynamischen
Variablen für relevant zu erklären. Dann liefert der traditionelle
Ansatz die Gibbsverteilung.

(Aus der Sicht von Interpretationen, die den Zustand als
Quintessenz der Information eines Beobachters über ein System
auffassen, liegt hier ein Zirkelschluss vor: Der Beobachter steckt
sein Wissen darum, dass die Umgebung im wesentlichen durch wenige
klassische Variablen charakterisiert ist, in die Modellierung hinein,
findet mittels dem Maximum-Entropie-Prinzip, dass sein Wissen einem
Gibbs-Ensemble entspricht, und findet dann durch eine Analyse der
mit diesem Ansatz für die Umgebung reduzierten Dynamik für das
Teilsystem die ''emergence of a classical world'' für das reduzierte
offene Quantensystem. Aus der angenommenen Klassizität folgt also
die Klassizität....)



Unter Voraussetzungen, die ungefähr einer von-Neumann-Messung
entsprechend (ungefähr, weil von-Neumann-Messungen idealisiert sind
und nur in einem Limes existieren), ergibt sich eine Dynamik, in der
aus einem gemischten Zustand im Grenzwert t--> unendlich ein
reiner Zustand wird, der ein Eigenvektor des gemessenen Operators
ist. Welcher, ist allerdings nur stochastisch bestimmt; die
Lindblad-Dynamik ergibt Konvergenz gegen eine diagonale Dichtematrix.
Da die Konvergenzrate sehr hoch ist, ist in vielen praktischen Fällen
die Zeit, die zur Konvergenz nötig ist, extrem kurz, so dass man
quasi einen instantanen Kollaps bekommt.

Im Limes unauflösbar kurzer Messzeiten ergibt sich _genau_ der alte
Kollaps. Ebenso wie man für kleine Geschwindigkeiten
nichtrelativistisch rechnen darf, darf man bei großen Zeiten
Quantensprünge als instantan annehmen, auch wenn sie es exakt nie
sind.

Wer das nicht als Gültigkeitsbeweis des Kollapses im
Grenzfall geringer zeitlicher Auflösung anerkennt, dürfte
keinerlei historische Kontinuität bei Begriffen in der Physik
unterstellen, wenn sich die Theorien verbessern. Man kann es Bohr,
Heisenberg und von Neumann nicht verübeln, dass sie bei den
damalig üblichen Auflösungen den dissipativen Messprozess als
idealisierten Kollaps modellierten und nicht, wie es heute geschieht,
als zeitlich aufgelöste dissipative Dynamik.

Für den experimentellen Nachweis von Quantensprüngen siehe etwa
RG Hulet, DJ Wineland, JC Bergquist, WM Itano
Precise test of quantum jump theory
Phys. Rev. A 37, 4544 - 4547 (1988)
oder
N Gisin, PL Knight, IC Percival, RC Thompson, and DC Wilson
Quantum State Diffusion Theory and a Quantum Jump Experiment
Journal of Modern Optics 40, 1663 (1993)
oder der überblicksartikel
MB Plenio, PL Knight
The quantum-jump approach to dissipative dynamics in quantum optics
Rev. Mod. Phys. 70, 101 - 144 (1998);


Unsere Diskussion ergibt also:

1. Kopenhagen's Kollaps ist ein Grenzfall der modernen Sicht
und kann durchaus (im Grenzfall) aufrechterhalten werden.

2. Die Tatsache, dass große Systeme einmalig sind und trotzdem
einen wohldefinierten, objektiven Zustand haben, muss vorausgesetzt
werden, damit die modernen Herleitungen des Verhaltens makroskopisch
großer (offener) Systeme stichhaltig sind. Notwendig ist, dass einer
einzelnen Umgebung (d.h. in letzter Konsequenz dem Universum mit
Ausnahme des Systems) ein wohldefinierter Zustand zugeschrieben
werden kann, der nicht über Ensembles charakterisierbar ist.

3. Die moderne Verwendung der Quantenmechanik erfordert also zwingend,
dass jedem System ein (gemischter, zeitlich veränderlicher)
quantenmechanischer Zustand zugeordnet ist, der alle objektiv
vorhersagbaren Aspekte seines Verhaltens beschreibt.

3. In Ballentines statistischen Interpretation kann man aber nur
Aussagen machen über Ensembles von Umgebungen, nicht über die
einzelne, in einem Ein-Atom-Experiment konkret vorliegende Umgebung.
Dies wird dem praktischen Gebrauch der Quantenmechanik in
Ein-Atom-Experimenten nicht gerecht. Daher ist die statistische
Interpretation zumindest in Ballentines Version von 1970 überholt.



---------------------------------------
S2l. Quantenmechanik von Einzelsystemen
---------------------------------------

Wir messen das Alter des Universums nicht an einem Ensemble von
Universen, und den Abstand Erde-Pluto am 1.1.2000
um 0:00 Uhr nicht an einem Ensemble von Planetensystemen.
Und doch ist zumindest das letztere unbestreitbar ein Quantensystem.
Offenbar muss es als möglich sein, Quantenmechanik auf Einzelsysteme
anzuwenden.

Allgemein gilt: Die Kenntnis des Zustands und damit
aller Erwartungswerte besagt in _jedem_ Einzelfall, dass eine reelle
Größe X mit an Sicherheit grenzender Wahrscheinlichkeit von
Xbar = <X>
um maximal ein kleines Vielfaches von
sigma(X) = sqrt{<(X-Xbar)^2>}
abweicht. Der Faktor für eine vorgegebene Irrtumswahrscheinlichkeit
kann auch vorhergesagt werden. Ohne dieses Faktum wäre es praktisch
unmöglich, Aussagen über Erwartungswerte (und Statistik aller Art)
zu verifizieren.

Ist sigma(X) genügend klein, braucht man kein Ensemble.
Der beim Einzelsystem gemessene Wert ist in jedem Einzelfall
vorhersagbar. Das verwenden wir bei den meisten alltäglichen
Messungen, die nur einmal (oder ganz wenige Male) durchgeführt
werden.

Ist sigma(X) allerdings relativ groß, bekommt man keine
Aussage von Wert über X. Aber man bekommt eine brauchbare
Aussage über gemittelte Größen
Z = (X_1+...+X_n)/n,
wenn die X_i genügend wenig korreliert sind. Denn nach dem
Gesetz der großen Zahlen ist im unkorrelierten Fall
Zbar = Xbar,
sigma(Z) = sigma(X)/sqrt{n},
und für genügend grosse n wird das beliebig klein.

Der (gemischte, zeitlich veränderliche) quantenmechanischer Zustand
eines Systems beschreibt die objektiv vorhersagbaren Eigenschaften in
seinem Verhalten, nicht mehr und nicht weniger. Manche der
Eigenschaften haben eine große Genauigkeit, andere nicht.
Statistische Observablen tendieren dazu, kleinere Ungenauigkeiten zu
haben und sind deshalb insbesondere bei winzigen Systemen
aussagekräftiger.

So wie beim Messen klassischer Observablen chaotischer
Systeme auch.


Diese Auffassung von Statistik ist 100%ig konsistent mit
der Art, wie Statistik in Mathematik, Physik und Ingenieurswesen
verwendet wird, und passt auf klassische Physik und Quantenphysik,
inklusive statistischer Mechanik. Sie erklärt zwanglos, warum
zunehmend große Systeme zunehmend besser klassisch beschrieben
werden können, ohne sagen zu müssen, wo man genau die Grenze
zwischen klassisch und quantenmechanisch ziehen muss.

Sie hat außerdem den großen Vorteil, dass sie unabhängig von
einer Ensemble-Interpretation ist, und sogar erlaubt, in
detaillierter Weise (und observablenabhängig) vorherzubestimmen,
wie groß ein Ensemble sein muss und wie unabhängig die einzelnen
Realisierungen, damit man brauchbare Vorhersagen bekommt.

Die Ensemble-Interpretation schweigt sich demgegenüber über die
nötigen Eigenschaften des Ensembles und die notwendige Größe
weitgehend aus.
L.E. Ballentine,
The Statistical Interpretation of Quantum Mechanics,
Rev. Mod. Phys. 42, 358-381 (1970),
der für die statistische Interpretation als massgeblich gilt,
redet sogar von 'infinite'...

-----------------------------------------------------
S2m. Der HERA Speicherring bei DESY als Quantensystem
-----------------------------------------------------

Betrachten wir einen Experimentator in einem typische Hochenergie-
Experiment. Er präpariert eine Versuchsanordnung (ein Einzelobjekt)
in einem bestimmten Zustand, in dem wiederholt Ereignisse des gesuchten
Typs stattfinden.

Die gesamte (einmalige) Versuchsanordnung [etwa der
HERA Elektron-Proton Speicherring bei DESY inklusive Messanordnung]
hat zu jedem Zeitpunkt einen wohldefinierten (unbekannten, gemischten)
Quantenzustand, von denen der Experimentator mindestens die für
seine Beobachtungen relevanten Kenngrößen kennt (die werden
nämlich vorher gut kalibriert).

Die Dynamik dieses Quantenzustands ist hinreichend genug bekannt,
um gewisse Voraussagen über seine änderung während der Dauer des
Experiments zu machen. Insbesondere gehören zu diesen Voraussagen die
über die Zeit gemittelten Erwartungswerte gewisser Observablen, aus
denen z.B. Streuquerschnitte berechnet werden können. Die kann man mit
den Messungen vergleichen.

All das ist objektiv. Man braucht kein Ensemble von DESY's,
um das gesamte System samt aller dort gemachter Beobachtungen
quantenmechanisch zu interpretieren. Man braucht weder subjektive
Elemente wie Deine Kenntnisse über DESY oder Dein Zurkenntnisnehmen
(oder nicht) von dort gemachten Beobachtungen noch esoterische
Annahmen der Art, dass die Quantenmechanik über ein einzelnes System
nichts aussage.




-------------------------------------
S2n. Kollaps und Relativitätstheorie
-------------------------------------


Die Kopenhagener Interpretation mit ihrem instantanen Kollaps des
Zustandes scheint der relativistischen Kausalitätsstruktur zu
widersprechen. Das ist aber nur bei oberflächlicher Betrachtungsweise
so. Die Argumentation (in etwas anderem Kontext) in
A. Peres
Classical interventions in quantum systems. II.
Relativistic invariance
arXiv:quant-ph/9906034
überträgt sich auf den Kollaps.
Die mikrokausale Struktur der Quantenfeldtheorie sorgt dafür,
dass Observable, die gleichzeitig an verschiedenen Orten gemessen
werden können, kommutieren, unabhängig von der Wahl der
Hyperfläche konstanter Zeit, solange sie nur raumartig ist.
Ein lokaler Kollaps ist also stets nur ein teilweiser, da man nie
ein vollständiges System kommutierender Observablen misst.


-----------------------------------------
S2o. Effektive quantenmechanische Modelle
-----------------------------------------

Quantenmechanische Modelle, die komplexere experimentelle Ergebnisse
reproduzieren, sind in der Regel effektive Modelle, die nicht
bei den fundamentalen Gegebenheiten ansetzen, sondern ''effektive''
Objekte (Constituent Quarks, Nukleonen, Atome, Valenzelektronen,
Leitungselektronen, Phononen, usw.) und empirisch zu fittende
''effektive'' Parameter enthalten.

Die Praxis in den meisten Anwendungen ist, dass man aus der
Quantentheorie unter vereinfachten Annahmen ein Modell herleitet,
und feststellt, dass dieses Modell mit geeigneten Parametern einen
wesentlich größeren Gültigkeitsbereich hat als die Herleitung
erkennen lässt. Man fittet daher die Parameter des Modells an gewisse
Schlüsseldaten und verwendet dann das Modell zur Vorhersage weiterer
Daten und zur Analyse der Experimente.

Das ist das Vorgehen nicht nur in der Quantenoptik, sondern auch in der
statistischen Physik, Atomphysik, Kernphysik und Elementarteilchentheorie.
Wenige rechnen direkt mit dem Standardmodell und den wahren
Fundamentalkonstanten (die übrigens auch gefittet werden müssen).
Die meisten benutzen abgeleitete effektive Theorien, Quarkmodelle,
Kernschalenmodelle, Schrödingergleichungen, Dichtefunktionale,
Zustandsgleichungen, deren Parameter zuerst an die vorhandenen Daten
angepasst werden.

Nur dank dieser Flexibilität ist die Quantenmechanik in der
Lage, so gut wie alle experimentellen Daten theoretisch zu erklären.




-----------------------------------
S2p. Quantenmechanik von Molekülen
-----------------------------------

In der Chemie (Thermodynamik, Phasengleichgewichte, chemische
Reaktionen, Infrarot- Raman, und Kernspinresonanz-Spektroskopie)
rechnet man grundsätzlich auf unterster Ebene in der
Born-Oppenheimer Näherung, wo man die Wellenfunktion in
psi = psi_Kerne * psi_Elektronen
faktorisiert. Sowohl Kerne als auch Elektronen sind hier
ununterscheidbar modelliert.

Da die Kerne viel schwerer sind als die Elektronen, reagieren sie so
langsam, dass sich in guter Näherung das Elektronengleichgewicht
einspielt, ohne dass sich die Kerne viel bewegt haben. Daher kann
man in der adiabatischen Näherung das Elektronenproblem abkoppeln
und die Kerne dort als klassisches externes Coulomb-Potential
modellieren.

In der Thermodynamik der Moleküle modelliert man auf der Ebene der
Atome. Die typischen chemischen Energien sind, wenn man von
Laser-induzierter Chemie und Ausnahmereaktionen absieht, so gering,
dass die Elektronen praktisch stets im Grundzustand sind.

Das ist das, was in typischen Quantenchemiepaketen geschieht:
Man rechnet sich den Grundzustand der Elektronenwolke in einer
passenden Approximation (z.B. Hartree-Fock, MP2, CC, DFP) aus
und bestimmt daraus die Ladungsdichte des Moleküls, in Abhängigkeit
von den (klassischen) Positionen x_1,...,x_N der N Kerne.
Außerdem bekommt man die Energie des Grundzustands, die aus der
Sicht der Kerne die potentielle Energie V(x_1,...,x_N) der Kerne
in den Positionen darstellen.

Eingesetzt in die Born-Oppenheimer Näherung und gemittelt über
den Elektronenzustand bleibt eine Vielteilchen-Schrödingergleichung
für die Wellenfunktion psi(x_1,...,x_n) der Kerne,
i hquer d/dt psi = (T+V) psi (*)
mit der kinetischen Energie T= sum p_i2/2m_i und dem obigen
effektiven Potential. Da die Elektronen heraus gemittelt wurden,
handelt es sich jetzt um durch Elektronen gedresste Kerne, also
Atome.

(*) ist immer noch eine Gleichung für ununterscheidbare Atome, die
Wellenfunktion psi ist nach wie vor ununterscheidbar, und das Potential
V ist symmetrisch in den Koordinaten, die zu gleichartigen Kernen
gehören.

Dummerweise kennt niemand dieses Potential (ausrechnen kann man es nur
punktweise, für jede Kombination von Kernkoordinaten eine Rechnung
eines Elektroneneigenwertproblems), und es ist außer für ein paar
Zwei- und vielleicht Dreiteilchenprobleme nicht einmal in groben Zügen
bekannt.

Außerdem gibt es auf der Ebene von (*) noch keine Chemie.
Denn die Chemie arbeitet
1. nur in einem kleinen relevanten Energiebereich, und
2. braucht sie Modelle, in denen man von Molekülen reden kann.
Das geht nur durch Symmetriebrechung.

Konkret setzt man bei einem einzelnen Molekül voraus, dass man schon
weiß, wo die Atome ungefähr sitzen, was man bei einem Molekül im
Schwerpunktsystem auch wirklich weiß. Das macht ja gerade die Chemie
aus, dass sie etwas über den Bau der Moleküle aussagen kann.
Man kann diese Kenntnis routinemäßig über Röntgenstrukturanalysen
oder verschiedener Arten von Spektroskopie erhalten. Zum Beispiel
bestimmt man über Röntgenstruktur oder NMR (Nuclear magnetic
resonance) Spektroskopie experimentell die Struktur biologischer
Proteine. über Proteinfaltung (das entsprechende Problem der
Strukturvorhersage) habe ich vor vielen Jahren einen dicken,
vielzitierten Überblicksartikel geschrieben (author:Neumaier protein
in scholar.google.com).

Damit ist die Symmetrie gebrochen. Es bleibt nur noch eine
reduzierte Symmetrie, nämlich all das, was von der Strukturformel
nicht festgelegt ist. Die bestimmt das notwendige Ausmaß der
Symmetrisierung in der molekularen Wellenfunktion.

Dass die Symmetrie hier wirklich gebrochen ist, sieht man an der
fehlenden Symmetrie der in der Chemie benutzten effektiven Potentiale
(potential energy surfaces = force fields). Der Hamiltonoperator H
eines Moleküls enthält nur noch die reduzierte Symmetrie, die sich
aus der Strukturformel ergibt. Würde man gleichartige Atome trotzdem
voll symmetrisieren, so würde das einer Modellierung im Hilbertraum
der voll symmetrisierten Wellenfunktionen entsprechen.
Der Hamiltonoperator bildet diesen Raum aber nicht in sich selbst ab:
Ist psi vollständig symmetrisiert, so ist H psi praktisch nie
vollständig symmetrisiert, wie man ohne große Rechnung sieht.


Ein System aus N Argonatomen im Gas hat eine reduzierte Symmetriegruppe
der Ordnung N!, da keine Bindungen vorliegen und damit keine
Einschränkung des Orts vorliegt. Ein System aus N Argonatomen im
gebundenen Cluster hat dagegen nur noch eine geringe Symmetrie,
die der des Clusters entspricht. Für N-->unendlich ergibt
sich hieraus, dass im festen Argon alle Atome voll unterscheidbar sind.

Ein System aus N H_2-Molekülen hat eine reduzierte Symmetriegruppe
der Ordnung N!*2^N, da man die Moleküle nicht unterscheiden kann
und innerhalb jedes Moleküls die beiden H_Atome nicht.

Ein einzelnes Molekül Ameisensäure H-CO-O-H (wobei das O des CO
aus der Schreibebene herausschauen sollte) hat keine Symmetrie mehr,
und ein System von N solchen Molekülen hat eine reduzierte
Symmetriegruppe der Ordnung N!.

So kommen also die Symmetriefaktoren zustande, und entsprechend dieser
reduzierten Symmetrie müssen die Wellenfunktionen symmetrisiert werden.

In der konkreten Arbeit sucht man also das Potential in einer
Umgebung einer festen Referenzlage (oder im Fall chemischer Reaktionen
mehrerer solcher) zu beschreiben, und kann durch eine beschränkte
Zahl von ab initio Rechnungen (d.h. der Lösung des entsprechenden
Elektroneneigenwertproblems) und multivariaten Datenfits
ein im interessierenden Bereich brauchbares effektives Potential
bekommen. Daraus bekommt man mit dem bekannten quantenmechanischen
Instrumentarium die Gleichgewichtsstrukturen, Spektren, freie Energien,
Ladungsdichten, Reaktionsraten, usw., die man mit dem Experiment
vergleichen kann.

Für die Interpretation von Ladungsdichten und die resultierende
operative Vorstellung der Chemiker von Atomen in Molekülen siehe
z.B. den Enzyklopädie-Artikel
R.F.W. Bader
Atoms in Molecules
http://59.77.33.35/non-cgi/usrd8wqiernb/5/20/Atoms20in20Molec_1193580192.pdf
oder Baders Webseiten
http://www.chemistry.mcmaster.ca/faculty/bader/aim/aim_0.html


-----------------------------------------
S2q. Der Hilbertraum des Wasserstoffgases
-----------------------------------------

Der Hamiltonoperator eines Wasserstoffmoleküls H_2 ist
H_1(p,q) = p_1^2/2m +p_2^2/2m +V(|q_1-q_2|),
wobei p=(p_1,p_2) und q=(q_1,q_2) beide 6-dimensional sind
und V(r) ein 1-dimensionales Potential ist mit einem Pol bei r=0,
einem Minimum mit V(r_0)<0 beim Gleichgewichtsabstand r_0 der Atome
eines Wasserstoffmoleküls, und V(r) --> 0 für r --> unendlich.

Der Ein-Molekül-Hilbertraum ist das symmetrisierte Tensorprodukt
V_1 = W_1 v W_1
des Hilbertraums W_1=L^2(R^3) des Wasserstoffatoms, wenn wir Spin
ignorieren. (v ist das symmetrisierte Tensorproduktzeichen.)

Der Hamiltonoperator eines aus N Wasserstoffmolekülen H_2
bestehenden Gases ist
H_N = sum_{k=1}^N H_1(p^k,q^k),
wo
p^k=(p_1^k,p_2^k), q^k=(q_1^k,q_2^k)
die Koordinaten des k-ten Moleküls sind. Er operiert auf dem
N-Moleküle-Hilbertraum
V_N = V_1 v ... v V_1 (N Faktoren)
der bezüglich der q^k symmetrisierten Wellenfunktionen.
Aus der entsprechenden Darstellung rechnet man die
freie Energie von gasförmigem Wasserstoff korrekt aus.

Die bezüglich aller 2N H-Atome symmetrisierten Wellenfunktionen
liegen im 2N-Atome-Hilbertraum
W_2N = W_1 v ... v W_1 (2N Faktoren).
H_N und W_2N sind physikalisch nicht äquivalente Hilberträume.
W_2N ist nur ein winziger Teilraum von V_N.

Der Hamiltonoperator H_N bildet diesen Raum nicht in sich selbst ab:
Ist psi vollständig symmetrisiert, also psi aus W_2N,
so ist H_N psi praktisch nie vollständig symmetrisiert, wie man
ohne große Rechnung sieht.

Wellenfunktionen von teilweise oder ganz in Molekülen gebundenen
gleichartigen Atomen dürfen also nicht vollständig symmetrisiert
werden, sondern nur soweit, wie es die Symmetrie der vorliegenden
Bindungen erlaubt.

Darstellungstheoretisch ist das ein gewaltiger Unterschied, der
sich auch beim Rechnen bemerkbar macht! Die Symmetriegruppe ist
von Sym(2N) auf 2^N Sym(N) gebrochen, und entsprechend reduzieren
sich die Symmetriefaktoren.



------------------------
S2r. Genauigkeit der QED
------------------------

QED in der Form der Dirac-Fock-Approximation
und CI- , MCHF- oder CC-Rechnung ergibt z.B. (bezogen auf
chemische experimentelle Daten) auf 0.001% genaue
Energielevels für viele Atome und kleine Moleküle.
Selbst für schwere Atome ergeben sich gute Werte.

Auf dem GSI Website gibt es z.B. die folgende Seite:
Probing Quantum Electrodynamics in Strong Fields
Lamb-Shift experiments on high-Z one-electron ions
http://www-ap.gsi.de/Thomas/ap_html_research/gsi-qed_2.htm
Dort findet man die folgenden Angaben aus dem Jahr 2003:

Grundzustandsenergie (= Bindungs-/Ionisierungsenergie)
eines Elektrons um einen Urankern: 131816 eV
Lamb Shift eines Elektrons um einen Urankern: 464 eV
Experimentelle Genauigkeit 2003: +- 13 eV
(2.8% des Shifts, 0.01% des Energieeigenwerts)
Theoretische Genauigkeit der Vorhersagen: +- 1 eV
(0.2% des Shifts, 0.0008% des Energieeigenwerts)
Die Theorie steht also gut im Rennen.
Abweichungen zum Experiment werden nicht berichtet.


Probleme mit Näherungsverfahren in der QED ergeben sich allgemein
nur in starken Feldern.
Uran hat ein wesentlich stärkeres Zentralfeld als die kleinen Atome.
Damit sind die relativistischen Störungen, die berücksichtigt
werden müssen, mit einfacher Störungstheorie nicht mehr zuverlässig
berechenbar. Auch ist der magnetische Formfaktor des Kerns hier schon
von Bedeutung und nur ungenau bekannt.

Ein Großteil der Chemie spielt sich aber mit Atomen der
Kernladungszahl Z<=8 ab, wo die Kräfte und die resultierenden
Geschwindigkeiten wesentlich kleiner sind. Die Verhältnisse
sind daher viel übersichtlicher, und mit einfacher Störungstheorie
ist eine noch höhere Genauigkeit erreichbar.





------------------------------
S3a. Beobachter, Raum und Zeit
------------------------------

Ein Beobachter im Sinn der speziellen Relativitätstheorie
hat einen zeitartigen 4-Impuls p und damit eine positive Masse
m := sqrt(-p^2)/c,
wo (\ = fett, Signatur -+++, p^2 = p dot p)
p^2 := \p^2 - p_0^2.
Das hat zur Folge, dass ein Beobachter, der zur Zeit t
im Punkt x(t) ist, den Raum
R(t) := {x | p dot x = p dot x(t)}
als seinen affinen Ortsraum (Eigenraum) zur Zeit t erlebt.
Im Lauf der Zeit sieht er die ganze Schar der parallelen Räume
R_tau := {x | p dot x = tau}.
Genauer sieht ein gleichförmig bewegter Beobachter
den Raum R_tau zur Zeit
t := t0 + (p dot x0 - tau)/mc^2.
wenn er zur Zeit t=t0 im Punkt x0 ist.

Da p zeitartig ist, ist nämlich
(x-y)^2 >=0 für x,y in R,
also ist R(t) ein 3-dimensionaler Euklidischer Raum mit Abstand
d(x,y) = sqrt((x-y)^2).
Für einen sich gleichförmig bewegender Beobachter (p = const)
ist die 4-Geschwindigkeit definitionsgemäss konstant gleich
xdot(t) = v := p/m,
normiert zu
v^2 = p^2/m^2 = -(mc)^2/m^2 = -c^2,
also ist seine Weltlinie durch
x(t) = x0 + (t-t0) v = x0 + (t-t0)/m p
gegeben. Wegen
p dot v = p dot p/m = p^2/m = -(mc)^2/m = -mc^2
findet man
tau(t) = p dot x(t) = p dot x0 + (t-t0) p dot v
= p dot x0 -mc^2 (t-t0),
also ist seine Zeit gegeben durch
t = t0 + (p dot x0 - tau)/mc^2.

Man kann statt dessen die Weltlinie einfacher durch
x(s) = x0 + s p,
parametrisieren, wo x0 ein fixer Punkt auf der Weltlinie
ist und s ein Parameter. Wir wollen mal x0 das 'Hier'
und s den 'Nu' (englisch 'instant') nennen.
Der durch s=0 definierte Zeitpunkt t0 ist dann das 'Jetzt'.
Damit bekommt man zwischen Nu s und Zeit t die Beziehung
t = t0 + s m.
x0 und t0 haben keine absolute Bedeutung, da Ort und Zeit nicht
absolut gemessen werden können, sondern nur Orts- und Zeitdifferenzen.
Der Nu misst hingegen vom Bezugspunkt (dem 'hier und jetzt') aus und
macht für jeden Beobachter Sinn, der den Bezugspunkt kennt.


Ein auf einem Lichtstrahl platzierter ''Beobachter'' ist aber kein
Beobachter mehr im oben präzisierten Sinn. Das Absprechen des
Beobachterstatus hat schon ganz praktische Gründe.
Z.B. muss ein auf einem Lichtstrahl platzierter ''Beobachter''
masselos sein, sonst fällt er sofort vom ihn befördernden Photon
herunter. Der ''Beobachter'' könnte also höchstens das
Photon selbst sein. In welchen Sinn es aber ohne innere Struktur
beobachten können sollte (es kann keine Messinstrumente,
Aufzeichnungen, und ähnliches bei sich haben), bleibt im Dunkeln...

Man kann aber durchaus auf der rein mathematischen Ebene diskutieren,
wie die Welt von einem Photon wahrgenommen würde, wenn es beobachten
könnte... Ein auf einem Lichtstrahl platziertes, sich also gleichförmig
bewegendes und masseloses Photon erlebt die Welt total anders als
ein 'klassischer' Beobachter!!!

Für ein Photon ist m=0 und p daher lichtartig,
p^2 = -(mc)^2 = 0.
Eine 4-Geschwindigkeit lässt sich nicht mehr definieren
(Division durch Null). Man kann die Weltlinie des Photons
(d.h. den Lichtstrahl) aber nach wie vor durch den Nu s
parametrisieren,
x(s) = x_0 + s p.
Die Beziehung zwischen Nu und Zeit wird zu
t = t0 + s m = t0
unabhängig von s. Das heißt, das Photon erlebt
zwar den Nu, aber keine Zeit.

Immerhin erlebt ein Photon den Raum, aber ebenfalls auf
seltsame Weise: Sein affiner Ortsraum
R(s) := {x | p dot x = p dot x(s)}
ist ein für allemal fix,
R(s) = R_tau, tau unabhängig von s
(aber abhängig von der Weltlinie als Ganzes),
da wegen p dot p = 0 auch
p dot x(s) = p dot (x_0 + s p) = p dot x_0
von s unabhängig ist.

Eigentlich kein Wunder - da für ein Photon die Zeit
stehenbleibt, lernt es von der 4-dimensionalen Raumzeit
nur drei Dimensionen kennen!

Sein Ortsraum hat außerdem einige Merkwürdigkeiten:
Wegen p dot p = 0 ist die Metrik degeneriert und nur noch
semidefinit. Der Abstand
d(x,y) = sqrt((x-y)^2).
hat die Signatur ++0 und verschwindet für alle Punkte x,y,
deren Verbindungsstrecke parallel zur Weltlinie ist.
Das Photon kann also nur zwei Raumdimensionen ausmessen,
die dritte (zum Impuls parallele) ist ''gleichortig'',
im selben Sinn wie in Newtons Raumzeit verschiedene Orte
gleichzeitig sind. Der 3-dimensionale Ortsraum hat also zwei
raumartige Dimensionen und eine lichtartige, ist also sozusagen
eine 3-dimensionale 'Raumlicht' (statt der 4-dimensionalen Raumzeit
des massiven Beobachters.)


Insbesondere befindet sich das Photon immer am 'gleichen' Ort in diesem
metrischen Sinn: Es bewegt sich nämlich mit fortschreitendem Nu auf
der in seinem 3-dimensionalen affinen Ortsraum befindlichen Weltlinie;
aber die Punkte dieser Weltlinie haben alle den Abstand null
voneinander. (Man nennt das die transversale Natur des Photons.)

Ein echter (massiver) Beobachter läuft dagegen stets senkrecht
aus seinem aktuellen Raum hinaus und in den infinitesimal
daneben liegenden Raum hinein.


Interessant ist auch, was zwei (massive) Beobachter, die nicht im
selben Boot (d.h. auf derselben Weltlinie) sitzen, erleben.
Die beiden von ihnen wahrgenommenen Räume haben (generisch) nur
zwei Dimensionen gemeinsam, nicht drei; sie leben daher in
verschiedenen Welten.

Kein Wunder, dass es unter uns Menschen so viele
Missverständnisse gibt, weil jeder seine eigene Welt hat !-)


Man hat also eine 4-dimensionale Version des Phänomens der
Perspektive. Eine freistehende Wand kann ganz breit oder ganz
schmal aussehen, je nachdem von wo aus man drauf schaut.
Natürlich ändert sich die Wand nicht durchs Draufschauen,
aber es sieht so aus. Das gibt den Lorentzkontraktionen eine
anschauliche Interpretation.



----------------------------------------
S3b. Was heißt 'homogen' und 'isotrop'?
----------------------------------------

Homogen = man kann nicht unterscheiden, wo man ist, da die Welt
von allen Punkten aus exakt gleich aussieht. Auf mathematischer
Ebene heißt das, dass es zu je zwei Raumzeitpunkten x und y einen
Diffeomorphismus gibt, der x auf y abbildet und alle vorhandenen
Felder unverändert lässt. Das bedeutet insbesondere, dass die
Materie (in der homogenen Idealisierung) gleichmäßig im All
verschmiert ist. Sonst könnte man einen materiell besetzten Punkt
(z.B. eine Sonne) von einem leeren Punkt (z.B. irgendwo
zwischen Erde und Mond) unterscheiden, was im homogenen
Fall definitionsgemäß nicht möglich ist.

Isotrop = man kann nicht unterscheiden, in welche Richtung man
schaut, weil es in jeder Richtung gleich aussieht.
Auf mathematischer Ebene heißt das, dass es zu je zwei Richtungen
u und v (Vektoren der Länge 1) einen Diffeomorphismus gibt,
der den Ort des Beobachters fest lässt, u in v überführt und
alle vorhandenen Felder unverändert lässt. Also wie es etwa
ein intelligenter Punkt mitten im Meer bei Windstille und
gleichmäßig bedeckten Himmel erleben würde.

Offensichtlich ist das ebenfalls eine Idealisierung, die darauf
beruht, dass man nicht so genau hinschaut. Das hat dafür den
Vorteil, dass man etwas rechnen kann - ohne diese Vereinfachung
wäre die Kosmologie hoffnungslos kompliziert. Zum Glück ist
die Natur so aufgebaut, dass das grobe Hinsehen alles vereinfacht...



---------------------------------------------
S3c. Wie homogen und isotrop ist das Weltall?
---------------------------------------------

Wie im Abschnitt ''Was heißt 'homogen' und 'isotrop'?''
beschrieben, kann das Universum nicht völlig homogen und
isotrop sein, da man sonst nicht zwischen leeren und
materiebesetzten Teilen des Universums unterscheiden könnte.

Innerhalb eines einzelnen Galaxienhaufens ist das
Universum offensichtlich weder isotrop noch homogen,
da ein Beobachter sich dort an den vorhandenen Galaxien
orientieren kann und ein kanonisches Koordinatensystem
- etwa an Hand der Entfernungen zu ausgezeichneten
Galaxien(schwerpunkten) - bilden kann.

Andrerseits ist das reale Universum approximativ ein
Robertson-Walker-Friedmann-Universum - aber erst in
einem Maßstab betrachtet, in denen Galaxienhaufen als
punktförmig angesehen werden können, und in einer
Genauigkeit, in der man die entstehende Punktwolke
als gleichförmige ideale Flüssigkeit betrachten kann.

Man beobachtet systematische Rotverschiebung nämlich nur
bei den entfernten Galaxien, die nicht mehr zum Virgo
Supercluster, in dem unser Planetensystem sitzt liegen.
Siehe http://en.wikipedia.org/wiki/Supercluster Wenn man das nicht beachtet, kommt man leicht zu unsinnigen Vorstellungen. Da es allerdings extrem viele (vermutlich ca. 10^7) solcher Haufen gibt, kann man sie näherungsweise als gleichförmige Verteilung ansehen, und die ist (bis auf relative Abweichungen von ca. 0.002%) homogen und isotrop, und war es seit der Zeit der Entkopplung der Hintergrundstrahlung, wie man auf Grund von Messungen der Hintergrundstrahlung weiß. Nur die Dichte dieser Wolke von Haufen verringert sich mit der Zeit, da auf Grund der Expansion des Universums alle Abstände ebenfalls im statistischen Mittel mit der Zeit immer größer wurden. Kosmologische Modelle beschreiben das Universum daher in der Regel als ideale Flüssigkeit, die sich mit Gravitationstheorie und relativistischer Hydrodynamik beschreiben lässt. Diese Theorie vernachlässigt allerdings die körnige Detailstruktur des Weltalls und die dadurch verursachte Dissipation. Wie weit sich das auf unsere Rückschlüsse auf das ganz frühe Universum auswirkt, entzieht sich meiner Kenntnis. ------------------------------------------------------- S3d. Wie soll man sich den gekrümmten Raum vorstellen? ------------------------------------------------------- Wer sich ein mechanisches Spannungsfeld in einem durch äußere oder innere Kräfte verformten elastischen Körper vorstellen kann, hat schon eine Vorstellung von dem, was Physiker ein symmetrisches Tensorfeld nennen, in diesem Fall vom Spannungstensor. In der Elastizitätstheorie gehört dazu ein weiteres symmetrisches Tensorfeld, der Verschiebungstensor, der angibt, wie sehr der Körper im Vergleich zur Gleichgewichtslage verformt ist. Der gekrümmte Raum ist einfach ein symmetrisches Tensorfeld im Raum, analog zum Verschiebungstensor. Das Weltall wird sozusagen als elastischer Körper betrachtet, der von den vorhandenen Massen gegenüber der flachen (Newtonschen) Gleichgewichtslage verformt ist. In der Regel nur ganz wenig, außer in der Nähe von schwarzen Löchern oder tief im Innern genügend schwerer Sterne. Damit hat man eine klare Vorstellung, die nicht trügt - jedenfalls nicht, solange topologische Komplikationen keine Rolle spielen, der Raum also zu R^3 diffeomorph ist. Als flacher Vergleichsraum kann der Tangentialraum des Beobachters dienen, oder der eines daraus abgeleiteten Bezugssystems; für astronomische Beobachtungen z.B. den Tangentialraum der Sonne. Die Subjektivität, die in der Wahl des flachen Bezugssystems liegt, ist analog zur Subjektivität in der Wahl des flachen Koordinatensystems für die Gleichgewichtsform eines elastischen Körpers. Man kann das Verschiebungsfeld des gekrümmten Raums messen, indem man die Abweichungen von der Newtonschen Theorie misst und an die Formeln der Post-Newton-Näherung der allgemeinen Relativitätstheorie fittet. Man schaut sich also die Dynamik von messbaren Objekten an und misst ihre scheinbare Trajektorien. Das gibt genug Daten, um Abweichungen von der Newtonschen Theorie feststellen zu können. So waren etwa die Merkuranomalien schon lange vor Einstein gemessen worden. Ein mit der Messgenauigkeit konsistenter Fit des Planetensystems an die Newtonschen Bewegungsgleichungen war wegen dieser Anomalien schon damals nicht möglich; ein Fit an die Bewegungsgleichungen in einer Schwarzschildmetrik erwies sich dagegen als konsistent. Die scheinbaren Trajektorien enthalten also Information über die Krümmung. In der allgemeinen Relativitätstheorie ist allerdings die Metrik ein 4-dimensionaler symmetrischer Tensor, und die Raumkrümmung nur der Raumteil Teil dieses Tensors. Der Rest wird in der Lapse-Funktion und dem Shift-Vektor kodiert (s. etwa http://www.tat.physik.uni-tuebingen.de/~lehle/diplom/node9.html ), die natürlich auch in die Dynamik eingehen und mit gefittet werden müssen. ---------------------------- S3e. Universum ohne Urknall? ---------------------------- In der Tradition der Astrophysik spricht man davon, dass als Folge des Urknalls und der resultierenden Expansion des Weltalls der Raum zwischen den Galaxienhaufen immer leerer wird, sozusagen, weil sich letztere immer weiter voneinander entfernen.

Der Raum ist immer expandiert, nach den gängigen, als gesichert
geltenden Vorstellungen. In Wirklichkeit wissen wir natürlich
über die ferne Vergangenheit nur wenig und extrapolieren mit Hilfe
von Modellen, die wir an heutigen Daten mehr oder weniger gut
überprüfen können.

Die Größe des Weltalls ist durchaus bestimmbar relativ zu
den Maßstäben, die wir hier auf der Erde definiert haben.
Und wenn man diese als konstant betrachtet, was man nach
der ART darf (aber nicht muss) und was wir per Konvention tun,
dann wächst das Weltall ständig. Und das nennen wir die
Expansion.

Man sagt, man 'weiß', wenn sich alles konsistent in einer
einzigen Theorie interpretieren lässt. Und das ist hier
der Fall - jedenfalls, wenn man die ersten Bruchteile einer
Sekunde außer Acht lässt.


Man könnte allerdings ebenso konsistent sagen, dass der Raum schon
immer gleich groß war und sich nur unsere Maßstäbe immer weiter
verkleinern. Im Sinne der ART ist das nur eine
Koordinatentransformation (Diffeomorphismus).

In diesem Bild schrumpfen dann die Galaxienhaufen und alles,
was darinnen ist, entsprechend mit der Zeit; in der Vergangenheit
waren die Maßstäbe entsprechend riesig.

Man kann natürlich auch die Zeit beliebig monoton transformieren,
ist ebenfalls nur ein Diffeomorphismus, und damit den Urknall auf
t --> minus unendlich
verschieben.

Damit hat man ein plausibles, konsistentes Bild ohne Urknall.
Ist natürlich weniger mediengerecht und darum nicht beliebt.
Außerdem macht das Rechnen in einem solchen Koordinatensystem
etwas mehr Arbeit.

Es befreit einen aber nicht davon, dass auf der mathematischen
Ebene alle Weltlinien nach endlicher Eigenzeit in die Vergangenheit
eine Singularität haben. Aber diese Weltlinien lassen sich
physikalisch nicht beliebig weit zurückverfolgen, da eine
physikalische Weltlinie physikalisch identifizierbare Einzelobjekte
erfordert, die es im frühen Universum nicht geben konnte.




-----------------------------
S3h. Das Alter des Universums
-----------------------------

Wie kann man angesichts der Relativitätstheorie von einem
wohldefinierten ''Alter des Universums'' reden?

Obwohl die Allgemeine Relativitätstheorie invariant unter ist,
ist es die Natur nicht. Sie hat ein ausgezeichnetes Koordinatensystem,
nämlich das Ruhsystem der Materie, die in allen kosmologischen
Modellen als homogene und isotrope Flüssigkeit betrachtet wird.
Dieses Koordinatensystem ist bis auf eine räumliche Drehung durch
die Wahl des Ursprungs ('hier und jetzt') eindeutig bestimmt.
Um von Alter zu sprechen, muss man den Raum-Zeit-Punkt festlegen,
von dem aus man das Alter misst, Dies ist beobachterabhängig,
da ein Beobachter, der vor 500 Millionen Jahren gelebt hat, natürlich
ein anderes Alter ermittelt als wir heute. Die Ursprungs- und
Rotationsabhängigkeit des Koordinatensystems hat auf Zeitdifferenzen
(also Alter) keinen Einfluss. Damit ist das Alter des Universums
(und ebenso das 'jetzt' - also eine raumartige Hyperfläche, die
die gegenwärtigen Ereignisse definiert) durch Ort und Zeit des
Beobachters festgelegt.

Das hat nichts Subjektives an sich. Man kann das Alter des Universums
relativ zu einem Raumzeitpunkt auch koordinatenunabhängig definieren
als die Länge der längsten zeitartigen, in die Vergangenheit
gerichteten Geodäte. Das Ergebnis ist dasselbe; die Geodäte endet
in einer Singuarität, die man gewöhnlich 'den Urknall' nennt.

In einem räumlich isotropen Modell des Universums (üblicherweise
dem Robertson-Walker-Modell) ist das Alter des Universums die
ausgezeichnete Zeitkoordinate.

Für die Astronomen ist der Ursprung konventionell
der Mittelpunkt der Erde (oder unserer Galaxis) zum Zeitpunkt
01.01.1900 oder ein ähnliches Datum; für kosmologische
Betrachtungen kommt es auf ein paar hundert Jahre mehr oder weniger
und auf ein paar Hundert Millionen Lichtjahre nicht an, und die
Angaben sind im Wesentlichen dieselben, ob wir den Beobachter
als im konventionellen Ursprung befindlich annehmen, oder am Ort
und zur Zeit, wo die Frage 'Wie alt ist das Universum?' gestellt wird.


Für die verwandte Frage, wie man zur Behauptung kommt, wir sähen
den Andromedanebel heute so, wie er vor 2.5 Millionen Jahren in
Wirklichkeit war (obwohl diese Aussage keinen relativistisch kovarianten
Sinn macht) siehe das theoretical physics FAQ auf
http://www.mat.univie.ac.at/~neum/physics-faq.txt unter dem Thema ''What is time?'' ---------------------------------- S3g. Woraus besteht das Universum? ---------------------------------- Die Bestandteile des Universums sind gerade die Objekte (samt ihren Eigenschaften), die das Standardmodell und die allgemeine Relativitätstheorie behandeln. Die Information dazu findet man in Büchern über Standardmodell und Relativitätstheorie. Als (anspruchsvolles) Überblicksbuch empfehle ich das Buch PDB Collins, AD Martin, EJ Squires Particle physics and cosmology Wiley, New York 1989 Es gibt einen schönen Überblick, behandelt auch die offenen Fragen und Alternativmodelle, und hat genug Referenzen zur Vertiefung. Man sollte aber schon ein bisschen Quantenfeldtheorie und Relativitätstheorie gesehen haben... Interessiert man sich nur für die Welt im Großen (so ab Sterngröße), und ignoriert man elektromagnetische Felder (die Sterne i.A. haben), so kann man das Standardmodell praktisch vergessen (außer für das ganz frühe Universum), und alles wird vergleichsweise einfach. Aber etwas Differentialgeometrie muss man dazu schon zu lernen bereit sein. (Die fehlende Mathematik kann man sich aus Büchern aneignen. Ohne Mathematik ist es wirklich so gut wie unmöglich, die moderne Physik zu verstehen. Man kann natürlich einfach Hawkings Kleine Geschichte der Zeit oder ähnliches lesen; das gibt qualitative Einsicht, soweit sie ohne Mathematik verständlich ist. Aber das ist doch eine grobe Vereinfachung dessen, was man verstehen kann, wenn man sich um die mathematischen Hintergründe kümmert.) Auf dieser Ebene (Sterne und größeres, ohne elektromagnetische Feinheiten) besteht die Welt aus einer 4-dimensionalen Mannigfaltigkeit von (Raumzeit-)Punkten, einem Materiefeld, beschrieben durch einen Energie-Impulstensor, ein Feld mit 10 Komponenten und aus einem Gravitationsfeld, beschrieben durch die Metrik, ebenfalls ein Feld mit 10 Komponenten. Das ist alles. Ein Feld mit n Komponenten ordnet jedem Punkt
n Zahlen zu (jedenfalls, wenn man sich mal für ein
Koordinatensystem entschieden hat).

Dann kommen als Konsistenzbedingungen noch ein paar sogen.
Feldgleichungen dazu, die insbesondere die Vergangenheit und
Zukunft aus der Gegenwart vorherzusagen erlauben.

Um die Felder wirklich zu 'kennen', braucht man dann natürlich
jede Menge Beobachtungsdaten. Die besorgt die Astronomie.
Sterne sind einfach Regionen der Raumzeit, in denen der
Energie-Impuls-Tensor signifikant von Null verschieden ist.


Koordinaten sind allerdings subjektive Größen.
Ein Stern 'hat' Koordinaten in den Augen dessen, der
das Koordinatensystem definiert hat. Ändert man das letztere,
ändern sich auch die Koordinaten. Objektiv sind
nur die raumzeitlichen Abstände zwischen Teilchen, aber keine
Koordinaten. Um Koordinaten wenigstens einigermaßen
objektiv zu definieren, muss man (subjektiv) 4 Positionen,
also ausgezeichnete Sterne wählen, und diese dazu benutzen,
wie im GPS (Global Positioning System, Details siehe
http://www.kowoma.de/gps/) andere Punkte durch 4 relative Koordinaten
(da unser universum 4-dimensional ist) zu bezeichnen.
Differentialgeometrisch nennt man das, was sich so ergibt,
eine Karte, die wenigstens einen Teil des Universums mit
Koordinaten versieht.



-------------------------------
S3h. Ist die Welt determiniert?
-------------------------------

Wir werden es wohl nie entscheiden können; es könnte aber durchaus
sein, und mir erscheint es sehr wahrscheinlich, trotz der
Quantenmechanik, die dagegen gewöhnlich ins Feld geführt wird.
(Warum Quantenargumente nicht ziehen, siehe die Diskussion
der Thermischen Interpretation unten ab S10.)


Determiniert heißt in der naturwissenschaftlichen Terminologie,
dass die vollständige Kenntnis der Gegenwart die vollständige
Kenntnis der Vergangenheit und Zukunft impliziert. Die Gegenwart
vollständig zu kennen ist uns Menschen aber verwehrt.

Wir kennen einzeln vielleicht ein paar Gigabyte an Informationen
über die Welt und kollektiv vielleicht ein paar Milliarden Gigabytes
an unterschiedlicher, zuverlässiger Information.
Daran wird sich auch in der Zukunft nichts Gravierendes ändern;
vielleicht ein Faktor 1000 oder 100000, wenn man großzügig ist.

Aber schon die genaue Beschreibung des Abstands von zwei Punkten
enthält unendlich viel Information, die uns mangels exakter
Messbarkeit nicht verfügbar ist. Vom genauen Zustand der ganzen
Welt (und ihrer mikroskopischen Details) ganz zu schweigen...


Dass diese Unkenntnis relevant ist, zeigen schon winzige Modellwelten:

Ein 3-dimensionales chaotisches System - wie das den Lorentz-Attraktor
bestimmende - ist determiniert im gebräuchlichen Sinn des Wortes,
nämlich dass die Anfangsbedingungen - sechs reelle Zahlen -, wenn
exakt bekannt, den Zustand zu jeder beliebigen Zeit festlegen.

Da wir aber nur mit endlicher Genauigkeit messen können,
haben wir nie genug Information über das System, um es exakt zu
kennen (auch nicht nach beliebig langer Zeit!), und jegliche
Ungenauigkeit macht die Vorhersage der Zukunft nach relativ kurzer
Zeit unmöglich.

Dasselbe gilt auch für die Vergangenheit, auch diese ist aus
gegebenen Bedingungen in der Gegenwart nach relativ kurzer
Zeit unmöglich vorherzusagen. Und auch ein simulierender idealer
Computer mit beschränkter Speicherkapazität macht nach kurzer
Zeit riesige Vorhersagefehler!

Was für kleine chaotische Systeme wie das Lorentz System gilt,
gilt erst recht für große chaotische Systeme wie unsere Welt.

Wir wissen z.B. über den Zustand der Welt am 09.02.1009 nur eine
extrem winzige Menge von Bits, im Vergleich zu unserer Kenntnis
vom Zustand heute, am 09.02.2009; allerdings wegen vorhandener
schriftlicher Aufzeichnungen, denen wir vertrauen, ein klein wenig
mehr als über den Zustand der Welt am 09.02.3009. Und wenn wir
- statt alten Aufzeichnungen einfach zu vertrauen - an die Geschichte
denselben strengen Maßstab stellen würden wie an die
Naturwissenschaft, wäre der Unterschied im Wissen über den
09.02.1009 und den 09.02.3009 kaum der Rede wert.


Dass wir die Vergangenheit als determiniert und die Zukunft
als nicht determiniert betrachten wollen ist einfach eine Folge
der Tatsache, dass unser (individuelles und kollektives) Gedächtnis
so gebaut ist, dass wir ein paar für uns markante Bits aus der
Vergangenheit in die Gegenwart retten, aber das Markante der Zukunft
uns überrascht - wenn etwas vorhersehbar ist, betrachten wir es ja
als Gesetz, nicht als markantes Ereignis. Das Wesen der Welt hängt
aber nicht vom Denkvermögen von ein paar intelligente Stäubchen
in ihr ab.

Es gibt also keinen stichhaltigen Grund, die Vergangenheit
für besser determiniert als die Zukunft zu halten. Ist also die
Zukunft nicht determiniert, so auch die Vergangenheit nicht.
Ist aber die Vergangenheit determiniert, wofür man gute Gründe
ins Feld führen kann, so kann man mit demselben Recht die
Zukunft für determiniert halten.



---------------------------------------------------
S3i. Diffeomorphismeninvariante klassische Mechanik
---------------------------------------------------

Man darf die Noether-Erhaltungsgröße eines mechanischen Systems
nicht kritiklos mit der Energie in eins setzen.

In der traditionellen Lagrange-Formulierung der allgemeinen
Relativitätstheorie (ART) ist der Noether-Energieimpulstensor
des Systems aus Gravitation+Materie ist identisch Null.
Noether-Energie bleibt insofern erhalten.

Aber mit unserer Vorstellung von Energieerhaltung hat das nichts
mehr zu tun, da die Noether-Energie auch nicht fließt:
der Strom ist auch identisch Null.

Was wir unter Energie verstehen, ist ein Raumintegral über
den Energieimpulstensor der Materie allein, und dafür gibt es
keinen Erhaltungssatz in Integralform.


Etwas Analoges gilt auch in der Newtonschen Mechanik von
N-Teilchensystemen. Hier ist die relevanten Mannigfaltigkeit
1-dimensional (Zeit alleine), und Diffeomorphismen sind stetig
differenzierbaren bijektiven Transformation der Zeit, deren
Umkehrabbildung ebenfalls stetig differenzierbar ist.
Der Raum besteht sozusagen nur aus den in der Wirkung betrachteten
N Massenpunkten; die Raumzeit besteht aus der Vereinigung der
Weltlinien aller Massenpunkte. Das vereinfacht die Theorie ungemein.

Nehmen wir z.B. die Lagrangefunktion
L(q,qdot,t) := U(q(t)) qdot(t),
wo q ein n-dimensionaler Spaltenvektor und U ein n-dimensionaler
Zeilenvektoren ist. Die Wirkung
S = integral L(q,qdot,t) dt
ist diffeomorphismeninvariant, d.h., die Wirkung ändert sich
nicht unter einem beliebigen Diffeomorphismus.
Nachzurechnen mit der Substitutionsregel.

Es passiert daher genau dasselbe wie in der ART:
Die Noether-Energie ist identisch null und hat keinerlei
physikalischen Inhalt. Denn man kann ein beliebiges
Hamiltonsches System
xdot=H_p(p,x) , pdot=-H_x(p,x)
in die obige Form bringen, indem man
q^T = (x^T,p^T,s),
U(q) = (p^T,0^T,-H(p,x))
wählt. H ist die physikalisch relevante Energie.

Man nennt so etwas einen trivialen Erhaltungssatz...

Für eine sorgfältige Diskussion siehe etwa
PJ Olver
Applications of Lie groups to differential equations
Springer, New York 1993
Section 4.3.


Nebenbei bemerkt, kann man die ART diskretisieren, indem man endlich
viele Punkte x_1,...x_n in R^3 nimmt, die Felder nur an diesen Punkten
betrachtet, und räumliche Ableitungen durch die Ableitung einer
geeigneten Interpolationsfunktion an diesen Punkten ersetzt.
Aus einem 4D Feld Phi(x,t) wird dann ein zeitabhängiger Vektor phi(t)
mit Komponenten phi_k(t)=Phi(x_k,t). Der Raum hat in dieser
Approximation keine kontinuierliche Struktur mehr, sondern besteht nur
noch aus endlich vielen Punkten. Die Raumzeit also aus endlich vielen
Geraden, die eine unzusammenhängende 1-dimensionale Mannigfaltigkeit
bilden. Im Sinn der Differentialgeometrie ist die Dimension die
Dimension des Tangentialraums. Und der hat in einer so diskretisierten
Raumzeit nur noch eine zeitliche Dimension, da räumlich keine
infinitesimalen Bewegungen mehr möglich sind.

Für quantitative Berechnungen braucht man natürlich numerisch
adäquate Diskretisierungen und benutzt z.B. adaptive Gitter.
Aber obige Primitivdiskretisierung würde (für N--> unendlich)
korrekte Approximationen liefern, wenn man die Metrik entsprechend
diskretisiert (etwa in harmonischen Koordinaten) und Finite Elemente
Verfahren für hyperbolische Differentialgleichungen
verwendet, um die kausale Struktur korrekt zu repräsentieren.





------------------------------------------
S4a. Sind alle dynamischen Systeme linear?
------------------------------------------

Auf den ersten Blick nicht, da es dynamische Systeme gibt,
die nicht linear sind.

Trotzdem gibt es für jedes nichtlineare dynamische System
dy/dt = F(y)
ein äquivalentes lineares System in einem viel größeren
Raum, nämlich im Raum aller Funktionen f(y).

Es ist nämlich
df(y)/dt = f'(y) dy/dt = f'(y) F(y)= Df(y)
mit dem linearen Differentialoperator
D = F(y)^T Nabla.
Dieses vergrößerte lineare System ist durchaus nützlich,
da man damit eine Spektralanalyse des nichtlinearen Systems
vornehmen kann, ähnlich wie in der Quantenmechanik.








------------------------
S5a. Was sind Anomalien?
------------------------

Eine Anomalie in der Quantenfeldtheorie ist, wenn das zur Definition
des Pfadintegrals nötige Maß notwendig weniger Symmetrien hat
als die Wirkung im Integranden.

Das äußert sich in zusätzlichen Schwinger-Termen in den
Kommutatorrelationen. (Daher hat das Kind seinen Namen - wenn
keine Anomalien da ist, sieht hier alles 'normal' aus!)

Diese werden übersichtlich durch kohomologische Begriffe klassifiziert.

Man braucht also Kenntnisse über Darstellungen von Lie-Algebren
und elementare Kohomologietheorie. Verschiedene Arbeiten von Jackiw
(siehe http://scholar.google.com) geben einen guten Überblick.







-------------------------
S6a. Gibt es Kugelblitze?
-------------------------

Vermutlich schon, wenn auch selten. Das Phänomen findet man
schön beschrieben in
http://www.gwup.org/skeptiker/archiv/2001/2/kugelblitze.html Die folgenden Referenzen (aus http://www.kfki.hu/fszemle/archivum/fsz0410/tard0410.html) habe ich zwar nicht gelesen, aber sollten die relevante Information enthalten. 1. J.D. BARRY: Ball Lightening and Bead Lightening: Extreme forms of Atmospheric Electricity Plenum Press, 1980 2. K. BERGER: Kugelblitz und Blitzforschung Naturwissenschaflen 60 (1973) 485 3. G. DIJKHUIS, J. PIJPELINK: Performance of high voltage tesl facility. Science of Ball Lightning First Intern. Symp. on Ball Lightning, Tokyo, 1988, Word Scientific Publ. 1989, p. 337 4. GY. EGELY: Hungarian Bull lightning Observations Hungarian Academy of Science, KFKI, 1987 5. M.S. HOWE: Theory of Vortex Sound Cambridge Univ. Press, 2003, p. 91 6. K. KAHLER: Die Elektrizität der Gewitter Sammlung Borntraege, Band 3 (1923) 7. P. KAPITZA Dokl. Acad. Nauk, USSR, 101 (1955) 245-248 8. H. KIKUCHI: Ball Lightning, Handbook of Atmospheric Electrodynamics, Vol. 1 ed. Volland, H., CRC Press, 1995, p. 167-187 9. P. KOLOC: The Plasma Configuration and Ball Lightning In: Science of Ball Lightning, ed. Y. Ohtsuki, Japan, Word Scientific Publ., 1989, p. 289-309 10. V. KOPIEV: Theory of Vortex Ring Noise In: Advances in Aeroacustics, ed. J. Anthoine, C. Schram, Karman Institute for Fluid Dynamics, 2001, p. 10, Fig. 8 11. J. LINGEMANN, G. PREMINGER: New Developments in the Management of Urolithiasis Igaku-Shoin Press, 1996, p. 29 12. L. LOEB: Static electrification Berlin, Springer, 1958 13. R. MUHLEISEN, H. FISCHER: Elektrische Aufladung von Hubschraubern Bonn, 1978, Forschungsbericht aus der Wehrtechnik: BMV g-FBWT 78-7 14. K. NICKEL: A fluid dynamical model for ball lightning ed. Y. Ohtsuki, 1988, p. 156; The Lifetime of Hill's Vortex Word Scientific Publ. Press, 1988, p. 177 15. Proc. of First Intern. Symp. on Ball Lightning, ed. Y. Ohtsuki, in: Science of Ball Lightning, Tokyo, 1988, Word Scientific Publ., 1989 16. Proc. of 5th Intern, Symp. on Ball Lightning, 1997, ed. Y. Ohtsuki, Tsugawa, Japan, 1997 17. V. RAKOV, M. UMANN: Lightning Physics and Effects Cambridge Univ. Press, 2003 18. M. SANDULOVICIU, ET AL.: Ball lightning like structures formed under controllable laboratory conditions Proc. of 5th Intern. Symp. on Ball Lightning, Tsugawa, Japan, (1997) p. 170-75 19. S. SINGER: The Nature of Ball Lightning Plenum Press, New York, 1971 20. M. STEINHOFF: Ball Lightning. An unsolved problem in atmospheric physics Kluwer Acad. Plenum Publ., 1999 21. S. STEFANOV: On the Energy of Ball Lightning Proc. of 5th Intern. Symp. on Ball Lightning, Tsugawa, Japan, 1997, p. 61-62 22. S. STEFANOV, ET AL.: Electric Machine in Ball Lightning Proc. of 5th Symp. on Ball Lightning, Tsugawa, Japan, 1997, p. 183-187 23. K. SUSLICK: Die chemischen Wirkungen von Ultraschall Spektrum der Wissenschaft, Apr. 1989, p. 60-66. 24. A. VLASOV: A ball lightning is u natural nuclear reactor? Proc. of 5th Symp. on Ball Lightning, 1997, p. 75-79 25. H. VOLLAND (ed.): Handbook of Atmospheric Electrodynamics, Vol. l. CRC Press, London, 1995 ---------------------------------------- S7a. Erklärt die Wissenschaft die Welt? ---------------------------------------- Man liest und hört immer mal wieder Aussagen von der Art ''Physik erklärt die Welt nicht, sondern beschreibt sie nur'' oder ''In der Physik haben metaphysische Aussagen keinen Platz''. Aber Wissenschaft ist auch Welterklärung, und ihre Grundprinzipien sind metaphysisch. Der Glaube an die Allgemeingültigkeit naturwissenschaftlicher Gesetze ist etwas zutiefst Religiöses, da wir aus dem stichprobenhaften Nachprüfen einiger Beobachtungen auf deren Gültigkeit in der unbeobachtbaren Zukunft schließen. http://de.wikipedia.org/wiki/Religion :
''Als Religion bezeichnet man eine Vielzahl ganz unterschiedlicher
kultureller Phänomene, die menschliches Verhalten, Denkweisen und
Wertvorstellungen normativ beeinflussen. [...] Eine einheitliche
Definition des Begriffs existiert nicht. [...] Allerdings erfasst der
westliche Ansatz einer Definition mit Hilfe des Begriffs 'Glaube'
nicht alle Religionen, da dieser Terminus in einigen Religionen nicht
oder kaum existiert und damit nicht das eigentliche Merkmal dieser
Religionen sein kann. [...] In der Religionswissenschaft beispielsweise
existieren viele unterschiedliche Definitionen nebeneinander.''

http://de.wikipedia.org/wiki/Religion :
Dabei steht religio als gewissenhafte Beachtung überlieferter Regeln
im Gegensatz zu superstitio als freier, ekstatischer Spiritualität.

Also: Sich Halten an die Regeln gewissenhafter wissenschaftlicher
Methodik (eine für Wissenschaftler ziemlich unabdingbare persönliche
Wertüberzeugung) ist Religion und nicht Aberglaube.

Einstein classified scientists very well during the celebration of
Max Planck's 60th birthday in 1918. In the temple of science, he said,
are three kinds of people. Many take to science out of a joyful sense
of their superior intellectual power. For them research is a kind of
sport that satisfies personal ambition. A second class of researchers
engage in science to achieve purely utilitarian ends. But of the
third: If "the angel of the Lord were to come and drive all the people
belonging to these two categories out of the temple, a few people
would be left, including Planck, and that is why we love him."
http://www.americanscientist.org/template/AssetDetail/assetid/28525


Das Eingeben von
erklärt beschreibt Physik
und
erklärt-nicht beschreibt Physik
in Google ist auch instruktiv. Z.B. findet man in
http://de.wikipedia.org/wiki/Theoretische_Physik :
''So erklärt die Quantenmechanik, warum die Materie fest ist und
warum die chemischen Elemente sich so und nicht anders verhalten,
sie ist Grundlage der Halbleiterphysik und damit der gesamten
modernen Elektronik, und sie hat unser Weltbild grundlegend
revolutioniert''

Roy J. Glauber, Physik-Nobelpreisträger 2005, schreibt in seiner
Nobel Lecture u.a.:
''In those developments de Broglie, Heisenberg, Schrödinger and others
accomplished literal miracles in explaining the structure of atoms.''
http://nobelprize.org/nobel_prizes/physics/laureates/2005/glauber-lecture.pdf

Astronomen versuchen, Erklärungen für ihre beobachteten Daten zu
finden, z.B.
http://pds-rings.seti.org/reference/abstracts/porco1991_01.html
An explanation for Neptune's ring arcs

http://eric.ed.gov/ERICDocs/data/ericdocs2/content_storage_01/0000000b/80/25/ca/7e.pdf
''Science educators agree with philosophers that explanation is the
very purpose of science itself.''

http://de.wikipedia.org/wiki/Naturwissenschaften ''Naturwissenschaften sind Wissenschaften, die sich mit der unbelebten und belebten Natur befassen, diese zu beschreiben und zu erklären versuchen. [...] Die Ansicht, was materiell (physisch) zu erklären sei, und was metaphysisch, also jenseits der materiellen Phänomene liegend und damit der materiellen Erklärung nicht zugänglich sei, unterliegt einem beständigen Wandel.'' http://en.wikipedia.org/wiki/Scientific_method ''Scientific researchers propose specific hypotheses as explanations of natural phenomena, and design experimental studies that test these predictions for accuracy.'' hhttp://en.wikipedia.org/wiki/Explanandum ''the event under discussion is explained by subsuming it under general laws, i.e., by showing that it occurred in accordance with those laws, by virtue of the realization of certain specified antecedent conditions'' http://plato.stanford.edu/entries/scientific-explanation ''Issues concerning scientific explanation have been a focus of philosophical attention from Pre-Socratic times through the modern period.'' Der Erfolg der Naturwissenschaft besteht darin, durch klare Beschreibungen zu erklären. Dass man mit wenigen fundamentalen Naturgesetzen den Mikrokosmos bis zum ganzen Universum recht ordentlich beschreiben kann, ist keineswegs selbstverständlich. Das empfinden wir als Erklärung. Allerdings: Auch die beste Erklärung endet bei Unerklärtem. Trotzdem _ist_ es eine Erklärung. Jede Art von Erklärung ist von dieser Art, eine konsistente Beschreibung von etwas durch Dinge, die wir als grundlegend und keiner weiteren Erklärung bedürftig voraussetzen. http://www.dwds.de/?woerterbuch=1&qu=erkl%C3%A4ren ''etw. (schwer zu Verstehendes) durch Worte klar, deutlich machen: eine Abbildung, die schwierige Textstelle e.; der Lehrer erklärt den Schülern eine Aufgabe; jmdm. etw. genau, gründlich, ausführlich, umständlich, kurz, wissenschaftlich, allgemeinverständlich, stundenlang e.; das erkläre ich dir am besten durch ein Beispiel, an einem Bild, mit Zahlen; erkläre mir doch einmal den Unterschied zwischen ...; wie erklärst du dir das?; eine erklärende Beschreibung; erklärende Worte; sich e. seine Erklärung finden: dieser Kreislauf erklärt sich schwer, ganz einfach, von selbst; das Defizit erklärt sich aus den hohen Unkosten'' ---------------------------- S7b. Das Uhrmacheruniversum ---------------------------- Die moderne, mit allen Aussagen der Naturwissenschaften verträgliche Version des alten teleologischen Arguments von Robert Hooke und Voltaire http://de.wikipedia.org/wiki/Uhrmacher-Analogie für die Auffassung, dass das Universum durch das Wirken intelligenten Bewusstseins entstanden sein muss, lautet so: Das durch Quantenmechanik gesteuerte Universum funktioniert nach den darin implementierten Naturgesetzen in so großem Maßstab, dass Konstruktionen von Menschenhand neben der ''Allmacht und Vollkommenheit des großen Schöpfers'' verblassen müssen. Daher muss das Universum wie eine Uhr von einem intelligenten Wesen geplant worden sein. -------------------- S7c. Gott und Physik -------------------- Gott - die große Unbekannte http://www.mat.univie.ac.at/~neum/sciandf/ger/unbek.html Mathematik, Physik und Ewigkeit (mit einem Augenzwinkern betrachtet) http://www.mat.univie.ac.at/~neum/sciandf/ger/neumann.pdf Wahrheit und Vertrauen - ein Mathematiker redet von Gott http://www.mat.univie.ac.at/~neum/sciandf/ger/wuv.html Was ist Wahrheit? Gedanken eines Mathematikers http://www.mat.univie.ac.at/~neum/sciandf/ger/wiw.html Wissenschaft und Dogma http://www.mat.univie.ac.at/~neum/sciandf/ger/dogma.html How to Create a Universe - Instructions for an Apprentice God. A fantasy to be read at leisure time http://www.mat.univie.ac.at/~neum/other/turing.txt Eine nicht gehaltene Vorlesung http://www.mat.univie.ac.at/~neum/sciandf/ger/vorl.html --------------------------------------------------------- S7d. Warum kann die Welt mathematisch beschrieben werden? --------------------------------------------------------- Für jeden, der sich auskennt und das Wundern noch nicht verlernt hat, ist es verwunderlich, dass unsere Welt mit so großem Erfolg mathematisch beschrieben werden kann. Denn die Mathematik hat ein Eigenleben, das nicht durch die Beschreibung der Natur präjudiziert ist. Man kann sie nicht willkürlich basteln, sondern muss sich an die ihr innewohnenden Gesetzmäßigkeiten halten. Wer immer die Gesetze der klassischen Logik anerkennt, findet auf ihrer Basis im wesentlichen (d.h. bis auf Bezeichnungsweise, Betonung und genauen Formulierung) dieselben zentralen Konzepte und Sätze. Man kann z.B. keine Mathematik machen, in dem die Klassifikation der semisimplen Liegruppen anders aussieht als in der unseren. Diese Klassifikation wurde gefunden lange bevor sie sich als extrem nützlich in der Quantenmechanik herausstellte, ja sogar lange bevor es Quantenmechanik überhaupt gab. Dass die durch die Logik schon im Wesentlichen determinierte Mathematik auf die (von der Logik unabhängige) Natur anwendbar ist und soviel erklärt, ist also überhaupt nicht selbstverständlich. Warum sich die Natur und das gesamte Universum, soweit wir es kennen, so verhält, dass sie mit mathematischen Methoden exzellent beschrieben werden kann, ist eine Frage, die den Rahmen der Naturwissenschaften sprengt. Die naheliegende Antwort ist: Weil sie von jemandem entworfen wurde, der viel von Mathematik versteht. Jedenfalls würden wir so argumentieren, wenn es sich um ein kleineres Objekt als das Universum handeln würde. Für mich ist dies ein klarer Hinweis auf die Existenz und die Macht Gottes. Für eine ausführlichere Diskussion siehe die beiden Klassiker E.P. Wigner The unreasonable effectiveness of mathematics in the natural sciences, Comm. Pure Appl. Math. 13 (1960), 1-14. R. W. Hamming The unreasonable effectiveness of mathematics, Amer. Math. Monthly 87 (1980), 81-90. Und viele weitere Essays, die unter dem Stichwort „unreasonable effectiveness" in scholar.google.com gefunden werden können. ----------------------- S8a. Andere Physik-FAQs ----------------------- http://theory.gsi.de/~vanhees/faq/index.html Physik und das Drumherum (Physics FAQ in German) http://theory.gsi.de/~vanhees/faq/anettes-faq/anettes-faq.txt Anettes FAQ (Tensorrechnung und Allgemeine Relativitätstheorie) http://www.mat.univie.ac.at/~neum/physics-faq.txt A theoretical physics FAQ http://math.ucr.edu/home/baez/physics/ Usenet Physics FAQ http://www.physik.tu-dresden.de/studdocs/skripte.htm Vorlesungsskripte und Lehrmaterial Physik http://www.physik.tu-berlin.de/~erat/lect.html Skripte im WWW (Mathematik, Physik) http://www.freidok.uni-freiburg.de/volltexte/82/ Digitale Reproduktion des Lehrbuchs Honerkamp/Römer, Klassische theoretische Physik, 3. Aufl., 1993. --------- S9a. Dank --------- Ich danke Hendrik van Hees für eine ausführliche Diskussion über die Thermische Interpretation im Frühjahr 2004, und Lothar Wiese für die Erstellung einer HTML-Version auf http://www.mat.univie.ac.at/~neum/physik-faq.html Außerdem danke ich denen, die mich durch ihre mehr oder weniger herausfordernden Beiträge in der Newsgruppe de.sci.physik dazu angeregt haben, die obigen Antworten niederzuschreiben. Schließlich danke ich Gott für sein wundervolles, interessantes Universum und für die Einsicht, die er mir geschenkt hat, dieses Wunder zu verstehen. ---------------------------------- S10. Die Thermische Interpretation ---------------------------------- Hier stelle ich meine eigene Interpretation der Quantenmechanik vor, die Thermische Interpretation. Die traditionelle Interpretation des orthodoxen Formalismus wird geändert, indem statt der Tradition, die nur von_Neumann-Messungen von Eigenwerten als objektiv wertet, davon ausgegangen wird, dass alle Rohmessungen Messungen von Erwartungswerten gewisser Operatoren (oder daraus berechenbarer abgeleiteter Konzepte) sind, und dass die quantenmechanischen Erwartungswerte nichts direkt mit statistischen Erwartungswerten (im Sinn von Mittelwerten von Messreihen) zu tun haben. Die so entstehende Interpretation.erlaubt, alle Experimente konsistent zu beschreiben, ohne die Quantenmechanik zu ändern. ändern braucht man nur die Interpretation des orthodoxen Kalküls, im Einklang mit der Tatsache, dass Thermodynamik, Hydrodynamik, und Kinetik - also die Theorien, die unsere Messgeräte beschreiben - im Rahmen der statistischen Mechanik alle als Theorien von Erwartungswerten erscheinen. Die beobachteten Quanten-Wahrscheinlichkeiten erklären sich - genau wie die im Lorentzattraktor beobachtbaren - durch sensitive Abhängigkeit der gemessenen Erwartungswerte von den präparierten Erwartungswerten. Zufälliges Verhalten stellt sich genau dann ein, wenn diese Sensitivität gegeben ist. (Das lässt sich leicht mit semiklassischen Rechnungen zum Thema Quantenchaos bestätigen.) Details werden in den weiter unten folgenden FAQ-Beiträgen gegeben. Ein kleiner Teil der Thermischen Interpretation ist schon in der Arbeit quant-ph/0303047 = Int. J. Mod. Phys. B 17 (2003), 2937-2980. http://www.mat.univie.ac.at/~neum/papers.html#ensembles veröffentlicht, auf die ich mich in diesem FAQ mit [EECQ] beziehe. Diese Arbeit zeigt, wie sich die Thermische Interpretation aus meiner Analyse der quantenlogischen Grundlagen ergeben hat, sagt aber nichts über die Konsequenzen der neuen Interpretation und die Analyse echter Messprozesse. Eine Publikation dazu ist in Vorbereitung. In der Arbeit gehe ich aber zweigleisig vor und beschreibe zugleich die traditionelle statistische Interpretation. In dem meiner Darstellung zugrundegelegten abstrakten Ensemblebegriff ist eine probabilistische Interpretation möglich, aber nicht
notwendig, und bei Einzelsystemen auch nicht sinnvoll.

Wenn man Section 6 (über Wahrscheinlichkeit) aus [EECQ] entfernt,
ist der Rest immer noch
1. vollkommen verständlich,
2. hundertprozentig mit der Praxis der Quantenmechanik kompatibel,
3. physikalisch interpretierbar,
obwohl das Wort 'Wahrscheinlichkeit' kein einziges Mal mehr
in den Mund genommen wird.

Die ''squared probability amplitude''-Formel (24),
die die Basis der traditionellen Interpretation der
Quantenmechanik ist, ist bei mir nur eine Randbemerkung in
dieser Section 6, und damit völlig unerheblich für die
Interpretation. Ich habe diesen Abschnitt nur deshalb eingefügt,
um zu zeigen, dass meine Axiome vollständig genug sind,
um den traditionellen quantenmechanischen Wahrscheinlichkeitsbegriff
bekommen zu können (falls man will).

Dadurch, dass man gewöhnlich diese Formel an den Anfang stellt,
schafft man sich erst die ganzen Interpretationsprobleme.

Zudem verdirbt man sich so den engen Zusammenhang zwischen
klassischer Mechanik und Quantenmechanik, und bekommt ihn erst
wieder, wenn man (bei den meisten Studenten erst mindestens
ein Jahr später) in der statistischen Mechanik die Dichtematrix
kennenlernt.

Bis dahin ist aber schon soviel Porzellan kaputtgeschlagen
worden, dass im Verständnis der Quantenmechanik ein heilloses Chaos
herrscht, das dann kaum mehr zu heilen ist...


Einiges über die Thermische Interpretation habe ich professionell
aufgeschrieben; siehe dazu
A. Neumaier,
Optical models for quantum mechanics,
Slides of a lecture given on February 16, 2010 at the
Institute for Theoretical Physics, University of Giessen,
http://www.mat.univie.ac.at/~neum/ms/optslides.pdf
und Kapitel 7 meines Online-Buchs
Arnold Neumaier and Dennis Westra,
Classical and Quantum Mechanics via Lie algebras,
2008. (arXiv:0810.1019)


------------------------------------
S11. Wie liest man [EECQ] am besten?
------------------------------------

Beim ersten Lesen kann man alle Beweise einfach übergehen
(außer man will einen bestimmten Punkt genau verstehen);
außerdem fast alle Formeln, die man nicht in einer Minute
Überlegung verstehen kann. (Nur die Formeln in Definition 4.1,
sowie (33)-(34) sind unverzichtbar zum Verständnis.)

Nummerierte Aussagen geben das formale Gerüst der Theorie wieder;
dabei sind Theoreme eher wichtig, Propositionen eher technisch.

Es ist genug normaler Text dazwischen, der einen roten Faden
liefert.

Beim zweiten Lesen schaut man sich das genauer an, was man
besser verstehen will, und geht bei Verweisen oder wenn
Symbole oder Begriffe unbekannt sind, soweit zurück,
bis man die entsprechenden Erklärungen findet.



------------------------------------------------------------
S12. Kernaussagen der Thermischen Interpretation
------------------------------------------------------------

Die Quantenmechanik beschreibt das Universum als Ganzes,
und damit insbesondere alles, was man darin reproduzierbar
messen kann - inklusive Einzelsysteme, Ensembles im
statistischen Sinn, Detektoren und Physiker.

Das Universum als Ganzes verhält sich deterministisch,
und lässt sich mit einer klassischen Hamiltonschen,
durch eine Hamiltonfunktion und eine Poissonklammer
definierten Dynamik beschreiben. Die klassischen Größen
in dieser Dynamik sind die traditionell als Erwartungswerte
bezeichneten Felder und Korrelationsfunktionen.

Das Universum hat einen klassischen Zustandsraum, dessen reine
Zustände alle Dichtematrizen der QM sind. (Die traditionelle
Interpretation hat dagegen einen Quantenzustandsraum,
dessen reine Zustände nur die Rang 1 Dichtematrizen sind.
Diese Einschränkung verursacht die traditionellen
Interpretationsprobleme.)


Alle Eigenschaften physikalischer Systeme im Universum werden
innerhalb eines einzigen mathematischen Modells des Universums
und seiner Evolution hergeleitet. Insbesondere ist der Zustand
jedes physikalischen Systems durch den Zustand des Universums
vollständig festgelegt.

Die Dynamik eines solchen Systems ergibt sich durch Projektion
der Dynamik des Universums auf die Algebra der Größen des
Systems und kann in der Markovnäherung durch eine dissipative
Differentialgleichung in Lindblad-Form beschrieben werden.
Ist die Dissipation vernachlässigbar, so erhält man die
traditionelle von-Neumann-Gleichung für die Dichtematrix
des Systems.


Ein wichtiges Merkmal der Thermischen Interpretation
ist die konsequente Berücksichtigung der Forderung, dass man
(außer in motivierenden Bemerkungen) nur über vorher
mathematisch präzise definierte Objekte reden darf.

Dies gewährleistet ein logisch konsistentes Modell, innerhalb
dessen die traditionellen Bestandteile unseren Universums samt
ihrer mathematischen Beschreibung definiert und analysiert werden.


Insbesondere wird der Messprozess modellimmanent durch
Wechselwirkung eines Quantensystems mit einem Detektor,
beide als Teilsysteme des Universums verstanden, modelliert.
Damit wird die der Kopenhagen-Interpretation eigene
Teilung der Welt in Quantensysteme und klassische Messgeräte
überwunden. Was eine Messung darstellt, wird präzisiert.


Der Zufall und die Quantensprünge ergeben sich als
ausschließliche Folge der in einer Beschreibung von
Quantensystem und Detektor allein auf Grund der
Markov-Näherung nicht vollständig berücksichtigten
Wechselwirkung mit dem Rest des Universums.

Insbesondere stehen die beobachtbaren Verteilungen der
messbaren Zufallsvariablen im Einklang mit der
statistischen Interpretation der Quantenmechanik.

Die Bornsche Regel über Wahrscheinlichkeit als Quadrat
des Absolutbetrags einer Amplitude ergibt sich in der
Thermischen Interpretation direkt aus der
Projektion des Vielteilchensystems auf die traditionelle
reduzierten Beschreibung durch klassische Zeigervariable
des Detektors plus Quantenzustand des gemessenen Quantensystems.


Die Interpretation ist also hundertprozentig mit der
Praxis der Quantenmechanik kompatibel, leitet aber die
Wahrscheinlichkeitsinterpretation aus einfachen
deterministischen Grundannahmen ab, statt sie als
unerklärliches (und philosophisch problematisches)
Postulat zu den Geheimnissen unseres Universums zu zählen.



---------------------------------------------------------------
S13. Motivation für die Thermische Interpretation
---------------------------------------------------------------

Alle quantitativen Beobachtungen in der Physik beruhen auf dem
Ablesen makroskopischer Objekte - Zeiger, Filme, Zähler, usw.,
deren Physik durch die Thermodynamik beschrieben wird. Daraus
erschließen wir mit Hilfe der Theorien der Physik und
numerischen Rechnungen Information über die Mikrosysteme,
die uns interessieren.

Die 'rohen' Messgrößen sind also immer thermodynamische Größen;
nach den Aussagen der statistischen Physik also Funktionen von
Erwartungswerten im entsprechenden großkanonischen Ensemble.
Auch bei einer einzelnen Rohmessung hat man da ein Ensemble,
aber kein echtes statistisches, sondern ein fiktives, virtuelles.
Dieses wurde als Fiktion von Gibbs eingeführt, damit man den
formalen Apparat der statistischen Mechanik zur Herleitung der
thermodynamischen Gesetze benutzen darf.

In meiner Thermischen Interpretation nehme ich diese
Tatsache zum Anlass, jedem Einzelsystem (und nicht nur
den thermodynamischen) ein virtuelles Ensemble zuzuordnen, das man
natürlich nicht mehr statistisch deuten darf, ebenso wenig wie
man eine Temperaturmessung an einer Tasse Tee x-mal wiederholen muss,
bevor man von einem Wert sprechen darf.

Das Ablesen eines Zeigers oder das Auswerten eines Bildes, um ein
Messergebnis daraus zu gewinnen.wird ebenso in der Regel nur einmal
vorgenommen, evtl. zur Kontrolle ein zweites Mal. Stellt sich das
Ablesen bei der Kontrolle als unzuverlässig heraus, so wird
die Einzelmessung zwar notiert, gilt aber nicht als reproduzierbar.
Man macht dann (und nur dann) viele Einzelbeobachtungen und fasst
diese dann statistisch zu einer Gesamtbeobachtung zusammen,
etwa in Form von Bestwert (Mittel) und Fehler (Standardabweichung).

Jede Einzelbeobachtung ist also ein thermodynamischer Erwartungswert
auf Grund eines virtuellen, nichtstatistischen Ensembles;
die Gesamtbeobachtung dagegen ist ein statistischer Erwartungswert
auf Grund eines realen Ensembles aus vielen wiederholten Messungen.

Beide Arten von Ensembles/Erwartungswerten genügen denselben
mathematischen Gesetzen, haben aber eine völlig unterschiedliche
Bedeutung.

Daher definiere ich ein abstraktes Ensemble lieber axiomatisch
durch die relevanten Eigenschaften, siehe [EECQ] für Details.
Ebenso wie die Definition eines Vektorraums (der ja ursprünglich
auch durch 3-dimensionale Vektoren motiviert war und heute für
allerlei virtuelle Vektoren - Zahlenlisten, Matrizen, Funktionen,...
benutzt wird), erlaubt das die saubere Trennung formaler
Eingenschaften und ihrer inhaltlichen Bedeutung.


Die grundlegende Annahme der Thermischen Interpretation
ist nun die, dass die objektiven Aspekte des Universums durch
ein Ensemble in diesem abstrakten Sinn gegeben ist, und alles
Messbare durch Erwartungswerte in diesem Universalensemble oder
Funktionen von solchen Erwartungswerten.

Diese Annahme erfasst alles reproduzierbar Messbare auf
einheitliche Weise: Naturkonstanten, Streuquerschnitte,
Zerfallswahrscheinlichkeiten, Reaktionsraten, Transportkoeffizienten,
thermodynamische Größen - kurz alles, was Experimentatoren messen
und steuern können oder wollen!

Ausgehend von dieser grundlegenden Annahme findet man objektive
Grundlagen für einen konsistenten Aufbau der Quantenmechanik, der
1. logisch einwandfrei ist,
2. eine klassische deterministische (symplektische oder Poisson-)
Dynamik hat,
3. das Auftreten des klassischen und quantenmechanischen Zufalls
in Übereinstimmung mit den Experimenten erklärt,
4. Vollständig ist in dem Sinn, dass die Kenntnis des Zustandes
des Universums (d.h. des Universalensembles) eine Kenntnis aller
denkbaren Messwerte impliziert.

Die Thermische Interpretation postuliert also eine
deterministische Dynamik für das Universum als Ganzes, und
deduziert daraus eine approximative stochastische Dynamik
für jedes Teilsystem.

Im Sinne der traditionellen Nomenklatur handelt es sich also
um eine Theorie verborgener Variablen, in denen die klassischen
Variablen die Gesamtheit der quantenmechanischen Erwartungswerte
(oder gleichwertig alle Korrelationsfunktionen der BBGKY-Hierarchie)
sind.



-------------------------------------------------------------
S14. Die einzigen Observablen der Physik sind Erwartungswerte
-------------------------------------------------------------

Die Thermische Interpretation sagt, dass
die einzigen Observablen der Physik Erwartungswerte sind,
die sich zeitlich und räumlich genügend langsam ändern.

Dazu gehören

1. Auf der globalem Ebene: die Materialeigenschaften von Stoffen,
inklusive den Massen von Atomen, Protonen, Elektronen sowie
Massen und Lebensdauern von instabilen Teilchen, die Spektren
von Atomen und Molekülen, etc..

2. Auf der Ebene des lokalen Gleichgewichts: Ströme, Massendichten
verschiedener Materialien, Druck, Temperatur, mechanische und
elektromagnetische Spannungsfelder, etc., die approximativ den
hydrodynamischen Gleichungen (Navier-Stokes) genügen und alles
Erwartungswerte von mikroskopischen Operatoren oder daraus mittels
Thermodynamik berechneter Größen sind.

3. Auf der Ebene der kinetischen Beschreibung (mikrolokales
Gleichgewicht): Phasenraumdichten (Wignerfunktionen), die approximativ
den Boltzmann-Gleichungen oder Vlasov-Gleichungen genügen und alles
Erwartungswerte von Einteilchenoperatoren sind.

4. Auf der molekularen Ebene: Dichtefunktionale, die approximativ den
Hartree-Fock-Gleichungen oder CI-Gleichungen, etc. genügen und
ebenfalls Erwartungswerte von Einteilchenoperatoren sind.

5. Auf noch tieferer Ebene: effektive Feldgleichungen für Atomkerne,
die approximativ den Hartree-Fock-Bogoliubov--Gleichungen genügen
und ebenfalls Erwartungswerte von Einteilchenoperatoren sind.

6. Außerdem: räumliche und zeitliche Korrelationsfunktionen,
die das lineare Antwortverhalten bei Anregungen beschreiben;
auch diese sind Erwartungswerte, diesmal aber von
Zweiteilchenoperatoren.

Nirgends beobachtet man mehr als Erwartungswerte.

Auch die Interferenzmuster auf der Photoplatte sind Erwartungswerte
von Teilchendichtefeldern, und die Klicks im Geigerzähler
sind Erwartungswerte von Druckfeldern, die unser Ohr
(oder ein entsprechender Detektor) wahrnimmt, etc.

Auch die Klicks im Geigerzähler sind Erwartungswerte von Druckfeldern,
die unser Ohr (oder ein entsprechender Detektor) wahrnimmt, etc.
Die Klicks als irreduzible Messung eines diskreten Zustands eines
einzelnen Mikrosystems aufzufassen ist zwar alte Tradition, aber
höchstens indirekt (über selbst schon interpretationsbedürftige
Theorie) gerechtfertigt, und wird in der Thermischen
Interpretation einfach fallengelassen. Sobald man das tut,
verschwinden die Interpretationsprobleme der Quantenmechanik.


Dass die statistische Mechanik Erwartungswerte statistisch
interpretiert, ist rein historisch bedingt, da zur Zeit von Gibbs
noch keine formal in der Maßtheorie begründete
Wahrscheinlichkeitstheorie existierte. Aber Gibbs, der Begründer
der statistischen Thermodynamik, war vorsichtiger als die meisten
seiner Epigonen. In seinem Buch
W. Gibbs,
Elementary Principles in Statistical Mechanics,
Yale Univ. Press, 1902
führt er Ensembles als fiktive, gedachte identische Kopien des
beobachteten Einzelsystems im thermodynamischen Gleichgewicht ein,
um eine Rechtfertigung für seinen 'Missbrauch' der
Wahrscheinlichkeitsrechnung für die Modellierung eines Einzelsystems
zu rechtfertigen:
''Let us imagine a great number of independent systems, identical
in nature, but differing in phase, that is, in their condition with
respect to configuration and velocity.'' (S.5)
''The application of this principle is not limited to cases in
which there is a formal and explicit reference to an ensemble of
systems. Yet the conception of such an ensemble may serve to give
precision to notions of probability.'' (S.17)
Es ist offensichtlich, dass Gibbs anschließend davon ausgeht,
dass seine Theorie für jede einzelne Messung an einem Material im
thermodynamischen Gleichgewicht gilt. Und die Erfahrung bestätigt
ihn darin.



------------------------------------
S15. Zwei Arten von Erwartungswerten
------------------------------------

Ein häufiges Missverständnis beruht auf der mehrfachen Bedeutung
des Begriffs 'Erwartungswert'. Mathematisch gesehen ist der
Erwartungswert das Bild eines linearen Operators unter einer
monotonen linearen Abbildung, nicht mehr und nicht weniger.
Welche Interpretation wir dieser monotonen linearen Abbildung geben,
steht uns (wie bei jedem mathematischen Begriff) frei.

Die traditionelle Interpretation bezieht den Begriff auf die Tatsache,
dass der Mittelwert einer Reihe von Messwerten in gleichartigen
(''identisch präparierten'') Szenarien omega_k (k=1:N)
<f>_empirisch = 1/N sum_k f(omega_k) (*)
die Eigenschaften des mathematischen Erwartungswert hat (daher auch der
Name).

Schon die Identifikation dieses empirischen Begriffs
mit dem mathematischen macht aber philosophische Schwierigkeiten,
etwa bei einer Gauss-verteilten Zufallsvariable, weil man nicht genau
sagen kann, wie die formale Variable zu den Messungen steht.
Man stammelt dann etwas von Wahrscheinlichkeit als Grenzwert der
relativen Häufigkeit (aber nur im Grenzfall unendlich vieler
Messungen, und nur mit Wahrscheinlichkeit 1) und hat Schwierigkeiten,
einem endlichen Ensemble eine stetige Verteilung zuzuordnen.
Oder man redet von einem gedachten Ensemble aller Möglichkeiten,
die die Messung hätte haben können, um dies zu rationalisieren.
Beides sind Zeichen, dass da etwas faul ist. Schon klassisch!
Ebenso wie zu den Grundlagen der Quantenmechanik gibt es daher
zu den philosophischen Grundlagen der Wahrscheinlichkeitstheorie
eine ausgedehnte, kontroverse, Literatur!


Sieht man sich dann ein klassisches chaotisches System an, so
hat man einerseits eine deterministische Dynamik, andererseits
ein in der Praxis stochastisches Verhalten. Dieses praktische
stochastische Verhalten äußert sich darin, dass man mit (*)
zu zufällig genommenen Zeiten (die als Szenarien omega_k
fungieren) einigermaßen robust reproduzierbare Ergebnisse bekommt,
wenn f eine genügend schöne Observable ist. (Und nur dann;
man kann beliebig hässliche Observable bauen...)
Die so erhaltenen statistischen Mittelwerte sind typischerweise
bis auf Fehler der Ordnung O(N^{-1/2} gleich dem durch
<f> = integral dmu f
definierten Erwartungswert, wo mu das invariante Maß des zum
deterministischen Orbit gehörigen Attraktors ist. Dieses Maß
und damit alle <f> sind ebenso objektiv dem System zugeordnet,
wie die deterministische Trajektorie selbst. Die durch dieses
invariante Maß für das klassische Einzelsystem objektiv
festgelegten Erwartungswerte <f> genügen aber genau denselben
Regeln wie der empirische Erwartungswert, in dem man Mittelwerte
aus einem Ensemble von zufälligen zeitliche Stichproben bildet.
Die <f> sind also genauso objektive Observable des Einzelsystems
wie die f selbst, nur dass sie zeitlich konstant sind.

Messen lassen sie sich aber nur approximativ, genauso wie die
Trajektorie selbst. Aber im Unterschied zu den Werten der Trajektorie
sind die <f> reproduzierbar approximativ messbar!

Dasselbe lässt sich (mit einigen Modifikationen, die ich hier nicht
näher erörtern will) auch im instationären Fall machen,
und man bekommt zeitabhängige objektive Erwartungswerte.
Natürlich sind die so 'renormierten' Observablen <x(t)> nicht
dasselbe wie die 'nackten' Observablen x(t), aber beide sind
objektive Eigenschaften des Systems; nur die Vorschriften zur
Messung sind verschieden, da es sich um verschiedene Observable
handelt.


Sieht man sich nun ein klassisches deterministisches, aber
turbulentes System an, etwa hydrodynamische Gleichungen
im turbulenten Bereich, so ist es gar nicht mehr möglich,
etwa die Geschwindigkeit v(x,t) zu messen, da die hohen Frequenzen
zwangsläufig unaufgelöst bleiben. Und selbst
bei beliebig hoher aber fixer Auflösung ist der Einfluss der
noch höheren Frequenzen signifikant. Tatsächlich
ist das Feld, das ein Ingenieur in einer Einzelmessung
im Windkanal misst, also stets eine Approximation an den
Erwartungswert <v(x,t)> einer Zufallsvariable v(x,t),
die den Feldgleichungen genügt. Die in einer Einzelmessung
anfallenden Messwerte sind also Approximationen des
'renormierten' <v(x,t)>, und nicht die 'nackten',
unbeobachtbaren v(x,t).

Da <v(x,t)> noch sehr irregulär ist und sich kaum vorhersagen
lässt, ist ein Ingenieur statt dessen an einer mittleren, aber
einigermaßen vorhersagbaren grobkörnigen Geschwindigkeit interessiert,
die er aus dem Mitteln vieler Momentanfelder <v(x,t)> bekommt:
vbar(x,t) := <<v(x,t)>>_emp = 1/N sum_k <v(x+z_k,t+s_k)>
(mit kleinen Verschiebungen z_k,s_k). Diese mittlere
Geschwindigkeit lässt sich einigermaßen gut durch
Simulationen vorhersagen und ist deshalb praktisch relevant.
Ebenso ist für die Praxis wichtig, Information über die Abweichungen
d(x,t) = <v(x,t)>-vbar(x,t)
zu bekommen. Die ist in den Korrelationen
<d(x,t)d(x',t')>_emp = 1/N sum_k d(x+z_k,t+s_k)d(x'+z_k,t'+s_k)
und evtl. höheren Momenten enthalten. Man sieht also, dass es
_zwei_ stochastische Ebenen gibt, die eine empirische, die
mit den Messungen normale Statistik macht und der traditionellen
Interpretation entspricht, und eine darunterliegende objektive,
in der der Erwartungswert nicht mehr die Bedeutung eines statistischen
Mittelwerts hat, sondern den eines rein mathematisch definierten
Maßes, das aus unmessbaren, beliebig hochfrequente Anteile enthaltenden
'nackten' Observablen v(x,t) beobachtbare 'renormierte' Variablen
<v(x,t)> macht. Das ist genauer beschrieben in:
H Grabert,
Projection Operator Techniques in Nonequilibrium
Statistical Mechanics,
Springer Tracts in Modern Physics, 1982.
Eine andere hierzu relevante, häufig zitierte Arbeit (die statt
Projektionsoperatoren aus der Quantenfeldtheorie entliehene
Diagrammtechniken verwendet) ist:
PC Martin, ED Siggia, HA Rose,
Statistical Dynamics of Classical Systems,
Phys. Rev. A 8, 423-437 (1973).


Interessanterweise (und daher leider die traditionelle Verwechslung
nahe legend) haben beide Formen des Erwartungswerts genau dieselben
mathematischen Eigenschaften, obwohl sie grundlegend verschiedene
Dinge ausdrücken. Turbulente klassische Systeme haben eben keine
nackten Observablen mehr (die sind dort genauso ill-defined wie
in der Quantenfeldtheorie), sondern nur noch die renormierten.
Aber um die Dynamik zu beschreiben und mit dem Experiment zu
korrelieren, braucht man beide. Für Hintergrundmaterial: z.B.
"turbulence renormalization" in http://scholar.google.com/


Diese Situation entspricht nun schon fast vollkommen der der
Quantenmechanik. Der einzige Unterschied ist der, dass in der
Quantenmechanik die nackten Observablen aufhören, klassisch
zu kommutieren, und zu Operatoren im Hilbertraum mutieren.
Und in der relativistischen Quantenfeldtheorie kommt noch
dazu, dass der Einfluss der unmessbar hohen Frequenzen in
einem gewissen Sinn unendlich groß ist, so dass selbst
das Renormierungsproblem schon zu erheblichen Schwierigkeiten
führt.


Ich hoffe, damit deutlich gemacht zu haben, in welchem Sinn
die Thermische Interpretation zu verstehen ist.
Die objektiven Größen sind die renormierten Erwartungswerte
<f> der nackten Feld-Operatoren f, in vollständiger Analogie
zur oben geschilderten klassischen Situation.

Gewisse makroskopische Operatoren S(omega) des Messapparats
haben renormierte Werte <S(omega)>, die mit Eigenschaften
wie dem Spin eines einzelnen Teilchens im Experiment omega
korrelieren, und werden daher als Pointervariablen benutzt.
Macht man nun eine Messung N mal in zufälligen,
unabhängigen Experimenten omega_k, ist die Verteilung
der im Prinzip messbaren renormierten <S(omega_k)>
(und nicht, wie die Kopenhagen-Interpretation behauptet,
die Verteilung irgendwelcher angeblich durch zufällige
Zustandsreduktion entstehender Eigenwerte des Teilchens
im Experiment omega_k) ungefähr die, die man bekommt,
wenn man statt der Messwerte im Detektor die aus dem
präparierten Einteilchenzustand resultierende Verteilung
gemäß der Bornschen Regel zugrunde legt.

Dies ist der nichttriviale Punkt, für dessen Demonstration
man den Projektionsoperatorformalismus der statistischen
Mechanik braucht.



----------------------------------------------------------
S16. Thermische und Kopenhagen-Interpretation
----------------------------------------------------------

Die Kopenhagener Interpretation fordert nach Bohr
''die Grenzziehung zwischen einem zu untersuchenden Quantensystem
und der klassisch zu beschreibenden Messapparatur''
http://theory.gsi.de/~vanhees/faq/epr/node4.html Klassisch beschrieben bedeutet: makroskopisch, mit Mitteln der Thermodynamik, beschreiben. Die einzigen thermodynamischen Größen, die es gibt, sind laut statistischer Mechanik Erwartungswerte von langsam veränderlichen mikroskopischen Operatoren. Diese Erwartungswerte (z.B. der mittlere Ort einer Zeigerspitze)
werden also in einem Experiment gemessen, und daraus werden
Rückschlüsse über die Eigenschaften des damit gekoppelten
Quantensystems deduziert.

Alles, was die Thermische Interpretation tut,
ist, diese Forderung der Kopenhagener Interpretation konsistent
weiterzudenken, um die Spaltung der Welt in klassische Objekte
und Quantenobjekte zu überwinden.

Die Wahrscheinlichkeitsstruktur ergibt sich dabei aus der
deterministischen Dynamik des Universums durch Reduktion der
Komplexität auf die für ein Modell relevanten Größen.
Dies geschieht mittels traditioneller Instrumente der
statistischen Mechanik (Projektionsoperator-Formalismus)
auf analoge Weise, wie sich die stochastische Brownsche Bewegung
für ein Kolloid-Teilchen in einer klassischen Flüssigkeit
aus der deterministischen Hamiltonschen Vielteilchendynamik
ergibt.


-----------------------------------------------
S17. Warum ist niemand vor mir darauf gekommen?
-----------------------------------------------

Versuche, die Quantenmechanik als eine deterministische Theorie
des gesamten Universums zu verstehen, gab es sicher viele;
allerdings nur wenige in wissenschaftlich publizierter Form.
Die Schwierigkeit liegt darin, aus philosophischen Überlegungen
ein quantitatives Konzept zu machen, mit dem die Bornsche Regel
abgeleitet und die traditionellen Gegenargumente (insbesondere von
Wigner) überzeugend außer Kraft setzt.


Die beiden wichtigsten Alternativen (Bohm, Vielwelten)
haben aber ihre eigenen gravierenden Probleme und haben sich
deshalb nicht durchgesetzt.

In Bohms Interpretation ist vieles gegenintuitiv;
z.B.ist das Elektron eines Wasserstoffatoms im Grundzustand an einem
fixen Ort (relativ zum Atomkern) lokalisiert und bewegt
sich nicht von der Stelle! Daher wird diese Interpretation nur von
einer Minderheit akzeptiert. Siehe auch
http://www.mat.univie.ac.at/~neum/papers/physpapers.html#bohm In der Vielwelteninterpretation werden eine Vielzahl prinzipiell unbeobachtbarer Alternativwelten postuliert, und der Wahrscheinlichkeitsbegriff verliert seine traditionelle Bedeutung in der einen, tatsächlich beobachteten Welt. Daher ist auch diese Interpretation sehr umstritten. Siehe auch http://www.mat.univie.ac.at/~neum/papers/physpapers.html#manyworlds Das Neue an der Thermischen Interpretation ist die Identifikation der beobachtbaren Größen mit den langsam veränderlichen Erwartungswerten mikroskopischer Größen, im Sinne der statistischen Mechanik, statt diesen Erwartungswerten einen statistischen Sinn zu unterlegen. Dies ergibt einen klaren und intuitiven Hintergrund, auf dem sich eine widerspruchsfreie Begründung der Wahrscheinlichkeitsstruktur aufbauen lässt. In seinem Buch 'Direction of Physics' schreibt Dirac (1975) auf S.10: ''And I think it is quite likely that at some future time we may get an improved quantum mechanics in which there will be a return to determinism and which will, therefore, justify the Einstein point of view.'' (Damit meinte er _nicht_ die Bohmsche Mechanik, denn die lag schon in der Vergangenheit, hat ihn also offensichtlich nicht überzeugt.) ''But such a return to determinism could only be made at the expense of giving up some other basic idea which we now assume without question.'' Was man aufgeben muss, ist die bisher fraglos akzeptierte Interpretation des Erwartungswerts als statistischen Begriff.
(Dirac bezieht sich allerdings außerdem auf Renormierungsprobleme
in der Quantenfeldtheorie, die dadurch noch nicht behoben sind,
aber vielleicht in einem neuen Licht erscheinen; vgl. die
Bemerkungen zur Renormierung im Abschnitt ''Zwei Arten von
Erwartungswerten''.)


Es gibt einige leise Andeutungen einiger Autoren, die,
wenn sie weiter in diese Richtung gedacht hätten,
wahrscheinlich auf dieselbe Interpretation gekommen wären:

''In a statistical description of nature only expectation values
of correlations are observable.''
(Christof Wetterich, 1997, in hep-th/9703006)
Wetterich http://www.thphys.uni-heidelberg.de/~wetteric/
ist einer der heutigen Experten für Quantenfeldtheorie im
Nichtgleichgewicht und die zugehörige Renormierungstheorie,
mit numerisch eindrucksvollen Rechnungen, die mit dem Experiment
verglichen werden können.

''the only reasonable interpretation of the variables p and q is
as mean values rather than truly sharp values since we live in
a world where hbar is nonzero.''
(John Klauder 2001, in: quant-ph/0112010)
''One is almost tempted to assert that the usual interpretation in
terms of sharp eigenvalues is 'wrong', because it cannot be
consistently maintained, while the interpretation in terms of
expectation values is 'right', because it can be consistently
maintained.''
(John Klauder 1997, in: quant-ph/9710029)
Klauder http://www.phys.ufl.edu/~klauder/ ist einer der Physiker, der das Modellieren mit kohärenten Zuständen in der Quantenoptik durchgesetzt hat. Der von Klauder erwähnten Versuchung nachzugeben ist allerdings zunächst ein riskantes Abenteuer, da man zunächst damit allein auf weiter Flur steht. Wenn man es aber eingeht, wird man reich dafür belohnt... ---------------------------------------- S20. Der Messprozess im Quantenuniversum ---------------------------------------- Die traditionelle Interpretation der Quantenmechanik teilt die Welt in kleine Quantensysteme und grosse Messapparaturen auf, und beschränkt sich darauf, statistische Aussagen über die dann gemessenen Ergebnisse zu diskutieren. Andrerseits herrscht ein Konsens, dass die Quantenmechanik für das gesamte Universum gelten sollte. (Sonst wäre z.B. die Suche nach eine Quantengravitationstheorie müßig.) Dann ist das gesamte Universum also ein Quantensystem, und die traditionelle Interpretation ist nicht mehr anwendbar. Offensichtlich vermessen wir aber das Universum, also braucht man eine verbesserte Theorie des Messprozesses. Der Messapparat ist hier, anders als sonst, ein Teil des vermessenen Systems. Man muss also den traditionellen Formalismus etwas verallgemeinern oder modifizieren. Das hat dazu geführt, dass eine Reihe von Alternativen diskutiert werden (Vielweltentheorie, Dekohärenz, Konsistente-Geschichte-Theorie, usw.), die aber alle ihre Haken haben und die ich deshalb hier nicht bespreche. Meine Meinung dazu findet man in http://www.mat.univie.ac.at/~neum/manyworlds.txt
http://www.mat.univie.ac.at/~neum/zeh.txt Statt dessen diskutiere ich hier meine eigene Interpretation. In Kurzfassung sagt die Thermische Interpretation über Messen im Universum das Folgende: Das Universum ist das einzige abgeschlossene System, das es in unserer Nähe gibt. Es ist unmöglich, dafür zu sorgen, dass aus einem System keine Photonen entweichen, kein Energieaustausch mit dem Gefäß stattfindet, usw. Genau diese Dinge aber sorgen dafür, dass man das System nicht mehr als deterministisch betrachten kann, weil die Einflüsse von außen unkontrolliert sind. (Auch wenn das auf den ersten Blick vernachlässigbar erscheint, ist es das nur, wenn man makroskopische Phänomene untersucht. Mikroskopische Phänomene hängen extrem empfindlich von ihrer Umgebung ab. Das wird genauer diskutiert in der Literatur über Dekohärenz.) Da das Universum abgeschlossen ist, genügt es einer deterministischen Dynamik. Da wir stets nur einen kleinen Ausschnitt des Universums präparieren, wenn wir Versuche machen, diese dann aber so beschreiben als wären sie isoliert in der Welt, ist es kein Wunder, dass wir anscheinend unerklärliche Zufälligkeiten beobachten. Das ist einfach eine Begleiterscheinung unserer Begrenztheit und der traditionell schlampigen Argumentationsweise. Sobald man nämlich nur ein Teilsystem eines beliebigen deterministischen Systems betrachtet, verliert man nämlich Information und kann das Teilsystem daher nur noch stochastisch beschreiben. Die praktisch beobachteten Phänomene lassen sich mit Diffusions- und Sprungprozessen modellieren, die sich aus der deterministischen Dynamik mit Hilfe des Projektionsformalismus der statistischen Mechanik herleiten lassen (rigoros allerdings nur unter sehr einschränkenden Voraussetzungen). Dies gilt natürlich ebenso für Teilsysteme des Universums, und erklärt damit den Zufall vollständig. Wenn man den Zustand des Universums also exakt kennen würde, würde man alles, was man messen kann, auch vorhersagen können. Dass Messwerte trotzdem nicht hundertprozentig sicher sind, ist also nicht eine Folge eines irreduziblen Quantenzufalls, sondern eine Folge der Tatsache, dass man mit einem Vielteilchensystem Eigenschaften eines anderen misst. Unsichere Messergebnisse und irreduzibler Quantenzufall haben nicht notwendig etwas miteinander zu tun. Denn Messwerte sind auch klassisch schon unsicher, wenn man mit einem Vielteilchensystem ein anderes misst. Und das trotz klassischer Determiniertheit aller Ereignisse (inklusive Messwerte) durch den Zustand des Universums. Die Thermische Interpretation postuliert nun eine deterministische Dynamik für das Universum als Ganzes, und deduziert daraus eine approximative stochastische Dynamik für jedes Teilsystem. Die Thermische Interpretation stellt also (nur) den Determinismus wieder her. Die Unsicherheit der Messung wird nicht vermieden, aber wieder auf das vor 1900 herrschende, philosophisch unproblematische Maß zurückgeführt. --------------------------------------------- S21. Das Quantenuniversum als formales Modell --------------------------------------------- Die deterministische Dynamik des Universums ist im Schrödingerbild durch die von-Neumann-Gleichung und im Heisenbergbild durch die Heisenberggleichung gegeben. Das Heisenbergbild ist das vollständigere, weil es uns erlaubt, Zeitkorrelationen zu modellieren und sich außerdem leicht relativistisch verallgemeinern lässt. In dieser Diskussion gehen wir aber stets von einem nichtrelativistischen Universum aus und verwenden das Schrödingerbild, um die Dinge zu vereinfachen. Der Zustand des Universums wird hier also durch ein zeitabhängiges Ensemble modelliert. In der Thermischen Interpretation ist das Universum als Ganzes klassisch deterministisch und hat eine klassische Hamiltonsche Beschreibung (symplektisch, wenn man annimmt, der Zustand des Universums sei rein, Poissonsch im Allgemeinfall. Um Schreibarbeit beim Umgang mit Poissonklammern (vor allem auf dem Papier - ich habe Tausende von solchen Klammern geschrieben...) zu ersparen, habe ich mir die einfache Bezeichnung f \lp g = {g,f} erfunden. \lp ist ein Symbol, da wie ein um 180 Grad gedrehtes L aussieht (aber mit gleichlangen Schenkeln, siehe (37) in [EECQ}) und 'Lie' zu lesen ist. In LaTeX sieht das entsprechende Macro so aus: \def\lp{\mbox{\Large$\,_\urcorner\,$}} Die Poissonklammer erscheint so als binäre Operation. Wir nennen f \lp g das Lie-Produkt von f und g. Es ist bilinear in den Argumenten und hat die Eigenschaften f \lp f = 0, f \lp g = - g \lp f f \lp gh = (f \lp g)h + g(f \lp h) (Leibniz) f \lp (g \lp h) = (f \lp g) \lp h + g \lp (f \lp h) (Jacobi) und zwei analoge gespiegelte Formeln, die sich aus f \lp g = - g \lp f ergeben. Die Größen in der Thermischen Interpretation sind Elemente f einer fixen Algebra E von Operatoren auf einem dichten Teilraums eines universellen Hilbertraum. Typische Größen sind z.B. Integrale f = integral dx^3 a*(x) c(x,Nabla x) a(x) wobei a(x) ein Quantenfeld ist und c(x,Nabla x) ein Differentialoperator, Linearkombinationen von Produkten solcher Funktionen, sowie Verallgemeinerungen davon. Die deterministische Differentialgleichung für den Wert <f> =<f>_t der Größe f an der Stelle t (wir unterdrücken im folgenden das
Zeitargument, wenn nicht unbedingt nötig) ist nun eine klassischen
Hamiltonschen Dynamik der Form
d/dt <f> = <H> \lp <f>, (*)
wobei H der Hamiltonoperator des Universums, eine spezielle,
selbstadjungierte Größe, ist und
<f> \lp <g> := <i/hbar [f,g]>
(wie man unschwer nachrechnet) eine klassische Lie-Poissonklammer
im Raum der glatten Funktionen
F(<f_1),<f_2>,...,<f_n>) (**)
mit beliebigen <f_i> definiert.

Im Einklang mit der quantenmechanischen Tradition setzen wir voraus,
dass Ensembles 'normal' sind, sich also durch Dichtematrizen
beschreiben lassen. Wir beschreiben den Zustand des Universums zum
Zeitpunkt t also durch eine Dichtematrix rho(t), einen Hermitischen,
semidefiniten Spurklasseoperator mit Spur 1. (Ob der Zustand rein
ist, lässt sich nicht entscheiden, da der uns zugängliche Teil des
Universums klein ist und die Projektion darauf schon die Reinheit
verdirbt.)

Die universelle Dichtematrix rho(t) bestimmt das
Universalensemble durch die Vorschrift
<f>_t := trace rho(t) f
für den objektiven Wert jeder Größe f zum Zeitpunkt t.
Im Prinzip beobachtbar sind davon die genügend langsam zeitlich
und räumlich veränderlichen Größen; alle anderen sind 'verborgen',
d.h. der Messung unzugänglich.


Alle in der Praxis aus Rohmessungen berechneten Messwerte haben die
Form (**), wenn man von möglichen Stellen absieht, wo die
Rechenvorschrift nicht differenzierbar ist. Allerdings sind die auf
der rechten Seite von (*) auftretenden Ausdrücke in der Regel keine
direkt messbaren Variablen mehr, sondern enthalten 'verborgene'
Korrelationen. Das macht die klassische Dynamik nichtlokal und
produziert Quanteneffekte.

Mit ein bisschen Überlegung findet man nun auch wirklich heraus,
dass (*) nichts anderes ist als die traditionelle von-Neumann Dynamik
für den Dichteoperator, nur ausgedrückt durch die Variablen
der Thermischen Interpretation. Daraus folgt die
vollständige Konsistenz der Thermischen Interpretation
mit dem quantenmechanischen Formalismus.




------------------------
S22. Ein Modelluniversum
------------------------

In der Thermischen Interpretation ist das Universum
durch drei mathematische Objekte festgelegt:
1. eine fixe Algebra E von Operatoren auf einem dichten Teilraums
eines universellen Hilbertraum,
2. einem selbstadjungierten universellen Hamiltonoperator H
aus dieser Algebra,
3. einem normalen Zustand rho auf dieser Algebra.

Für das reale Universum ist die Algebra E der Größen von den
Feldern des Standardmodells zusammen mit der Raumzeitmetrik
erzeugt, und der Hamiltonoperator der aus der zugehörigen
Wirkung kanonisch hergeleitete. Der Zustand des Universums ist
hingegen weitgehend unbekannt, da eine Kenntnis desselben
im Rahmen der Thermischen Interpretation die Kenntnis
aller Werte sämtlicher Felder und Korrelationsfunktionen beliebiger
Ordnung an allen Orten und zu jeder Zeit impliziert.
Dagegen sind die Zustände vieler Teilsysteme einigermaßen bekannt,
insbesondere derer, mit denen Physiker experimentieren.

Wegen der bisher ungelösten Probleme der Quantengravitation
und der Schwierigkeiten, aus dem Standardmodell Aussagen über
makroskopische Detektoren abzuleiten, eignet sich dieses voll
realistische Modell allerdings nicht für die konkrete Analyse
eines realen Messprozesses.

Damit man sich unter dem abstrakten Universumsbegriff aber etwas
konkretes vorstellen kann, sei hier ein Beispiel eines
Modelluniversums gegeben, das einfach genug ist, um es leicht
zu spezifizieren und doch wichtige Aspekte des realen Universums
wiedergibt. Wie beim realen Universum spezifizieren wir die
Algebra E der Größen und den Hamiltonoperator H, lassen aber
den Zustand rho offen und betrachten höchstens die dadurch
induzierten Zustände auf Teilsystemen (siehe den Abschnitt
''Physikalische Systeme und ihre Messung''),
soweit sie für ein Experiment relevant sind.


Wir betrachten dazu ein nichtrelativistisches Modelluniversum,
dessen Materie aus einer unbestimmten Zahl von
elementaren Kernen mit Masse m_l, Spin s_l und Ladung eZ_l
(l=1,...L, Z_l>0 ganz) sowie
Elektronen mit Masse m_0, Spin 1/2 und Ladung eZ_0 (Z_0=-1)
besteht. Die Teilchen jeder Sorte sind ununterscheidbar.
Zugehörige Basisgrössen sind ausser Massen, Ladungen und
(für Elektronen) Paulimatrizen die 3-dimensionalen Ortskoordinaten
x^a und die zugehörigen Impulse p^a, je einer pro Teilchen a,
mit den üblichen Kommutatorrelationen.

Außerdem enthält das Modelluniversum Strahlung, ausschließlich
in Form einer unbestimmten Zahl von
Photonen, Bosonen mit Masse 0, Spin 0 und Ladung 0
und einer Frequenz im sichtbaren Bereich, also mit 3-dimensionalen
Wellenvektoren k mit Frequenz omega=|k| aus einer Oktave in diesem
Frequenzband entspricht. Zugehörige Basisgrößen sind die
Strahlungsenergie H_rad := integral dk |k| a^*(k)a(k)
und für jedes Teilchen a ein
Strahlungspotential U^a := g integral dk a(k) exp(ik dot x^a).
Dabei gehen die Integrale über die Kugelschale
K = {k | |k| in ]1,2[}
die a(k) sind Vernichteroperatoren und die a^*(k) die
dazu adjungierten Erzeugeroperatoren mit den üblichen
Kommutatorrelationen.
(Die Einheiten sind so gewählt, dass c=hbar=1 und die Frequenz
des unteren Randes des Strahlungsspektrums 1 ist.)


Die Algebra der Größen ist die von den Basisgrößen und allen
Schwarzfunktionen in den x^a erzeugte Algebra von linearen
Operatoren auf dem Raum
\H = \H_matter tensor \H_rad,
wobei \H_matter aus dem Nullraum des Ladungsoperators
Q = sum_l=0^L a_l^*(x) Z_l a_l(x)
im Fockraum über dem Raum der Schwarzfunktionen
im R^3 (mit der dem jeweiligen Spin entsprechenden Statistik) ist
und \H_rad der Fockraum über dem Raum der Schwarzfunktionen auf K..

Den Hamiltonoperator des Universums setzen wir an als
H = H_\matter + H_rad + g sum_a (U^a+(U^a)^*)
mit einer Kopplungskonstante g. Dabei ist H_matter
der Hamiltonoperator der Materie, gegeben durch die traditionelle
Formel mit der Coulomb-Wechselwirkung.


Das Modelluniversum ist translations- und rotationsinvariant,
und Streuprobleme lassen sich mit den traditionellen Formeln
ohne Infrarot- oder Ultraviolettprobleme lösen.

Es gibt
Atome, Moleküle, chemische Reaktionen und Molekülspektren,
keine Radioaktivität oder Kernreaktionen,
kein polarisiertes Licht,
keine Gravitation,
keine mikroskopischen Felder,
jedoch erzeugt die Coulomb-Wechselwirkung makroskopische
elektromagnetische Felder.

Damit ist ein Großteil der realen Wirklichkeit qualitativ
präsent, ohne dass die typischen Schwierigkeiten der
Quantenfeldtheorie und der Eichtheorie auftreten.

Insbesondere ist die gesamte Chemie des Universums
(mit Ausnahme von Laserchemie) reproduzierbar,
ebenso die gesamte Strömungsmechanik, die geometrische Optik
und fast die gesamte Festkörperphysik. Das hat zur Folge, dass
sich alle mechanischen oder hydraulischen Messgeräte,
und die meisten optischen und elektrischen modellieren lassen.


Insbesondere lassen sich fotographisch aufgezeichnete Versuche
mit Blenden und Bildschirmen, wie das Doppelspaltexperiment,
mit Licht und Photodetektoren, sowie Versuche mit Magneten,
wie das Stern-Gerlach-Experiment in unserem Modelluniversum
durchführen.

Vorausgesetzt, es gibt darin Physiker, die die Experimente
durchführen. Dies aus einem mikroskopischen Modell abzuleiten,
übersteigt die Möglichkeiten gegenwärtiger Physik,
so dass dies ohne Beweis angenommen wird. Während der
Durchführung des Versuchs sind Physiker entbehrlich; ihre
Rolle beschränkt sich auf Präparation des Experiments und
das spätere Anschauen von Fotografien. Wir werden daher
Physiker stets als Teil der irrelevanten Umgebung des Experiments
betrachten. Damit ist keine Herabsetzung der Zunft der
experimentellen Physiker, die für all unser Detailwissen
über das Universum verantwortlich sind, beabsichtigt.


In unserem vereinfachten Modell des Universums bedeutet also
Präparation eines Experimentes einfach die Behauptung (Annahme),
es gäbe im Modelluniversum ein physikalisches System in einem
Anfangszustand mit den für die vollständige Beschreibung
des Experiments notwendigen Eigenschaften. Die Aufgabe der
Analyse ist es, die daraus resultierenden Beobachtungen zu
erklären.



-------------------------------------------
S23. Physikalische Systeme und ihre Messung
-------------------------------------------

In der Thermischen Interpretation ist ein
physikalisches System S definiert durch eine Algebra E_S
von zugehörigen Größen, auf der eine Spur trace_S mit der
Eigenschaft
trace_S fg = trace_S gf
definiert ist. Typische Beispiele sind die Algebra der Operatoren
auf einem kleinen Teilhilbertraum des universellen Hilbertraums
(das gibt das traditionelle kleine Quantensystem),
oder Algebren die von gewissen Integralen der Form
f = integral dx^3 a*(x) c(x,Nabla x) a(x)
erzeugt werden, wo c(x,Nabla x) nur am (mehr oder weniger ausgedehnten)
physikalischen Ort des Systems wesentlich von Null verschieden ist
(das gibt klassische Subsysteme auf der hydrodynamischen Ebene).

Ein physikalisches System S hat zur Zeit t die objektiven
Werte <f>_t für alle f in E_S, wo <.>_t
das universelle Ensemble ist.

Diese objektiven Eigenschaften lassen sich vollständig
durch die reduzierte Dichtematrix rho_S(t) in E_S beschreiben,
die durch
<f>_t = trace_S rho_S(t) f für alle f in E_S
und den Zustand des Universums vollkommen festgelegt.

Es ist, nebenbei bemerkt, extrem unwahrscheinlich,
dass der Zustand rho_S(t) rein ist, außer man beschränkt die Algebra
der Größen auf eine niedrigdimensionale Matrixalgebra
(also z.B. 8-dimensional für ein System von drei Spinvariablen)
und präpariert den Zustand sorgfältig.


Um der Bezeichnung 'objektiv' gerecht zu werden, muss geklärt werden,
wie man sie misst. Der Wert <f>_t einer Größe f des physikalisches
System S ist im Prinzip beobachtbar, falls er sich zeitlich und
räumlich nicht zu schnell ändert und falls das System langlebig
genug ist, die Messung vorzunehmen. Um <f>_t zu messen,
braucht man ein makroskopisches Gerät (d.h. Vielteilchensystem,
durch statistische Mechanik beschrieben), das so mit dem System
gekoppelt ist,
1. dass die durch die Kopplung entstehende Dynamik den zu messenden
Wert <f> nicht verdirbt (sonst misst man zwar etwas, aber nicht
das Gewünschte), und
2. dass es eine makroskopische Pointervariable x besitzt, von der die
Theorie zeigen kann, dass die Kopplung impliziert, dass
nach genügender Wartezeit ein Gleichgewichtszustand erreicht
ist, für den (im einfachsten Fall) <x>=K<f> mit einer
bekannten Konstante K gilt. Dann kann man nämlich <f>
aus der Beobachtung von <x> ausrechnen: <f>=<x>/K.
Ist das der Fall, so sagt man, man habe <f> gemessen.

Das ist eine präzise Definition des Messprozesses, die es
- anders als in allen bisherigen Interpretationen der Quantenmechanik -
erlaubt, den Messprozess auf der Basis des zugrundegelegten Modells
allein zu analysieren, insbesondere ohne Annahmen über
Wahrscheinlichkeiten, Kollaps, oder ähnliches.

Im Allgemeinen gibt es bei 1. und/oder 2. gewisse Probleme,
das auf Grund der Dynamik des Vielteilchenquantensystems
(was die einzige erlaubte Basis für das Vorgehen in 2. ist)
sicherzustellen, und als Folge davon kann man <f> in der
Regel nur approximativ bestimmen. Wenn man genauer rechnet,
wird man feststellen, dass man zwar ein einzelnes <f> im Prinzip
genügend genau bestimmen kann. Aber für ein Paar
komplementärer Variablen (Ort und Impuls, oder Spin in
unterschiedliche Richtungen) ist die gleichzeitige
Messgenauigkeit grundsätzlich durch eine Unschärferelation
beschränkt; siehe Section 3 in [EECQ].



-------------------------------------------
S24. Vorhersage im Stern-Gerlach Experiment
-------------------------------------------

Kennt man den Zustand des Universums vollständig,
so kann man laut Thermischer Interpretation
alle Einzelmessungen vorhersagen.

Insbesondere sollte man also den Ort, wo Teilchen in einem
Stern-Gerlach Experiment auftreffen, vorhersagen können.

Da wir den Zustand in Wirklichkeit nicht kennen, ist das
natürlich eine müßige Überlegung, aber es geht ja ums Prinzip.
Wir arbeiten dazu im Schrödingerbild, und bezeichnen den
Zustand des Universums zum Zeitpunkt t mit rho(t).
Das ist eine Dichtematrix, ein Operator auf dem Hilbertraum
des Universums. Sie bestimmt das Universalensemble durch die
Vorschrift
<f>_t := trace rho(t) f
für den objektiven Wert jeder beobachtbaren (genügend langsam
zeitlich und räumlich veränderlichen) Größe f.

Im klassische Stern-Gerlach Experiment gehen
Silberatome an einem Magneten vorbei und werden anschließend
auf einem Detektor aufgefangen. Die Verteilung des Silbers
auf dem Detektor ist dann durch ein Feld S(x) gegeben,
das man mittels Quantenfeldoperatoren im Prinzip
hinschreiben kann.

Da dieses Feld in der für den Versuch relevanten Auflösung
makroskopisch ist, kann man lokales Gleichgewicht voraussetzen
und erhalte (gemäß statistischer Mechanik im lokalen
Gleichgewicht, wie sie in vielen Büchern beschrieben wird)
als (bis auf eine gewisse Genauigkeit) beobachtbare
Silberverteilung
S(x,t) := <S(x)>_t = trace rho(t) S(x).
Was man messen kann, ist also offensichtlich durch den Zustand
des Universums bestimmt. Wenn Gott
1. diesen Zustand zum Zeitpunkt t=0 kennt,
2. den Hamiltonoperator des Universums kennt, und
3. die von-Neumann-Gleichung lösen kann,
kann er damit rho(t) ausrechnen und daher die beobachtbare
Silberverteilung zu jedem Zeitpunkt vorhersagen.

Wenn man etwas genauer modelliert, und außerdem die Quantenquelle
samt Magneten mit einbezieht, findet man, dass S(x,t)
einem Sprung-Diffusionsprozess genügt, der mit den
zufälligen Einzelbeobachtungen und der vorhersagbaren
Verteilung (nämlich zwei Flecken, die den beiden
Spin-Eigenwerten entsprechen) gut übereinstimmt.



----------------------------------------------------
S25. Was passiert einzelnen Photonen am Doppelspalt?
----------------------------------------------------

Das einzelne Photon ist in meiner Interpretation nicht
messbar, es hat keine zugehörige Beobachtungsgröße.
Photonen sind ununterscheidbar, man kann also nicht sagen,
wo ein bestimmte Photon ist. Was existiert, ist die
Photonendichte. Die drückt sich sozusagen wie ein Öl
durch den Doppelspalt, und bildet nach den Regeln der
Quantenmechanik ein Interferenzmuster in der Dichte aus.
(Vergleich als Bild nehmen und nicht allzu wörtlich!)

Beim Auftreffen auf dem Schirm sorgt die nichtlokale Dynamik
dafür, dass von Zeit zu Zeit proportional zur Photonendichte
ein Elektron in einen angeregten Zustand versetzt wird,
eine chemische Reaktion stattfindet, oder was immer als
Detektionsmechanismus gerade relevant ist. Dass dies
stochastisch geschieht, liegt daran, dass das Experiment
hochempfindlich auf den Rest des Universums reagiert.

Wie man dieses stochastische Verhalten auf der formalen Ebene
begründen kann, wird im FAQ im Abschnitt
''Wie erklärt sich der Zufall?'' abgehandelt. Das ist allerdings
etwas technischer und erfordert fortgeschrittene Techniken
der statistischen Mechanik.


V.B. Braginsky and F.Ya. Khalili,
Quantum measurement,
Cambridge Univ. Press, Cambridge 1992
ist ein ausgezeichnetes Buch über _reale_ Quantenmessprozesse,
die sich eng an moderne Quantenoptikexperimente anlehnen, und nichts
von der von-Neumann-Karikatur einer Quantenmessung haben.
Auf S.3 unten schreiben sie:
''Experiments on the interference and diffraction of light,
when performed with very low intensities, revealed further that an
interference pattern (a classical, pure wave effect) shows up on
a photographic plate only when the number of photons falling on
the plate is very large. Each photon in such an experiment
is _completely_destroyed_ [original italic] (ceases to exist)
by interacting with the plate's silver chloride molecules.
When the photon is destoyed there appears somewhere on the
photographic plate an atom of free silver, which acts as an
embryo from which, by photographic developing, a small seed
of silver will grow. The silver embryo is much smaller than
an electromagnetic wavelength.
This is remarkable. In the interference process (e.g. in
the two-slit experiment of Fig. 1.1), [standard picture] the
photon must have been influenced by the locations of both slits,
since the interference pattern depends on the distance between
them. This means that the photon must have occupied a volume larger
than the slit separation. On the other hand, when it fell on the
photographic plate, the photon must have been localized into the
tiny volume of the silver embryo. Later the terms 'collapse of
the wave function' and 'reduction of the wave packet' were used
to describe such localization.''
Im Einklang mit der Thermischen Interpretation
schreiben sie einem Photon im Doppelspaltexperiment ein Volumen zu,
das größer ist als der Abstand der beiden Spalte.



----------------------------------------------------------------------
S26. Der Quantenradierer in der Thermischen Interpretation
----------------------------------------------------------------------

(Für den Hintergrund des Experiments siehe
http://theory.gsi.de/~vanhees/faq/qradierer/qradierer.html wo der experimentelle Rahmen und seine statistische Interpretation ausführlich diskutiert wird.) Ein elektromagnetisches Quantenfeld wird von der Quelle erzeugt. Bis zum Spalt ist die (im Prinzip messbare) Feldstärke <E(x)> auf eine kleine Umgebung eines Strahls konzentriert. Nach dem Passieren des Filters ist die Feldstärke (in der Schnittebene senkrecht zu den Spalten) die zweier Kugelwellen. Hinter dem lambda/4-Plättchen ist die Feldstärke auch noch die eines Sektors der zwei Kugelwellen, aber in den beiden Hälften sind beide Kugelwellen unterschiedlich polarisiert (gibt insgesamt 6 Fälle: Jede Kugelwelle kann L- R- oder unpolarisiert sein, je nachdem, an welchem Ort. All das kann man objektiv nachprüfen, indem man einen Detektor an die zu prüfende Raumstelle stellt und eine genügen lange Statistik macht. Die Detektoren messen die jeweilige Intensität, da diese proportional zum statistischen Mittel der Entladungen ist. Die Polarisationsrichtung kann man auch messen, indem man vor den Detektor noch ein Polarisationsfilter stellt. Benutzt man nun (durch parametrische Downkonversion erzeugte) verschränkte Photonenpaare, ist die Situation ein bisschen komplizierter, da das System nun durch lokale Größen nicht mehr hinreichend gut beschrieben ist. Aber die Thermische Interpretation kennt nichtlokale Größen: Paarkorrelationen von elektromagnetischen Feldern. Diese spielen auch schon klassisch eine wichtige Rolle, wo sie benötigt werden, Polarisationsphänomene klassisch zu beschreiben. Siehe etwa L. Mandel and E. Wolf, Optical Coherence and Quantum Optics, Cambridge University Press, 1995. Um Paarkorrelationen zu messen, braucht man zwei an unterschiedlichen Orten aufgestellte Detektoren. (Darin besteht die Nichtlokalität.) Ebenso wie bei der Vermessung der Feldstärke im einfachen Experiment kann man sich davon überzeugen, dass die Paarkorrelationen objektive Eigenschaften des Quantenfelds sind. (Die Messzeiten, um genaue Messergebnisse zu bekommen, sind allerdings erheblich höher.) Die Messung an den im Experiment fest aufgestellten Detektoren unterscheidet sich nicht von der Messung an beliebigen Stellen. Es sind imperfekte Messungen der Intensität bzw. der Paarkorrelationen, die wegen der Quantenstruktur der Detektoren ein erratisches Verhalten haben, aber im Mittel die Intensität und die Paarkorrelationen korrekt wiedergeben. Teilchen kommen in der Beschreibung des Experiments im Rahmen der Thermischen Interpretation gar nicht vor. Die übliche Annahme, dass einzelne Klicks im Detektor durch Eintreffen einzelner Photonen entstehen, ist rein metaphysisch und kann im Rahmen der Quantenmechanik weder bewiesen noch widerlegt werden. Sie wird in der Thermischen Interpretation verworfen. Statt dessen sind, was sich phänomenologisch wie Teilchen verhält, einfach lokal (entlang kleiner Umgebungen von Weltlinien) hohe Konzentrationen von Feldern. Also etwa wie Buchstaben lokale Konzentration eines Druckerschwärzefelds sind. Damit lassen sich alle experimentellen Befunde ohne Probleme im Rahmen der durch die Thermische Interpretation erklärten Quantentheorie quantitativ verstehen. ------------------------------------------------------- S27. Muss man den ganzen Zustand des Universums kennen? ------------------------------------------------------- Nach gängiger Auffassung kann man Experimente unabhängig vom Rest des Universums analysieren; der Zustand des Universums steht dagegen in der Thermischen Interpretation an prominenter Stelle - alles hängt davon ab, da nur seine Dynamik deterministisch ist. Allerdings dürfte der mikroskopische Zustand des gemessenen Systems samt Messgerät reichen, um die dominanten Effekte zu bekommen. Nur hängen deren zeitliche Entwicklung von _deren_ Umgebung ab, usw., so dass man nach genügend vielen Zwiebelschalen das ganze Universum hat. Die äußeren Schichten tragen natürlich nur noch wenig bei, verhindern aber ein vollständig deterministisches Bild. Man kann aber vermutlich bei genügend vorsichtiger Definition des Randes eines Experiments die Rolle des Rests vom Universum auf die Vorgabe der zeitlichen Entwicklung des Mikrozustands des Randes beschränken. Man hat dann allerdings statt einem Anfangswertproblem ein Anfangs-Randwertproblem, da der Mikrozustand des Randes als zu allen für das Experiment relevanten Zeiten bekannt sein muss.




-------------------------------------------------------
S28. Kann man den Zustand des Universums falsifizieren?
-------------------------------------------------------

über den Zustand des Universums können wir nie alles herausfinden,
aber sehr vieles schon. Insbesondere können wir leicht falsifizieren,
dass der Zustand des gesamten Universums irgendetwas blind vorgegebenes
sein kann!

Nehmen wir der Einfachheit halber an, das Universum sei in einem
reinen Zustand psi. Nehmen wir außerdem an,
wir hätten ein Koordinatensystem irgendwie eindeutig festgelegt
(etwa nach GPS-Art). [Hier lassen sich natürlich Zweifel anmelden,
ob das geht, aber Physiker machen ja auch sonst viele
Plausibilitätsannahmen, die noch hinterfragt werden könnten.)

Im zugehörigen Hilbertraum gibt es dann Operatoren M(x)
(genauer, operatorwertige Distributionen), die die Massendichte
an der Stelle x beschreiben. über die Verteilung von M(x) für x
in unserem Sonnensystem wissen wir z.B. recht gut Bescheid,
da wir diese in beliebiger Wiederholung ziemlich genau messen bzw.
extrapolieren können, also durchaus genaue Statistik darüber
anfertigen. Sei rho(x,m) die gemessene Wahrscheinlichkeitsdichte.

Die Quantenmechanik sagt nun voraus, dass für genügend schöne
Funktionen f(m) die Formel
psi^*f(M(x))psi = integral f(m) rho(x,m) dm (*)
mit einer Genauigkeit gilt, die bei beliebig gewähltem
Konfidenzlevel ebenfalls vorhergesagt wird.

Gibt man nun psi blind vor (etwa mit einem Zufallsgenerator, der
Einheitsvektoren im Hilbertraum auswürfelt), so lässt sich (8)
mit denselben Verfahren testen, mit denen auch sonstige Vorhersagen
statistisch auf ihre Verträglichkeit mit der Theorie geprüft
werden. Rein zufällig gewählte psi werden diesen Test mit
extrem hohem Konfidenzlevel nicht überleben.

Der Zustand des Universums ist also ziemlich stark durch die
Gesamtheit aller uns bekannten Fakten über das Universum
eingeschränkt. Wenn man extrapoliert, kann man also durchaus
plausibel argumentieren - mit genau denselben Argumenten,
mit denen man argumentiert, dass der Zustand eines N_Teilchensystems
festgelegt ist - dass der Zustand des Universums
eindeutig festgelegt ist, auch wenn wir ihn nie genau bestimmen
können.







----------------------------------
S30. Wie erklärt sich der Zufall?
----------------------------------

Die Technik ist im Prinzip bekannt. Siehe etwa:
H Grabert,
Projection Operator Techniques in Nonequilibrium
Statistical Mechanics,
Springer Tracts in Modern Physics, 1982.
Das ist übrigens das beste Buch über Projektionstechniken,
das ich kenne. Nicht perfekt, aber sauber und klar geschrieben,
mit allen relevanten Details, und auf das Wichtige beschränkt.

Grabert wendet die Technik auf verschiedene interessante
Spezialfälle an, und bekommt so aus der fundamentalen
Liouville-Gleichung z.B. die Navier-Stokes-Gleichungen,
stochastische Diffusionsprozesse, Mastergleichungen und
Quanten-Markov-Prozesse a la Lindblad.
Seine Darstellung geht deutlich über die durchschnittliche
hinaus, da sie
1. zeigt, dass die Methode universell ist und _alle_ traditionellen
Gleichungen liefert (und daher meiner Meinung nach ebenso prominent
in die Physikausbildung gehört wie die Hamiltonsche Mechanik!)
2. ganz deutlich macht, wie der Zufall zustande kommt
(jedenfalls, wenn man das Buch mit den richtigen Augen -
und zwischen den Zeilen - liest).


Wenn man dieselbe Technik auf den Messprozess anwendet
(was vor mir anscheinend niemand in der notwendigen Allgemeinheit
getan hat), indem man als relevante Operatoren alle Funktionen von
(kommutierenden) Pointervariablen und alle (nichtkommutierenden)
Operatoren, die das quantenmechanische Teilsystem beschreiben,
bekommt man nach einer Markov-Approximation als reduzierte
Dynamik einen quanten-klassischen Diffussions-Sprung-Prozess
für die Erwartungswerte der reduzierten Variablen.

Man sieht also genau, wie der Zufall entsteht - er ist nichts
anderes als der hochfrequente, nicht mit modellierte Anteil
der Dynamik, der weg projiziert worden ist, sich aber natürlich
nicht ganz aus der Welt schaffen lässt, sondern seine Spuren
hinterlässt.

Wahrscheinlichkeiten sind also einfach Konsequenzen der
gewählten Beschreibungsebene.

Beschreibt man das Universum mit _allen_ Details,
die es objektiv darin gibt, so ist es deterministisch.
Beschreibt man dagegen ein Teilsystem, als ob es allein im
Universum wäre (und das tun wir praktisch immer), so ist der
Einfluss des nicht mit modellierten Teils zwar trotzdem da,
kann aber nur noch approximativ durch stochastische Einflüsse
modelliert werden. Daher sieht es in dieser reduzierten Beschreibung
wie zufällig aus. Und wenn man es quantitativ fasst, kommt
aus der Analyse mit dem Projektionsformalismus gerade
die Bornsche Regel heraus!

Es ist qualitativ nicht viel anders als beim Würfeln.
Modelliert man alle Kräfte, so ist die Bahn des Würfels
determiniert; modelliert man aber nur die Punktezahl,
bekommt man die traditionellen Wahrscheinlichkeiten.
Nur die Auswertungsformeln sind etwas verschieden.


Die effektive Nichtlinearität der beobachtbaren stochastischen
Dynamik ergibt sich aus dem Projektionsformalismus.
Dort ist natürlich ein unbewiesener Schritt von derselben Art,
wie er überall auftritt, wo aus einer reversiblen eine irreversible
Dynamik wird. Aber dies gilt im Gefolge von Boltzmann heute als
gut verstanden und hat den Konsens der Physiker.


------------------------------------------------
S31. Ist der quantenmechanische Zufall objektiv?
------------------------------------------------

Lässt sich das Teilsystem objektiv auszeichnen, so ist der
durch diese Auszeichnung und die dadurch notwendige Projektion
entstehende Zufall ebenfalls objektiv.

Das gilt jedenfalls für gewisse, kanonisch ausgezeichnete
Beschreibungsebenen (Hydromechanik = lokales Gleichgewicht,
Kinetik = mikrolokales Gleichgewicht) und verschiedene
Hybride mit quantenmechanischen Systemen.
(Man kann natürlich auch sehr subjektive Beschreibungsebenen
wählen, und der Zufall wird dann dementsprechend subjektiv.)


Es ist also objektiver Zufall, der durch die reduzierte
Beschreibung auf wohldefinierten Beschreibungsebenen
entsteht. Subjektiv höchstens in dem restriktiven
Sinn, dass, was Zufall ist, von der Beschreibungsebene
(Auszeichnung des Teilsystems durch Angabe der relevanten Größen)
abhängt, und dass diese Ebene unterschiedlich gewählt werden kann,
und in diesem Sinn vom beschreibenden Subjekt abhängt.

Aber durch die Beschreibungsebene, d.h. in der Praxis die Wahl
der zeitlichen und räumlichen Skala, auf der ein Phänomen
aufgelöst werden soll, und der dadurch implizierten Algebra
der relevanten Beobachtungsgrößen, ist der Zufall _objektiv_
festgelegt, und könnte bei vollständiger Kenntnis des
Zustandes des Universums vorhergesagt werden. Die bei der
Projektion gemachten Approximationen (um einen Markov-Prozess
zu bekommen) legen die Verteilung aller relevanten Variablen
vollständig fest als Ausdrücke, die sich aus dem Zustand des
Universums im Prinzip ausrechnen lassen.

Durch unscharfes Messen kommt natürlich ein weiteres
Zufallselement ins Spiel, das rein statistischen Charakter hat.

---------------------------------------------------
S32. Wie fasst man Wahrscheinlichkeitsverteilungen?
---------------------------------------------------

Ein Experiment, das Auskunft über die Wahrscheinlichkeitsverteilung
einer Zufallsvariable x macht, erscheint in der Terminologie von
Section 8 von [EECQ] als ein Experiment, das allen genügend einfachen
Funktionen f(x) als Wert v(f(x)) den Erwartungswert von f(x)
zuordnet. Die Kenntnis sämtlicher Erwartungswerte ist gleichwertig
mit der Kenntnis der Wahrscheinlichkeitsverteilung.

In der Praxis misst man aber nur endlich viele Realisierungen von x
und kann daher für Zufallsvariable mit kontinuierlichem Spektrum
immer beliebig verrückte, irreguläre Funktionen konstruieren,
die z.B. an allen bisher gemessenen Werten 1 sind und an allen
nächste Woche zu messenden Null sind. jegliche angewandte Statistik
geht davon aus, dass man solche irregulären Funktionen ausser
Acht lässt.

--------------------------------------------
S33. Was wird aus dem Superpositionsprinzip?
--------------------------------------------

In der traditionellen Analyse des Nesssprozesses nach von Neumann
wird radikal vereinfacht (wodurch die Probleme entstehen),
indem man Messungen als Reduktion auf Eigenwerte auffasst,
und allgemeinere Situationen dann mit Hilfe des Superpositionprinzips
analysiert.


In der Thermischen Interpretation ist das ein klein bisschen
komplizierter. Wenn man nämlich ein experiment wiederholt, hat sich
der Zustand des Rests der Welt schon verändert, und man hat daher
nicht mehr exakt dieselbe Situation.

Sondern nur noch im Mittel dieselbe. Das macht den ganzen Unterschied,
Man kann nämlich nicht ganze Universen superponieren. Jedenfalls
wüsste ich nicht, wie das präpariert werden soll. Es gibt in der
Thermischen Interpretation nur _einen_ Zustand,
den des gesamten Universums. Alles andere sind Derivate.

Das Superpositionsprinzip gilt nur für Systeme, die so klein sind,
dass man sie innerhalb dieses Universums in praktisch beliebiger
Anzahl herstellen und manipulieren kann. Makroskopische Systeme
gehören definitiv nicht mehr dazu!

Diese Einschränkung bringt Wigners klassisches Argument
J.A. Wheeler and W. H. Zurek (eds.),
Quantum theory and measurement.
Princeton Univ. Press, Princeton 1983,
Kapitel II.2, insbes. pp. 285-288.
(siehe dazu den Beitrag ''Does decoherence solve the
measurement problem?'' in meinem theoretical physics FAQ
auf http://www.mat.univie.ac.at/~neum/physics-faq.txt)
zu Fall, das die Unvereinbarkeit von uneingeschränkter Unitarität,
dem uneingeschränkten Superpositionsprinzip und dem Kollaps
des Zustands bei einer Messung beweist.

Wir betrachten das detailliert im nächsten Beitrag anhand der
Messung eines einzelnen Spins.



----------------------------------
S34. Spinmessung formal betrachtet
----------------------------------

Aus der Sicht der Thermischen Interpretation
stellt sich eine Spin-Messung im Schrödingerbild formal so dar:

<.>_t ist der Zustand des Universums zum Zeitpunkt t,
monoton und linear auf der Algebra \E aller Größen.

\E_S ist die Algebra der Grössen des Systems. Also für einen Spin die
Algebra der komplexen 2x2-Matrizen A = [A_11,A_12;A_21,A_22].
J:\E_S --> \E eine unitäre Darstellung, die angibt, welcher von den
vielen Spins im Universum genau das System darstellt.
Das Teilsystem wird ebenfalls im Schrödingerbild beschrieben:
Zu einem beliebigen Zeitpunkt t ist das System S im Zustand rho_t, der
durch
<J(A)>_t = trace rho_t A für alle A in \E_S.
eindeutig festgelegt ist. Falls zu einer Zeit t die Beziehung
rho_t = psi_t psi_t^*
gilt, sagt man, man habe das Teilsystem in einem reinen Zustand psi_t
präpariert.

S ist zum Zeitpunkt t im reinen Zustand |s> (s=1,2), falls
rho_t = |s><s| gilt, also
<J(A)>_t = A_ss für alle A in \E_S.
Das wird für gewisse, präparierte Zeiten t in EIG(s) der Fall sein,
aber im Allgemeinen in der Regel nicht. Zu gewissen anderen
Zeitpunkten t in SUP sei S statt dessen in einer reinen Superposition
psi_t präpariert, also rho_t = psi_t psi_t^* und daher
<J(A)>_t = psi_t^* A psi_t für alle A in \E_S.
Während den unpräparierten Zeiten ist das System in der Regel in
einem u.U. gemischten Zustand.


z ist die gemessene makroskopische Zeigervariable, die s messen soll.
Die Reaktionszeit des Detektors (bis sich Gleichgewicht eingestellt
hat) sei R; die anschließende Totzeit (bis eine weitere
zuverlässige Messung möglich ist) sei T. Gemessen wird das zur
Zeit t präparierte System also, indem der thermodynamische
Gleichgewichtswert
s_t := <z>_{t+R}
auf eine Genauigkeit eps genau abgelesen wird.

Für ein vernünftiges Messgerät wird vorausgesetzt, dass
(innerhalb der Messgenauigkeit eps)
s_t = s für alle t in EIG(s),
falls je zwei aufeinanderfolgende Messungen mindestens den
zeitlichen Abstand R+T haben. Das kann in einer Kalibrierungsphase
geprüft werden.

Die unitäre Dynamik im Universum ist gegeben durch
<f>_t := <U(t)^*fU(t)>_0,
wobei
U(t)U^*(t)=U^*(t)U(t)=1.
Mehr weiß man a priori nicht. Offenbar kann man daraus - ganz
anders als in Wigner's idealisierter Analyse - nicht allgemein
folgern, wie der Messwert bei den in einer Superposition
präparierten Systemen aussehen muss. Dies muss statt dessen durch eine
Analyse mit den Mitteln der statistischen Mechanik geklärt werden.
Diese liefert bei einer entsprechenden Modellwechselwirkung
die gewünschte Wahrscheinlichkeitsstruktur und die Bornsche Regel.


----------------------------------------------
S35. Was ist an Wigner's Analyse idealisiert?
----------------------------------------------

Wigner's Analyse setzt zusätzlich voraus, dass die Messung
eine sogen. von-Neumann-Messung ist, d.h., dass ein System,
das (im Beispiel des Spins) im reinen Zustand |s> ist,
diesen Zustand nach der Messung beibehält.

Diese weitverbreitete Annahme ist aber beim Stern-Gerlach
Versuch nicht gerechtfertigt.

Ein Silberatom in einem Spin-up Zustand |1> sitzt nach der Messung
irgendwo auf der Schirmoberfläche, sicher nicht mehr mit Spin up,
sondern auf komplizierte Weise mit dem Schirm verschränkt!
Es besteht also ein großer Unterschied zu einer idealisierten
von-Neumann Messung!


Die Annahme einer von-Neumann-Messung beruht darauf, dass
bei einem sicheren Ergebnis eine Wiederholung der Messung,
auch mit einem neuen Messgerät, das Ergebnis reproduzieren sollte.
Das ist nur bei nicht zerstörerischen Messungen der Fall.
Nicht zerstörerische Messungen sind aber schwierig, weil sie
indirekt messen müssen und sind vor 1980 undurchführbar gewesen.
Insbesondere lässt sich bei weitem nicht alles nicht zerstörerisch
messen.

Nicht zerstörerische Messungen werden in der Literatur diskutiert,
z.B. Braginsky et al, Science 209 (1980), 547-557. Sie begründen
auch, warum Ortsmessungen (also Zeiger ablesen, Bilder ausmessen)
nie nicht zerstörerisch sein können.

Typische Messvorgange, insbesondere die für die traditionellen
Paradebeispiele (Interferenz am Doppelspalt, Stern-Gerlach,
Photoeffekt, Geigerzähler, usw.) gehören also nicht zu
den von-Neumann-Messungen.


Eine von-Neumann-Messung ist hochgradig idealisiert und beschreibt
eigentlich keine echte (irreversible) Messung, sondern nur eine
weitere Präparation. Daher kann man auch die Reduktion an eine
beliebige Stelle im Kommunikationsprozess verschieben.


Meiner Meinung nach kommt die traditionelle Identifikation von
Messung und von-Neumann-Messung durch konzeptuelle Schlamperei
zustande.

Das Vorbeifliegen eines Silberteilchens am Magneten oder das
Durchfliegen eines Photons durch einen Doppelspalt ist keine Messung,
sondern erst das Auftreffen auf der Photoplatte. (Versucht man etwa
am Spalt wirklich zu messen, verschwindet die Interferenz.)

Man betrachtet aber stillschweigend das Passieren von Filtern
(Magneten, Doppelspalt, Prisma, Polarisationsfilter ...) als eine
Messung, weil man weiss, was herauskommen würde, _wenn_ man messen
würde. Darin liegt der Fehler.

Denn die QM macht einen Unterschied (und gerät in Widersprüche,
wenn man ihn nicht macht), ob die Versuchsanordnung tatsächlich
Information an die Umgebung verliert (und nur das
macht eine Messung aus), oder ob sie es nur tun würde, wenn...
(was keine Messung ist).

Fiktive Überlegungen im Konjunktiv haben keine physikalischen
Wirkungen.



---------------------------------------------
S36. Kollaps als bedingte Wahrscheinlichkeit?
---------------------------------------------

In meinem Collapse Challenge quant-ph/0505172 beschreibe ich
das Messproblem in seiner vielleicht einfachsten Form:
dem Kollaps des Zustands beim Passieren einer Blende.
Unabhängig von der Interpretation ist auf jeden Fall ein Kollaps da;
nur bezieht er sich auf Unterschiedliches, je nach Interpretation.


1. Wenn man den Zustand (die Wellenfunktion modulo Phase) mit 'Wissen'
identifiziert, hat man die statistische Interpretation,
und der Kollaps ist dann bekanntlich einfach der Übergang zur
bedingten Wahrscheinlichkeit. Er existiert, aber er birgt keine
Probleme.

Nach dieser Interpretation ('Zustand = Wissen') sagt die Quantenphysik
nicht das Geringste über die Wirklichkeit aus, sondern nur etwas
über mentale Vorgänge im Beobachter - nämlich, wie sich
sein Wissen ändert, wenn er Notiz von einem Messergebnis
nimmt und es als echt akzeptiert. (Wenn er es nämlich nicht
akzeptiert, bleibt sein Wissen das alte und die Wellenfunktion
daher unreduziert!)

Eine konsistente Haltung, zugegeben. Aber Physik auf Psychologie
zu reduzieren, ist ein Verzweiflungsschritt, nicht eine Lösung des
Grundlagenproblems.


2. Die Alternative ist, den Zustand mit dem Objekt zu identifizieren.
Das ist das, was Physiker pragmatisch tun, wenn sie Experimente
planen und durchdenken. Und es ist das, was mit der Praxis realer
Quantenmessungen - wie etwa in dem zitierten Buch von Braginsky
und Khalili beschrieben - in Einklang ist.

Dann ist der Zustand und das Photon auf der Modellebene identisch,
im selben Sinn wie ein klassisches Bohrsches Atom ein Paar von
Punkten im Phasenraum ist.

Dann ist der Kollaps ebenfalls da, aber nun ist er ein objektives
(beobachterunabhängiges) Problem der statistischen Mechanik:
das Ergebnis der Wechselwirkung des Quantensystems mit einer
Vielteilchen-Blende.


Vor Jaynes, der die unselige psychologische Interpretation (1.)
aufgebracht hat, war meines Wissens 2. der Default.
Von Neumann und Wigner haben zwar das 'Mind' ins Spiel gebracht,
aber nicht von ''state = knowledge' geredet.





---------------------------------------------
S40. Was sind die Beables der Interpretation?
---------------------------------------------

In meiner Thermischen Interpretation existieren
alle Erwartungswerte objektiv als Beables im Sinne Bells.
Manche davon sind der Messung zugänglich - nämlich die, die
zeitlich und örtlich langsam genug veränderlich sind sowie
eine kleine Unschärfe haben. Dazu gehören wegen dem Gesetz
der großen Zahlen insbesondere die thermodynamischen Größen.

Alle übrigen sind der Messung unzugänglich - es sind die
verborgenen Variablen, nach denen Einstein und andere so
lange suchten. Die meisten davon sind hochgradig nichtlokal,
im Einklang mit Bells Theorem.

Meine Thermische Interpretation stellt also den
Realismus wieder her, ohne wie Bohm zusätzliche Freiheitsgrade
in die Quantenmechanik aufnehmen zu müssen.



--------------------------------
S41. Was ist ein Erwartungswert?
--------------------------------

Ein Ensemble ordnet jeder (genügend 'guten') Grösse f
einen Erwartungswert <f> zu. Damit der Name Erwartungswert
gerechtfertigt ist, muss die Abbildung f --> <f>
linear und monoton sein,
<alpha f + beta g> = alpha <f> + beta <g> für alpha, beta in C,
<f> <= <g> falls f <= g
und eine Stetigkeitsbedingung genügen.

Das Standardbeispiel, das diesem Begriff den Namen gibt,
ist der Mittelwert aus einer Anzahl von Realisierungen
einer Zufallsvariable.

Aber wie überall in der Mathematik deckt ein namensgebendes
Beispiel selten das ganze Spektrum der Anwendungen ab.
Man darf sich also unter einem abstrakten Ensemble nicht
ein konkretes Ensemble aus vielen gleichartigen Objekten
vorstellen, sondern dies nur als oft nützliche, oft aber
auch irreführende Illustration ansehen.

Insbesondere wird das Universum in der
Thermischen Interpretation durch ein Ensemble
beschrieben, obwohl das Universum einzigartig ist,
es also insbesondere nur eins davon gibt. (Zumindest ist
es das einzige abgeschlossene physikalische System, von dem wir
jemals Kenntnis haben können! Denn Kenntnis setzt Wechselwirkung
voraus, und damit Nichtabgeschlossenheit.)


Erwartungswerte gibt es in unterschiedlichen Ausprägungen.
Der mathematische Formalismus beschreibt nicht nur die statistische
Auswertung von Versuchsreihen, sondern z.B. auch die statistische
Mechanik.

Wenn ich die Temperatur einer Tasse Tee messe, mache ich eine
einzige Messung, und erhalte eine Zahl, aus der ich dann
mit den Mitteln der Thermodynamik und bekannter Eigenschaften
des Materials 'Tee' die innere Energie ausrechnen kann. Die habe
ich also unfreiwillig mitgemessen. Die innere Energie ist aber
nach Auskunft der statistischen Mechanik der Erwartungswert
des mikroskopischen Energieoperators. Dieser Erwartungswert
genügt allen Anforderungen der Mathematik, obwohl er eine
Einzelmessung an einem Einzelsystem ist - man braucht nicht
über Hunderte Teetassen oder Hunderte von Messungen an
derselben Tasse zu mitteln.

Das großkanonische Ensemble, das hier betrachtet wird,
ist rein fiktiv, um mit statistischen Konzepten arbeiten zu
können. Es ist ebenso fiktiv wie der 6N-dimensionale Phasenraum
ein fiktiver 'Raum' ist, in dem wir denken wie in 3 Dimensionen,
um uns eine intuitive Vorstellung davon machen zu können.

Mathematische Konzepte sind ihrer Natur nach abstrakt:
Um sie verwenden zu können, ist es nicht nötig,
dass die reale Bedeutung der Konzepte dieselbe ist wie die,
in der das Konzept ursprünglich entwickelt wurde, sondern
nur, dass dieselben formalen Beziehungen erfüllt sind.


-------------------------------
S42. Was ist eine Präparation?
-------------------------------

Ich habe mir angeschaut, was Experimentatoren, insbesondere
in der Quantenoptik, tatsächlich tun, und nicht, wie sie
darüber reden.

Physiker reden oft davon, dass sie Teilchen in einem bestimmten
Zustand präpariert haben.

Was besagt die Aussage gewöhnlich? Die experimentelle Anordnung
enthält eine Primärquelle von Teilchen, über deren Zustand
man relativ wenig weiß. Durch ein System von Blenden und Filtern
wird daraus ein Teilchenstrahl gewonnen, der nur noch Teilchen
im behaupteten Zustand enthält.

Dieser Zustand (sagen wir Spin up) wird z.B. dadurch bestätigt,
dass man den Spin in up-Richtung mit einem Stern-Gerlach Experiment
misst und wirklich nur einen Fleck an der erwarteten Stelle findet.


Aus der Sicht der Thermischen Interpretation ist das
Messen des Flecks aber zunächst nur eine Messung makroskopischer
Eigenschaften des Schirms, nur indirekt mit dem objektiven Zustand
des Systems verknüpft.

Aus dieser Sicht stellt sich dieselbe Präparation so dar:
Präpariert wird die Quelle, d.h. der Strahl, aber nicht das
einzelne Teilchen.

Auf der Ebene der Quantenfeldtheorie ist ja nicht einmal klar,
ob man im Strahl Teilchen hat oder Felder.
Die Thermische Interpretation redet aber nur über
vorher mathematisch präzise definierte Objekte, darf sich also
in dieser Hinsicht nicht festlegen.

Stationäre Quellen haben einen wohldefinierten, vermessbaren
Quantenzustand, gegeben durch eine Dichtematrix. Mit dieser
Interpretation gibt es keinerlei Probleme hinsichtlich
Präparation.



-----------------------------------------
S43. Was ist eine mikroskopische Messung?
-----------------------------------------

Physiker reden viel vom Messen mikroskopischer Größen.
Aber was messen sie wirklich?

Da meine Thermische Interpretation hier eine
andere Auffassung vertritt als die Tradition, ist es wert,
letztere genauer zu betrachten.

Schauen wir uns also die Behauptung näher an, ein Physiker
habe in einem Experiment den Spin eines Teilchens gemessen.
Was ist wirklich passiert?


Gemessen wird eine makroskopische Größe, die stabil genug ist,
dass sie sich überhaupt reproduzierbar beobachten lässt.
Das ist der Roh-Messwert. Daraus wird mittels theoretischer
Überlegungen (alles Handwaving) auf den Spin des einzelnen Teilchens
geschlossen und einfach behauptet, man habe diesen Spin gemessen.

Bei der Menge an Approximationen, die von der Beschreibung von
System+Apparat+Umgebung zur gemachten Behauptung, man hätte einen
Spin gemessen, führen, und bei der Empfindlichkeit, mit der ein
kleines Quantensystem auf Störungen reagiert, ist das durchaus
hinterfragbar.

Wenn man genauer mit statistischer Mechanik nachrechnet, bekommt man
auch wirklich keine sicherere Basis für diese Behauptung, sondern nur
die Aussage, dass man im Mittel den Spin misst. Man beobachtet
auch tatsächlich ziemlich zufällige Einzelbeobachtungen und erst
im Mittel eine brauchbare, reproduzierbare Beobachtung, die als
physikalisch relevantes Faktum gelten kann.

Insgesamt kann man bei geeigneter Versuchsanordnung genug Daten sammeln,
um daraus die Dichtematrix der Quelle zu bestimmen, und damit alle
von der präparierten Quelle produzierten Erwartungswerte.

Genau das ist es, was die Quantenoptiker in ihren Experimenten auch
tun, sie wollen nicht mehr und nicht weniger über ein Quantensystem
(das heißt eine stationäre oder langsam veränderliche Quelle)
wissen.

über ein Einzelteilchen machen sie jedoch keine Aussage;
dafür interessieren sie sich auch nicht. Die physikalisch
interessante Messung ist nicht der einzelne Klick im Apparat,
sondern die Verteilung vieler Klicke.

Die Ergebnisse einer Einzelmessungen sind natürlich ganz
offensichtlich Messergebnisse - aber nicht des Teilchens,
sondern des Messapparats.

Sie geben z.B. die Position eines Zeigers oder eines Silberflecks
wieder. Die zugehörige mikroskopische Größe ist eine
massengewichtete Summe extrem vieler Atompositionen,
und was man misst, ist der Erwartungswert der Position
im Sinne der statistischen Mechanik.

Einen makroskopischen Silberfleck als 'exakte' Messung etwa des
Spins eines Teilchens zu interpretieren,
ist eine zusätzliche Annahme, die nicht wirklich
gerechtfertigt werden kann, aber für die Paradoxien
in der Quantenmechanik verantwortlich ist.

In der Thermischen Interpretation dürfen
Einzelergebnisse von sogenannten "Messungen' mikroskopischer Systeme
nicht mehr ohne Weiteres als das interpretiert werden, als was sie
gemeinhin gelten, nämlich als Aussagen über mikroskopische Größen,
sondern als das, was sie sind, als makroskopische
Erwartungswerte ausgezeichneter Größen eines
Vielteilchensystems (des Detektors). Das sind sie nämlich auf der
fundamentalen Ebene. Nur in dem Maß und mit der Genauigkeit,
mit der man theoretisch zeigen kann, dass dieses Ergebnis mit
dem Wert einer mikroskopischen Größe übereinstimmt, verdient
die Messung ihre Einstufung als Messung der mikroskopischen Größe.


-------------------------------------------------
S44. Aber man kann doch einzelne Photonen messen?
-------------------------------------------------

Es gilt als bekannt, dass man einzelne Photonen nachweisen kann,
die Einzelmesswerte aber von den Erwartungswerten abweichen.

Diese Auffassung ist aber nur dann haltbar, wenn man diese Messwerte
als gemessene Photonen interpretieren darf.

Das ist aber sehr fragwürdig. Denn auch bei einer klassischen
Modellierung des elektromagnetischen Felds tritt der Photoeffekt auf.

Es ist also offensichtlich nicht ein durch das Auftreffen eines
einzelnen Photons, bewirkter Effekt, sondern ein Artefakt,
der im Messgerät erzeugt wird.


In der Thermischen Interpretation zählt das daher
nur als Messung einer Größe des Messgeräts; diese Größe
ist wie alle Rohmessungen ein thermodynamischer Erwartungswert.

Die Theorie muss nun zeigen, ob dieser Messwert auch dem in der
Thermischen Interpretation objektiv festgelegten Wert
einer Größe des gemessenen Systems entspricht.
Die Theorie zeigt nun aber nur, dass es im Mittel
einen Erwartungswert des gemessenen Systems reproduziert,
da die Einzelheiten vom Rest des Universums abhängt.
Das gilt unabhängig davon, ob man das Licht klassisch oder
quantenmechanisch modelliert.

Daher darf man den Mittelwert vieler Blitze oder Silberpartikel
als Messung einer Eigenschaft des Feldes verstehen, nicht aber
den Einzelfall als das Auftreffen eines Photons (das es ja im
klassischen Modell nicht einmal gibt).

Für das letztere gibt es außer einem historischen Vorurteil,
das für die ganze Misere in den Grundlagen der
Quantenmechanik verantwortlich ist, nicht die geringste
Rechtfertigung.
----------------------------------------
S45. Was ist denn eigentlich ein Photon?
----------------------------------------

Die naive Vorstellung eines Photons ist das eines masselosen
Teilchens, das entlang eines Lichtstrahls mit Lichtgeschwindigkeit
dahersaust und sonst keine Eigenschaften hat, wenn es der
Experimentator nicht gerade mal zwingt, sich mit einem anderen
Teilchen zu verschränken.

Aber die Quantenoptik zeichnet ein ganz anderes Bild vom Photon.
Ein Photon ist ein kompliziertes Ding.
Selbst in einem reinen Zustand kann es eine beliebige Lösung
der homogenen Maxwellschen Gleichungen sein. (In einem unreinen
Zustand - und Photonen sind durchaus nicht immer so reinlich -
sind sie noch viel komplizierte Objekte, nämlich lineare Operatoren
auf dem Raum der homogenen Maxwellschen Gleichungen!)

Ein einzelnes Photon in einem reinen Zustand 'ist',
mathematisch gesehen, im Wesentlichen dasselbe wie eine
nichttriviale Lösung der Wellengleichung!
Das heißt, zu jeder solchen Lösung könnte man im
Prinzip ein Photon präparieren!

Zu sagen, dass ein Photon eine bestimmte Frequenz oder Richtung hat,
bedeutet schon, seinen Zustand ganz gehörig einzuschränken.
Außerdem kann es unpolarisiert, zirkulär polarisiert,
linear polarisiert, und alle möglichen Schattierungen davon sein.


Zur naiven Vorstellung gehört auch, dass man das Vorhandensein eines
Photons dadurch feststellen kann, dass man es auf dem Bildschirm,
auf den es auftrifft, blitzen sieht (oder im Photodetektor klicken,
etc.), und so die Zahl der Photonen zählen kann.

Nun wird zwar an einer Stelle ein Klick oder Fleck oder Blitz
festgestellt. Dass man dann sagt, man habe ein Photon gesehen
oder gezählt, ist aber ein Euphemismus.

Für den Photoeffekt braucht man zwar Quantenmaterie im Detektor,
aber keine Quantenstrahlung; da tut es das klassische Licht,
das ja bekanntlich eine reine Welle ist, genausogut wie
eine Photonenkanone! In klassischem Licht Photonen zählen
ist aber genauso verrückt wie in Abwesenheit einer Person
Photos von ihr zu machen!


Viele reden also von Photonen, als wüssten sie alles über sie.
Dabei sind es geheimnisvolle Objekte, deren wahre Natur erst nach
einer Vorlesung über Quantenoptik allmählich dämmert
(wenn überhaupt)! Genaueres ist nachzulesen in der Bibel der
Quantenoptiker:
L. Mandel and E. Wolf,
Optical Coherence and Quantum Optics,
Cambridge University Press, 1995.
und in etwas vereinfachter Darstellung, aber trotzdem sehr lesenswert:
U. Leonhardt,
Measuring the Quantum State of Light,
Cambridge, 1997

Mit der Thermischen Interpretation hat das aber noch
gar nichts zu tun - das bis jetzt gezeichnete Bild von den Photonen
war knochenharte Orthodoxie!

Die neue Interpretation kommt erst ins Spiel, wenn man konsistent
darüber reden will, was in einem Photonenexperiment eigentlich
objektiv passiert sein soll.

Die Kopenhagen-Interpretation verbietet uns da einfach den Mund.
''Pschscht - sonst kommen gleich Ungereimtheiten heraus!''
Mit der Folge, dass alles Mikroskopische ein gespenstisches
Wesen bekommt. Solange man es nicht misst, hat das Mikroskopische
angeblich gar keine Eigenschaften. Die entstehen also anscheinend erst,
wenn Physiker da sind, die die Kunst des Messens verstehen und die
Materie zwingen können, sich in einem Eigenzustand zu offenbaren.

Die beliebte informationstheoretische Interpretation setzt noch
eins drauf; sie behauptet sogar, dass der Zustand eines
mikroskopischen Systems vom Wissen oder Unwissen des Beobachters
abhängt!

Aber das ist offenbarer Unsinn. Die Natur schert sich bestimmt nicht
darum,was Physiker wissen oder nicht!

Sonst hätte sie, bevor der erste Physiker (oder die erste Amöbe?)
sie beobachtete, ja überhaupt keine Eigenschaften haben können!
Aber wie kann sie sich dann entwickelt und Physiker (oder Amöben)
hervorgebracht haben?


Die Thermische Interpretation macht mit diesem ganzen
Spuk ein Ende. Da lässt sich wieder alles objektiv beschreiben!
Photonen, Elektronen, und was es sonst noch an Kleinzeug gibt.
Aber nicht mehr als Teilchen auf einer schmalen Bahn, sondern als
Wolke mit einer Teilchendichte - so wie in der Chemie die Orbitale
von Molekülen, die ja auch Elektronendichten darstellen.
Und dazu gibt es noch jede Menge von verborgenen
Korrelationsfunktionen, die weitere Details offenbaren könnten,
wenn man so genau messen könnte...

Bei einem Doppelspaltexperiment quetscht sich also ein Photon
in Form einer Wolke, die die Teilchendichte beschreibt
(das, was früher Aufenthaltswahrscheinlichkeit hieß),
durch beide Spalte gleichzeitig, verändert dabei seine Form,
wird zu einer Superposition des Photons durch den linken und des
Photons durch den rechten Spalt, was sich darin äußert, dass
die Dichte zwei lokale Maxima bekommt, Mit dieser
Persönlichkeitsspaltung läuft das arme Teilchen weiter,
gerät in Verwirrung und bildet dabei in seiner Dichte ein
Interferenzmuster aus. Beim Auftreffen auf dem Schirm bekommt
das Photon einen fürchterlichen Schreck und zieht sich wieder
auf seine Ganzheit zusammen, wegen der großen Aufregung allerdings
etwas zufällig, in der Nähe eines der Maxima seines vorigen
Interferenzmusters.

Etwas weniger reporterhaft geschildert, sorgt die nichtlokale Dynamik
dafür, dass von Zeit zu Zeit proportional zur Photonendichte
ein Elektron in einen angeregten Zustand versetzt wird,
eine chemische Reaktion stattfindet, oder was immer als
Detektionsmechanismus gerade relevant ist. Dass dies
stochastisch geschieht, liegt daran, dass das Experiment
hochempfindlich auf den Rest des Universums reagiert.

Wie man dieses stochastische Verhalten auf der formalen Ebene
begründen kann, wird im FAQ im Abschnitt
''Wie erklärt sich der Zufall?'' abgehandelt. Das ist allerdings
etwas technischer und erfordert fortgeschrittene Techniken
der statistischen Mechanik.




-------------------------------------------------------------
S46. Gibt es Probleme mit Lokalität und Bells Ungleichungen?
-------------------------------------------------------------

Nein. Bells Ungleichungen sind rein kinematischer Natur und haben
mit Dynamik nichts zu tun. Siehe Section 7 und 8 in [EECQ].


Nichtlokalität der klassisch deterministischen Dynamik des
Quantenuniversums folgt daraus, dass die meisten objektiven Größen
(also Erwartungswerte) nichtlokal sind.

Die praktisch wichtigen nichtlokalen Beobachtungsgrößen
sind die Korrelationsfunktionen <F(x,s)F(y,t)^T>, wo F ein Feld
mit Erwartungswert Null und beliebig vielen Komponenten ist.
Diese hängen offenbar von Werten an zwei verschiedenen Orten
ab und sind daher nichtlokal.

Man braucht also keine Gespenster wie bei Bohm einführen,
um eine klassische nichtlokale Dynamik zu bekommen,
die mit der Quantenmechanik voll konsistent ist.



-----------------------------------------------------------------
S47. Wie vertragen sich denn objektive Messwerte und Unitarität?
-----------------------------------------------------------------

Für die Zuverlässigkeit einer realen Messung genügt es,
wenn der Messwert mit sehr hoher Wahrscheinlichkeit
im Rahmen der Messgenauigkeit den objektiven Erwartungswert
reproduziert. Und das ist wegen dem Gesetz der großen Zahlen
der Fall, auch bei unitärer Dynamik.

Die quantenmechanischen Unsicherheiten bei einer Ablesung des
makroskopischen Geräts sind winzig. Das Messgerät ist im lokalen
Gleichgewicht, die Zeigerspitze hat also approximativ eine
Dichtematrix, die einem großkanonisches Ensemble entspricht.
Man kann die Größenordnung der Fluktuationen daher nach den
Regeln der statistischen Mechanik aus der makroskopischen
Zustandsgleichung ausrechnen, und sieht, dass sie keine große
Rolle spielen.

Die relative Unsicherheit ist proportional zu N^{-1/2}, wo N die
Zahl der Teilchen in der Zeigerspitze ist. Für einen guten Zeiger
ist N ~ 10^20 oder jedenfalls nicht sehr viel kleiner.

Das ist ja das ganze Geheimnis der statistischen Mechanik.
Ohne das Gesetz der großen Zahlen wäre Thermodynamik unmöglich...

Das ist auch der Grund, weshalb die Kopenhageninterpretation
auf einem klassischen Messgerät bestehen musste - ein kleines
Quantensystem ist zum Ablesen einer Messung zu unzuverlässig,



----------------------------------------------------
S48. Wie verborgen sind die 'verborgenen Variablen'?
----------------------------------------------------

Der gewöhnliche Sprachgebrauch ist der, dass mit 'hidden variables'
klassische Variablen für Beables bezeichnet werden, die einer
deterministischen Dynamik genügen, und aus denen sich die
quantenmechanischen Vorhersagen deduzieren lassen.

Da ich die Erwartungswerte und Korrelationsfunktionen als Beables
betrachte, sind es verborgene Variablen in diesem Sinn.

Andrerseits sind es natürlich alte Vertraute der statistischen
Mechanik und daher in gewissen Sinn nicht verborgen. Aber während
sie in der orthodoxen statistischen Mechanik als Mittelwerte
über viele Realisierungen gedeutet werden, werden sie in meiner
Thermischen Interpretation als irreduzible Variablen
gedeutet, und in diesem Sinn gab es sie vorher nicht, sind sie
also verborgen gewesen.

Dass diese Doppelinterpretation möglich und sinnvoll ist, macht
gerade die Kompatibilität der neuen Interpretation mit dem
quantenmechanischen Formalismus aus.



-------------------------------------------------------
S50. Wofür steht das Fragezeichen auf S.30 von [EECQ]?
-------------------------------------------------------

Es steht als Platzhalter für Werte, die vom Experiment nicht
geliefert oder daraus berechnet werden können, aus was für Gründen
auch immer.

Z.B. weil das Gerät den Spin in x-Richtung misst, den in y-Richtung
also nicht, obwohl das auch ein f ist.
Oder weil man Personen in einem Raum zählt, und gerade
jemand hereinkommt, und man nicht weiß, ob man ihn mitzählen
soll/darf/muss.
Oder weil f eine Zeitkorrelation ist, man aber ein statisches
Experiment macht, das darüber keine Auskunft gibt. Oder weil
jemand Kaffee über das Protokoll geschüttet hat, und man
daher nicht mehr alle Werte entziffern kann. Oder... oder...

Die Algebra enthält ja sehr viele Größen, messen tut man
aber immer nur ganz wenige davon. Ausrechnen (und damit indirekt
messen) kann man daraus ein paar mehr, je nachdem, wie viele Regeln
man erlaubt. Aber das meiste bleibt ungemessen.

Ein Experiment im Sinn meiner Definition kann sowohl eine
einzelne Messung sein (wo dann fast alles den Wert ? hat),
eine Versuchsserie, in der man ein Instrument sorgfältig
kalibriert (also Verteilungsfunktionen herausfindet),
oder eine Riesenmaschinerie wie CERN, in der massenhaft
Daten produziert und ausgewertet werden. Der Unterschied liegt
eben darin, welcher Menge von Größen man auf Grund des
gemachten Experiments Werte zuordnen kann.

Reale Experimente sind natürlich nur Approximationen an
konsistent realisierte Experimente, ebenso wie reale Messungen
auch schon in der klassischen Physik nur Approximationen der
(theoretisch exakten) Größen liefern.

Dass mein rudimentärer Experimentbegriff dem wirklichen
nicht in allem gerecht wird, habe ich in der Einleitung zu
Section 8 diskutiert. Ich habe den Begriff
soweit eingeschränkt, dass man damit mathematisch etwas
anfangen kann, ohne von den Komplexitäten eines echten
Experiments Notiz nehmen zu müssen.

Ich habe nur den Aspekt eines Experiments formalisiert,
dass Experimente es erlauben, dass bestimmten Größen
Werte zugeordnet werden können.


'Vollständige' Experimente im Sinn meiner Definition sind
also Idealisierungen, außer für ganz winzige Systeme, die lange
genug stationär bleiben, um sie vollständig ausmessen zu können.

Von einen Radiergummi auf meinem Schreibtisch messe ich nicht die
Koordinaten aller Atome (obwohl die kommutieren, also laut
Kopenhagen prinzipiell gleichzeitig messbar sind; aber niemand
hat das bisher geschafft), sondern nur ein paar thermodynamische
Erwartungswerte und ein paar Angaben über Form und Lage.
Die meisten Größen, die der Radiergummi also prinzipiell hat,
bleiben ungemessen.

Das ? ist der diesen ungemessenen Grössen formal zugeordnete Wert,
damit ich nicht bei allen Formeln dazuschreiben muss, ''falls alle
Werte definiert sind'' - das würde alles unübersichtlich und
langweilig machen.



-------------------------------------------------
S51. Warum verlangt man (S1) auf S.30 von [EECQ]?
-------------------------------------------------

(S1) ist das Mindeste, was man verlangen muss. Denn
wenn man nicht mal gefahrlos verschieben und skalieren darf,
sind physikalische Messungen praktisch unmöglich -
dann machen ja schon die Wahl des Ursprungs und der Maßeinheit
eine Ortsmessung problematisch.

Für das Meiste, was Physiker mit Roh-Messwerten tun, um
daraus interessante Messergebnisse zu bekommen, braucht man
mehr Rechenregeln.

Schon um den Impuls eines Teilchens zu messen, das bei einem
Zerfallsprozess im Beschleuniger entsteht und dessen Spur
man fotografiert oder mit moderneren Detektorkammern
aufgezeichnet hat, rechnet man ja einiges: Fit der Spur an
eine Helix oder ein Kalman-Filter, dann aus mehreren solchen
Spuren den Zerfallszeitpunkt rückrechnen, dann die Tangente
an diesem Punkt bestimmen, usw. und alles noch mit einer
Sensitivitätsanalyse, um zu wissen, wie genau man geworden ist.
Ohne Regeln vorauszusetzen, wäre das alles verbotener Hokuspokus.

Eine auf den ersten Blick unverfängliche solche Rechenregel
ist etwa
(SQ1) v(f^2) = v(f)^2,
Quantenlogiker haben seit langem diskutiert, was alles sinnvoll
verlangt werden kann oder auch nicht, und was für Konsequenzen
das hat. Ich habe mir davon das herausgepickt, was sich elegant
präsentieren lässt und auf kleinem Raum viel über das
Messproblem aussagt. (z.B. also Bell-Ungleichungen.)

Mit (SQ1) fangen nun schon die Probleme an, wie die Diskussion in
Section 8 meiner Arbeit zeigt. Man darf diese Regel nicht exakt
verlangen, wenn man nicht die Quantenmechanik verwerfen will,
sondern nur in einer abgeschwächten, approximativen Form
|v(fg) - v(f)v(g)| <= Delta f Delta g
die mit der Unschärferelation kompatibel ist.

Für die Analyse makroskopischer Messungen sind diese Feinheiten
irrelevant, aber im Mikroskopischen essentiell.


http://www.mat.univie.ac.at/~neum/